10.07.2015 Views

Revista (format .pdf, 3.0 MB) - RECREAÅ¢II MATEMATICE

Revista (format .pdf, 3.0 MB) - RECREAÅ¢II MATEMATICE

Revista (format .pdf, 3.0 MB) - RECREAÅ¢II MATEMATICE

SHOW MORE
SHOW LESS

Create successful ePaper yourself

Turn your PDF publications into a flip-book with our unique Google optimized e-Paper software.

Semnificaţia formulei de pe copertă. Într-o formă concisă, formula e = −1leagăcele patru ramuri fundamentale ale matematicii: ARITMETICA - reprezentată de 1; GEOMETRIA –reprezentată de π ; ALGEBRA – reprezentată de i ; ANALIZA MATEMATICĂ – reprezentată de e.iπMembrii fondatori :Temistocle BÎRSAN Alexandru CĂRĂUŞU Adrian CORDUNEANUCătălin CALISTRU Constantin COCEA Gheorghe IUREAMembri onorifici :Acad. Constantin CORDUNEANUAcad. Radu MIRONCercet.pr. Dan TIBAProf.univ. Vasile OPROIURedactor şef :Temistocle BÎRSANRedactori principali : Gabriel POPA – coordonator al rubricilor de problemeGheorghe IUREA, Petru ASAFTEI, Maria RACUComitetul de redacţie :Dan BRÂNZEI Paul GEORGESCU Ioan SĂCĂLEANU (Hârlău)Alexandru CĂRĂUŞU Gheorghe ILIE Ioan ŞERDEAN (Orăştie)Constantin CHIRILĂ Gabriel MÎRŞANU Marian TETIVA (Bârlad)Eugenia COHAL Alexandru NEGRESCU Lucian TUŢESCU (Craiova)Adrian CORDUNEANU Dan POPESCU (Suceava) Adrian ZANOSCHIMihai CRĂCIUN (Paşcani) Florin POPOVICI (Braşov) Titu ZVONARU (Comăneşti)Paraschiva GALIANeculai ROMAN (Mirceşti)Materialele vor fi trimise la una dintre adresele: t-birsan@yahoo.com , profgpopa@yahoo.co.ukCOPYRIGHT © 2008, ASOCIAŢIA “RECREAŢII <strong>MATEMATICE</strong>”Toate drepturile aparţin Asociaţiei “Recreaţii Matematice”. Reproducerea integrală sau parţială atextului sau a ilustraţiilor din această revistă este posibilă numai cu acordul prealabil scris al acesteia.TIPĂRITĂ LA BLUE SIM&Co IAŞIBd. Carol I, nr. 3-5Tel. 0332 111021, 0721 571705; e-mail: simonaslf@yahoo.comISSN 1582 – 1765


Anul XIII, Nr. 2 Iulie – Decembrie 2011RECREAŢ II<strong>MATEMATICE</strong>REVISTĂ DE MATEMATICĂ PENTRU ELEVI Ş I PROFESORIe iπ =−1Revistă cu apariţie semestrialăEDITURA “RECREAŢII <strong>MATEMATICE</strong>”IAŞI - 2011


PETRU MINUŢ(1936 - 2011)În luna mai a acestui an a plecat dintre noi prof. univ. dr. Petru Minuţ,distins cadru didactic universitar, colaborator apropiat şi ataşat al revistei RecreaţiiMatematice.Elev şi student strălucit, după terminarea studiilor universitare a fost chemat lacatedra de algebră a Facultăţii de matematică şi fizică a Universităţii din Iaşi. Aurcat în timp şi rând pe rând treptele ierarhiei universitare până la cea mai înaltă.Ca elev, a fost un harnic rezolvitor de probleme propuse în Gazeta Matematică.Liceul l-a făcut în oraşul Piatra Neamţ, la matematică fiind îndrumat şi <strong>format</strong> deprofesorul emerit Constantin Borş – un didact desăvârşit, care a dăruit întreagasa viaţă matematicii şi învăţământului matematic. De profesorul său îşi aminteaîntotdeauna, atât în anii de studenţie cât şi mai târziu, cu recunoştinţă şi veneraţie.În anul 1998, anul stingerii profesorului C. Borş, va evoca personalitatea acestuia înnr. 3 al Gazetei Matematice, seria A, într-un articol-studiu documentat şi vibrând deemoţie în acelaşi timp.În toamna anului 1954, în urma unui concurs de admitere, am devenit colegi lasecţia matematica a Facultăţii de matematică şi fizică din Iaşi, secţie care la sfârşitulcelor cinci ani de studii a dat opt absolvenţi; cinci dintre noi au făcut cariere universitare,printre care şi regretatul Petru Minuţ.A fost apreciat şi îndrăgit de prof. dr. doc. Ion Creangă, şeful catedrei dealgebră, care i-a fost şi conducătorul ştiinţific al tezei de doctorat. Domeniile salede preferinţă au fost teoria numerelor, teoria probabilităţilor şi statistica matematică;a predat cursuri de acest fel şi a realizat lucrări în aceste domenii. Este coautor latratatul Introducere în teoria numerelor (1965), elaborat de un colectiv al catedreide algebră sub conducerea prof. dr. doc. I. Creangă, tratat deseori citat, actual şiacum.Era interesat şi preocupat de toate evenimentele importante: de la cele literaresau artistice şi până la cele politico-sociale. A fost iubitor de cunoaştere şi frumos,un democrat convins şi cu dragoste de ţară.Desigur, cele mai importante realizăriale sale sunt legate de cariera universitarăşicercetarea în domeniul matematicii. Nu este locul aici de a detalia aceste aspecte. Nevom referi numai la contribuţia avută de Petru Minuţ în matematica elementară.Pentru elevii şi profesorii din învăţământul preuniversitar, a publicat două cărţi deintroducere în teoria numerelor: Teoria numerelor. Capitole introductive (1997) şiNumere prime. Numere prime speciale (2005, în colaborare cu Cristina Simirad).A considerat bună iniţiativa editării revistei elementare Recreaţii Matematice şi apublicat în paginile acesteia articole de istoria matematicii şi teoria numerelor, precumşi probleme de aritmetică şi calculul probabilităţilor. A publicat mai multeportrete ale unor iluştri matematicieni ca: Pierre Fermat, Leonhard Euler, GustavLejeuneDirichletşiN.A.Kolmogorov, câtşialeunormatematicieniromâniconsacraţi:Grigore Moisil, Nicolae Teodorescu, Ion Creangă. De asemenea, popularizează în99


paginile revistei chestiuni de mare interes pentru elevi: ipoteza lui Goldbach, numereprime din progresii aritmetice ş. a.Cu competenţă şi pasiune a contribuit la ridicarea nivelului Recreaţiilor Matematice.A fost membru al Asociaţiei ”Recreaţii Matematice”, asociaţie care tuteleazăeditarea acestei reviste.Dispariţia profesorului Petru Minuţ este o pierdere grea pentru toţi cei care l-aucunoscut. Pentru toţi colaboratorii Recreaţiilor Matematice rămâne un exemplu demodestie, conştiinciozitate şi grijă pentru lucrul făcut. Cititori şi colaboratori, neunim într-un gând de recunoştinţă pentru ceea ce ne-a lăsat ca faptă şi de păstrare şiurmare a exemplului său de dăruire, abnegaţie şi omenie .Prof.dr. Temistocle BÎRSANLista articolelor publicate de Petru Minuţ în revista Recreaţii Matematice:• 90 ani de la naşterea prof. Ion Creangă (nr. 1/2001, pp. 1-2);• Pierre Fermat - patru secole de la naşterea sa (nr. 2/2001, pp. 4-5);• Asupra ipotezei lui Goldbach (nr. 1/2002, pp. 5-6);• Numere prime din progresii aritmetice (nr. 1/2003, pp. 15-18);• A.N. Kolmogorov - 100 de ani de la naştere (nr. 2/2003, pp. 3-4);• 200 de ani de la naşterea lui Dirichlet (nr. 1/2005, pp. 1-2);• 100 de ani de la naşterea matematicianului Grigore C. Moisil (nr. 1/2006,pp. 2-4);• 300 de ani de la naşterea lui Leonhard Euler (nr. 1/2007, pp. 1-3);• 100 de ani de la naşterea academicianului Nicolae Teodorescu (nr. 1/2009,pp. 1-2);• O extensiune a şirului Fibonacci (nr. 1/2010, pp. 27-30) (în colaborare cuCristina Simirad).100


Congresul al VII-leaal matematicienilor români de pretutindeniOraşul Braşov, cu frumoasa aşezare în apropierea Carpaţilor, celebre monumente,precum Biserica Neagră, şi valoroase tradiţii pe tărâm cultural-artistic, a fost gazdaprimitoare a celui de-Al VII-lea Congres al matematicienilor români. Precizareade pretutindeni este acum necesară, când - datorită evenimentelor din ultimajumătate de secol - peste 400 de matematicieni români, cu activitate creatoareîn acest domeniu, trăiesc în afara graniţelor României. Ei se află în toate continenteleTerrei, având concentrare mai ridicată în Europa de Vest şi America de Nord.Urmaşii spiritualiai lui Spiru Haret, Gheorghe Ţiţeica, Dimitrie Pompeiu, AlexandruMyller şi ai elevilor acestora s-au răspândit în toată lumea unde matematica secultivă şi este reprezentată în universităţi sau institute de cercetare. Numai dintrecei plecaţi de la Universitatea ”Al. I. Cuza” ne putem mândri cu 7-8 actuali profesorila câteva din marile universităţi ale lumii (Paris - 2, Stanford - 2, Univ. California,Berkeley - 1, Notre Dame Univ., Indiana - 1, Oxford - 1). Date similare sunt valabileşi pentru universităţile din Bucureşti, Cluj şi Timişoara.Congresele matematicienilor români au debutat cu cel organizat la Cluj, în anul1929, unde sufletul organizator a fost Petre Sergescu. Format la Bucureşti şi ocupândpoziţii profesorale la Bucureşti şi Cluj, şi-a încheiat cariera la Paris, în calitate depreşedinte al Academiei Internaţionale de Istoria Ştiinţei. La acest prim congresau luat parte Gheorghe Ţiţeica şi Dimitrie Pompeiu, atunci la apogeul carierei loracademice. Acest început a fost urmat de congresele de la Turnu Severin şi Bucureşti,după care a urmat o îndelungată pauză datorită evenimentelor politice din anii 1930-1945.Congresul al IV-lea a fost organizatcu mare fast şi deosebit de distinsă participarela Bucureşti, în anul 1956. Comitetul de organizare a fost alcătuit din cunoscuţimatematicieni români din acea perioadă (S. Stoilow, M. Nicolescu, N. Teodorescu,Grigore Moisil ş.a.), printreparticipanţi aflându-senumecaJ. Hadamard, A. Denjoy,W. Blaschke, L. Vietoris, B. Segre, I.N. Vekua, S. Eilenberg, E. Hille, M. Hukuhara,K. Kuratowski, T. Wazewski.Cu totul regretabil, succesul Congresului al IV-lea nu a ajutat la repetarea acesteivaloroase manifestări, Congresul al V-lea având loc după 47 de ani, la Piteşti.Am participat la congresele de la Piteşti (2003), Bucureşti (2007) şi la ultimul,care a avut loc, la Braşov (2011).Congresul al VII-lea şi-adesfăşuratlucrările înperioada29 iunie - 5 iulie, 2011,fiind organizat şi sponsorizat de către un mare număr de organizaţii şi instituţii,101


incluzând Preşedinţia României (înalt patronaj), Academia Română, Institutul deMatematică ”S. Stoilow” al Academiei Române, Societatea de Ştiinţe Matematicedin România, Universitatea Transilvania din Braşov. Congresul a fost organizat şi înparteneriatcuEuropean Mathematical Society (reprezentatădepreşedintelesocietăţii,prof. Marta Sanz - Solé, de la Universitatea din Barcelona).Au participat peste 400 de matematicieni, în majoritate din România sau plecaţidin România, comunicându-şi rezultatele cercetării lor în cele opt secţii ale congresului.AcademicianulSolomon Marcus a prezentatraportulAspects of the MathematicalResearch in Romania.În cadrul mesei rotunde Sistemul educaţional din Romînia (moderator prof. RaduGologan, preşedintele Societăţii de Ştiinţe Matematice), au participat mai mulţi vorbitori,iar D-na Marta Sanz-Solé a prezentat succint The European MathematicalSociety.Am remarcat, cu satisfacţie, prezenţa multor matematicieni, incluzând colegiromâni, de la universităţile: Paris, Roma, Tel Aviv, Princeton, Purdue, Sydney, SanDiego, Lund, Lausanne, Chişinău, Columbus, Madrid, Lyon, Syracuse, N.Y., Florida- Gainesville, Bologna, Grenoble, Teheran, Nancy, Leiden, Köln, Lille, Yaroslavl,Haifa, Hindu, Nice, Orlando (Florida), Toronto, Berlin, Skoplje, Bloomington (Indiana),Philadelphia.Comitetul local de organizare, <strong>format</strong> din prof. E. Stoica, prorector al UniversităţiiTransilvania, şi prof. E. Păltânea, decan al Facultăţii de Matematică şi In<strong>format</strong>ică,au asigurat, împreună cu matematicienii braşoveni, o reuşită desfăşurare aîntregii manifestări, inclusiv latura socială (concertul de orgă de la Biserica Neagră,spectacolul artistic cu concursul Operei din Braşov - balet, dansuri, vizite la diverseobiective).Desigur, participarea Academiei Române, prin şase membri ai Secţiei de ŞtiinţeMatematice, şi a Societăţii de Ştiinţe Matematice din Romania a contribuit la asigurareaprestigiului profesional al Congresului al VII-lea al matematicienilorromâni. Sperăm, aceasta să reprezinte o garanţie pentru organizarea şi desfăşurareacongreselor viitoare.Constantin CORDUNEANUUniversitatea din Texas (Arlington)Academia Româna102


Profesorul ION CREANGĂ – o sută de anide la naşterea saS-a scurs deja un veac de când, la 1 ianuarie1911, a văzut lumina zilei cel mai mic dintre ceişapte copii ai familiei Elena şi Luca Creangă, careîntimp aveasăfie matematicianulIon Creangă 1 ,profesor la Universitatea ”Al. I. Cuza” şi rector alacesteia.AsuprafamilieiCreangă,statornicităînNordulMoldovei (com. Adâncata, jud. Dorohoi) se abateo mare nenorocire: moare Luca Creangă pe cândmicuţul Ion avea doar un an. Familia, rămasă îngrija mamei, se mută în scurt timp în Iaşi. Întregaviaţă a lui Ion Creangă va fi legată de acest oraş,de care nu s-a despărţit şi pe care l-a iubit.Aici, în Iaşi, parcurge toate treptele de formareeducativă şi profesională: primare (Şcoala”V. Alecsandri”), secundare (Liceul Naţional) şisuperioare (Facultatea de Ştiinţe a Universităţiidin Iaşi, cu licenţă în matematici obţinută în 1931). Cu înclinaţii deosebite pentrumatematică, a beneficiat de îndrumările Silviei Creangă , sora sa mai mare, careeste prima femeie din România doctor în matematici (1925), cu o teză de geometriediferenţială sub conducerea lui Alexandru Myller. In facultatea ieşeană Ion Creangăa avut ca profesori nume de mare prestigiu: Alexandru şi Vera Myller, Octav Mayer,Simion Sanielevici, Grigore Moisil ş.a.Începecarierauniversitarăla1iunie1931,capreparatorlalaboratoruldemecanicăcondus de S. Sanielevici. Beneficiind de un concediu de studii pentru anul universitar1938-1939, pleacă la Roma pentru studii de doctorat; în iulie 1939 obţine doctoratulcu o teză de geometrie diferenţială susţinută în faţa unei comisii prezidate degeometrul Enrico Bompiani.În momentul reformei învăţământului din 1948, Ion Creangă ocupa o poziţiede profesor la Institutul Politehnic din Iaşi, după care este transferat la Universitateadin Iaşi ca şef al Catedrei de algebră de la Facultatea de matematică şi fizică nouînfiinţată. Îşi leagă destinul de catedra, facultatea şi instituţia în care activa.Ca şef al Catedrei de algebră, pe care a condus-o până în 1981, anul pensionării,profesorul Ion Creangă a încurajat direcţii de cercetare noi: cercetări operaţionale,logică matematică, lingvistică matematică etc. Sub conducerea sa şi-au susţinut doctoratulcu teme de algebră sau teoria numerelor următorii: C. Reischer, I. Enescu,P. Minuţ, D. Spulber, Gh. Radu, M. Ştefănescu, M. Guţan ş.a.Petotparcursulvieţiiactiveaavutfuncţiideînaltărăspundere: decanalFacultăţiide matematicăşi fizică(1949-1953),prorector (1953-1955)şi rector al Universităţii din1 În Recreaţii Matematice , nr. 1/ 2001, cititorul poate găsi evocarea 90 ani de la naştereaprofesorului Ion Creangă , articol semnat de Petru Minuţ – fost elev şi colaborator al acestuia.103


SPIRU HARET (1851-1912) - contribuţia ştiinţificăSpiru Haret a fost un savant multilateral: astronom, matematician, sociolog,organizator al învăţământului, promotor al culturii, patriot luminat.Contemporanii l-au elogiat cu titluri precum:mare patriot, părinte al ţărănimii, omul şcoalei,marele ministru sau marele Haret.S-a născut la Iaşi pe 15 februarie 1851 şiare înaintaşi în zona Panciu, unde o localitate senumeşte chiar Haretu. În unele materiale se dăca loc de naştere Hanu Conachi, judeţul Putna(cu capitala la Focşani). A început şcoala primarăla Dorohoi, a continuat la Iaşi şi a terminatla Bucureşti. Tatăl său a fost notar şi apoijudecător şi a trebuit să se mute de mai multe ori,în funcţie de cerinţele serviciului. Urmează liceulla Bucureşti, la ”Sf. Sava”. Din anii liceului, aurămas două cărţi de matematici redactate de viitorulsavant: o algebră şi o trigonometrie. Dupăcizelări minuţioase, publică, în 1873, cartea Elementede trigonometrie, care a servit ca manualde liceu până în anul 1928. După terminarea liceului, în 1869, Haret s-a înscris laFacultatea de Ştiinţe a Universităţii din Bucureşti, unde obţine licenţa în matematici.În 1874, obţine o bursă prin concurs şi pleacă la Paris spre a se specializa în matematici.Menţionez că, din cele patru burse acordate în acel an de Titu Maiorescu, treierau pentru astronomie (Spiru Haret, Constantin Gogu, Nicolae Coculescu). În fapt,mecanica cereasca era foarte la modă în acel moment! Se cuvine să amintesc că înaceaperioadă(1846)Le Verrierdescoperiseprincalcul(”dinvârfulcondeiului”cumleplăcea francezilor să se laude!) planeta Neptun, pe baza perturbărilor orbitei planeteiUranus. La Paris, în 1875, Haret îşi trece din nou examenul de licenţă în matematicăşi, în 1876, obţine şi licenţa în fizică. În ianuarie 1878 obţine titlul de doctor laSorbona, devenind primul român doctor în matematică la Paris. Teza sa purta titlulAsupra invariabilităţii axelor mari ale orbitelor planetare şi a fost elaboratăsub conducerealui V.A. Puiseux. Se abordaoproblemădeosebit de interesantălegatăde stabilitatea sistemului solar. După cum se ştie, mişcările planetelor în sistemulnostru solar sunt influenţate, în principal, de masa Soarelui care face ca mişcareasă se dezvolte pe cunoscutele elipse. Mişcările sunt descrise destul de bine de legilelui Kepler. Printre altele, rezultă că axele mari ale elipselor rămân neschimbate întimp. Este totuşi posibil ca mişcările să fie influenţate şi de masele fiecăreia dintreplanetele existente în sistemul solar şi atunci formele elipselor (în principal, valorileaxeloracestorelipse, dar şi aşezarealor înspaţiu) s-arputea modifica. MatematicieniiPierre Laplace (1773) şi, apoi, Louis Lagrange (1776) luaseră deja în consideraţie,cu mult înaintea lui Haret, influenţa maselor unor planete, care intervin în ecuaţiiprin dezvoltarea până la puterea întâi a forţelor perturbatoare. Rezultatul a fostcă axele mari ale elipselor pe care se mişcă panetele sunt invariabile. Chiar dacă105


se realizează dezvoltarea până la puterea a doua pentru forţele perturbatoare, teoriarămâne valabilă, aşa cum arătase Siméon Denis Poisson în 1808.Rezultatul obţinut de Haret în teza sa spune că, dacă se iau în studiu dezvoltărilepână la puterea a treia ale forţelor perturbatoare, apar nişte variaţii seculare carecontrazic rezultatele obţinute până atunci, când se luau în consideraţie doar dezvoltărilepână la puterea cel mult doi. La susţinerea tezei lui Haret a participat şiHenri Poincaré, proaspăt absolvent, care a apreciat cu ”mare uimire” rezultatele dinteză. Cercetările lui Haret au fost citate de către Félix Tisserand în al său Traité demécanique céleste care recomanda extinderea metodei la cazul planetei Saturn pentruverificarea rezultatelor lui Le Verrier relativ la perturbaţiile acestei planete. Maitârziu, 1955, cercetările au fost reluate de J. Meffroy prin utilizarea unor concepte şitehnici noi de calcul.Problema stabilităţii axelor mari ale traiectoriilor planetelor a constituit unul dintestele relativitaţii generale, alături de curbarea semnalului luminos în vecinătateaunui corpmasiv şi de deplasareaspreroşu înspectrul unui semnal luminos provenitdela o stea masivă. Concret, este vorba de cunoscuta problemă a avansului periheliuluiunei planete. Se ştia deja (Le Verrier, 1859) că elipsele pe care se mişcă planeteledescriu ele însele o mişcare de avansare. Acest fapt se pune în evidenţă cu ajutorulpunctului cel mai apropiat de Soare, periheliul, care nu e fix în spaţiu (aşa cum artrebui!), ci are o tendinţă de avansare către sensul din care vine planeta. Acest lucrueste perceput cel mai bine în cazul planetei Mercur care are o perioadă destul de micăde revoluţie (88 zile). Aşa că diversele avansuri ce apar la fiecare rotaţie se sumeazăcel mai adesea. În 100 de ani, periheliul lui Mercur a avansat cu circa 574 de secunde.Influenţa maselorcelorlalte planete a putut justifica doar 532 secunde din acest avans,restul de 42,9 secunde rămânând ca o enigmă pentru mecanica cerească. Pentru operioadă, s-a avansat ideea existenţei unei planete nedescoperite, care a fost chiarşi botezată Vulcan şi care ar fi fost responsabilă de avansul ce nu putea fi explicat.În urma elaborării teoriei relativităţii generale de către A. Einstein, în 1915, şi agăsirii soluţiei Schwarzschild, în 1916 (sub forma unei metrici pseudoriemanniene învarietatea spaţiu-timp), s-a putut explica cealaltă parte din avans pornind de la ideeacă traiectoriile planetelor sunt geodezice în metrica Schwarzschild.Mai târziu, unele rezultate au fost extinse şi aplicate pentru calculul orbitelorsateliţilor artificiali ai Pământului. S-a constatat că, chiar forma Pământului influenţeazătraiectoriilesateliţilorsăiartificiali.Înlegăturăcuaceastamenţionezcontribuţialui Eugen Grebenikov, membru înechipasovieticăceaefectuat calculelepentru traiectoriilesateliţilor în anii ′ 50 şi ′ 60. Au existat dificultăţi de calcul şi de concepţieîntâmpinateîntimpuldesfăşurăriioperaţiilordelansare. AşacumpovesteaGrebenikovîntr-o conferinţă ţinută în 2010 în aula Academiei din Iaşi, una din dificultăţi provinedin faptul că Pământul este turtit la poli. Diferenţa între raza de la ecuator şi razade la poli este de circa 18 km. Grebenikov menţiona că lucrurile ar fi fost mai simpledacă Pământul ar fi fost alungit (sub forma unui pepene şi nu ca un dovleac!).Bineînţeles, multe lucruri ar fi fost mai simple dacă s-ar fi dispus de tehnica de calculexistentă în momentul actual. În conferinţa sa, Grebenikov se plângea de acest lucruşi vorbea de embargoul pus de lumea occidentală în privinţa tehnicii de calcul.Menţionez că numele lui Spiru Haret a fost dat unui crater de pe partea nevăzută106


a Lunii, iar numele lui Grebenikov a fost dat unui asteroid din puzderia de asteroizice roiesc undeva între Jupiter şi Saturn. Există vreo 13 nume de români purtate deasteroizi, comete sau de cratere de pe diverse planete: Brâncuşi, Eminescu, Enescu,Elena Văcărescu, Victor Daimaca, Nicolae Sanduleak, Constantin Pârvulescu, HermannOberth, E. Grebenikov, Nicolae Donici, Mirel Bîrlan, Jean Dragesco etc.Deşi ocupat cu activităţi de interes obştesc (a fost ministru al educaţiei de trei ori,în 1897-1899, în 1901-1904 şi în 1907-1910, realizând o reformă substanţială a învăţământului,prin care s-a constituit sistemul de educaţie modern), Haret a mai publicatcâteva articole de astronomie: despre acceleraţia seculară a Lunii (1880) şi desprepata roşie de pe Jupiter (1912) şi meteorul luminos de la 29 noiembrie 1911 (1912).Una din lucrările sale cele mai interesante este monografia Mecanica socială(1910, Paris-Bucureşti), încare încearcă să aplice principiile mecanicii la viaţa socială- fiind astfel un precursor al utilizării metodelor matematice în studiul fenomenelorsociale. Rezultatele din aceastămonografieaustârnitcontroverseaprige, dovedindcă,prinuneleconcepteşiprinunelemetodedecercetare,SpiruHareteracumult înainteatimpului său. Prin această carte, Haret a încercat să pună la punct metoda modelariişi concepţiaprin cares-arputea descrieşi evaluaforţele ce guvernează înmod obiectivfenomenele sociale şi economice. A extins la viaţa societăţii principiul minimei acţiunişi a folosit probabilitaţile şi, inevitabil, statistica matematică în analiza sociologică.A elaborat şi unele lucrări cu aplicaţii practice: Despre măsura capacitaţii buţilor(1878), Consideraţii relative la studiul experimental al mişcării apei în canale descoperiteşi la constituţia internă a fluidelor (1882), Teorema ariilor în mişcarea sistemelormateriale (1894), Notă asupra poporaţiunii României (1903) etc.În 1892 a devenit membru al Academiei (membru corespondent devenise încădin 1879). Îmbinând munca de cercetare ştiinţifică cu cea de organizare şi de popularizarea ştiinţei, Haret a lăsat în urma sa şi numeroase manuale de şcoală.Mai menţionăm şi eforturile intense depuse de Spiru Haret pentru înfiinţareaObservatorului Astronomic din Bucureşti, realizată în 1908.Bibliografie1. G. Şt. Andonie – Istoria Matematicii în România, vol. I, Ed. Ştiinţifică, Bucureşti,1965; Spiru Haret, 213-223.2. M. Stavinschi – Un nume românesc pe harta Lunii: Spiru Haret (15 februarie 1851- 17 decembrie 1912), Astronomical Institute of the Romanian Academy.3. M. Stavinski, V. Mioc – Astronomical Researches in Poincaré ′ s and RomanianWorks,AstronomicalInstituteoftheRomanianAcademy,2004. (http://syrte.obspm.fr/ journees2004/PDF/Stavinschi.<strong>pdf</strong>)4. Spiru Haret – Biography at Mac Tutor History of Mathematics archive, St. AndrewsUniversity.5. Spiru Haret – Wikipedia, the free encyclopedia.6. L. Modan – Spiru Haret - reper al spiritualităţii româneşti, Gazeta Matematică (A),anul XIX (XCVII), 2001, 113-118.107Prof.dr. Vasile OPROIU


Alte proprietăţi caracteristice triunghiuluiechilateralTitu ZVONARU 1 , Neculai STANCIU 2Abstract. This article is a continuation of the paper [3]. We obtain several other characterizationsof the equilateral triangle.Keywords: orthocenter, centroid, incenter, Nagel point, Gergonne point, Spiecker point.MSC 2000: 51M04.Scopul lucrăriidefaţă estegăsireaunorproprietăţi caracteristicetriunghiului echilateral,pe linia celor stabilite în [3]. Vom folosi aceleaşi notaţii: astfel, punctului H(ortocentru) îi corespunde △A H B H C H (triunghiul pedal determinat de punctul H)având punctele importante, cu semnificaţiile evidente, O H ,G H ,H H ,I H etc. Ca şi în[3], avem în vedere numai triunghiuri ascuţitunghice. Vom rezolva câteva dintre problemelede tipul: dacă două puncte importante (luate din diferite triunghiuri pedale)coincid, atunci △ABC este echilateral. O atenţie deosebită va fi acordată punctuluilui Spiecker al triunghiului ABC – centrul cercului înscris întriunghiul median, adică,în notaţiile convenite, punctul I G .Procedeul utilizat decurge din observaţia: punctele X şi Y coincid dacă şi numaidacă picioarele cevienelor duse prin aceste puncte coincid (este suficientă coincidenţapicioarelor numai pe două laturi ale triunghiului). Vom pune acum la punct instrumentulde lucru; mai întâi, reproducem un rezultat din [4]:Lemă. În triunghiul ABC considerăm ceviana AD, cu D ∈ (BC). Dacă o secantăintersectează laturile AB,AC şi ceviana AD în punctele M,N, respectiv P, atuncisunt adevărate următoarele două relaţii:AMAB · BDDC · ACAN · PNPM = 1; (R 1)APPD =BC · AM<strong>MB</strong> · ANNCBD · AM AN+DC ·<strong>MB</strong> NC. (R 2 )Problemă. Fie ABC un triunghi şi punctele M,N,P pe laturile BC,CA respectivAB astfel încât BMMC = k 1, CNNA = k 2, APPB = k 3. În triunghiul MNP considerămcevienele MM ′ , NN ′ , PP ′ concurente în punctul Q (M ′ ∈ (NP),N ′ ∈ (PM),P ′ ∈(MN)), cu M′ NM ′ P = p N ′ P1,N ′ M = p 2, P′ MP ′ N = p 3. Dacă {A ′ } = AQ ∩ BC, să secalculeze raportul BA′A ′ C .Soluţie. Conform teoremei lui Ceva avem p 1 p 2 p 3 = 1. Notăm {S} = AA ′ ∩PN,{T} = AA ′ ∩MN şi SPSN= t,BA′A ′ Csimilare). Rezultă uşor că A′ MA ′ C = A′ B −BMA ′ C= x (dacă AQ intersectează [PM], calculele sunt= x− k 1 ·BCk 1 +1 · x+1BC = x−k 1k 1 +1 .1 Comăneşti, e-mail: tzvonaru@yahoo.com2 Profesor, Şcoala generală ”George Emil Palade”, Buzău, e-mail: stanciuneculai@yahoo.com108


Conform relaţiei (R 1 ), avem APAB · A′ BA ′ C · ACAN ·SNSP = 1 ⇔ k 3k 3 +1 ·x·(k 2 +1)· 1 = 1, adicăt(1)tx = k 3(k 2 +1)k 3 +1 .Cu relaţia (R 2 ), deducem că(2)QNQN ′ =PM · SNSP · TNTMPN ′ · SNSP +N′ M · TNTMPNA1 B M= t · TNTMp 2p 2 +1 · 1t + 1p 2 +1 · TN ,TMM NSPQTACiar cu relaţia lui Van Aubel avem QNQN ′ = NM′M ′ P + NP′P ′ M = p 1 + 1 = p 1 + p 1 p 2 =p 31 2p 1 (p 2 +1) şi atunci (2) se scrie succesiv: p 1 (p 2 +1)·p 2 +1pt + TM= TN 1 t · TNTM ⇔p 1 p 2t(3)+p 1 · TNTM = 1 t · TNTM , deciTMTN = 1−p 1t.p 1 p 2Cu teorema lui Menelaus în △MNC şi transversala A−T −A ′ obţinem TMTN · ANNC ·A ′ CA ′ M = 1 sau(4)1−p 1 t 1·p 1 p 2 k 2 +1 · k1 +1= 1.x−k 1Înlocuind în (4) pe t dat de (1), se obţine o ecuaţie de gradul I cu necunoscuta x,având soluţia(∗) x = k 3 +1k 2 +1 · k 1 p 1 p 2 (k 2 +1)+(k 1 +1)p 1 p 2 (k 3 +1)+k 3 p 1 (k 1 +1) .Dacă X este un punct important în △A Y B Y C Y , vom nota cu x XY raportul încare dreapta AX împarte latura BC a triunghiului ABC. În cazul punctului luiSpiecker, I G , avem k 1 = k 2 = k 3 = 1 şi p 1 = c b , p 2 = a c , p 3 = b ; deci, utilizând relaţiaa(∗), obţinem x IG = a+ba+c .Propoziţia 1. Dacă I G ≡ G I (i.e. punctul lui Spiecker coincide cu centrul degreutate al triunghiului pedal al punctului I), atunci △ABC este echilateral.109


Demonstraţie. Relativ la punctul G I avem k 1 = c b , k 2 = a c , k 3 = b a şi p 1 =p 2 = p 3 = 1; cu relaţia (∗), obţinem x GI = c(a+b)(a+b+2c)b(a+c)(a+2b+c) . Dacă I G ≡ G I ,a+bîn mod necesar x IG = x GI , i.e.a+c = c(a+b)(a+b+2c)b(a+c)(a+2b+c) ⇔ ab + 2b2 + bc =ac + bc + 2c 2 ⇔ (b − c)(a + 2b + 2c) = 0, deci b = c. Analog deducem a = b, decia = b = c şi △ABC este echilateral.Propoziţia 2. Dacă I G ≡ G H (i.e. punctul lui Spiecker coincide cu centrul degreutate al triunghiului ortic al △ABC), atunci △ABC este echilateral.Demonstraţie. Relativ la G H avem k 1 = ccosBbcosC , k 2 = acosCccosA , k 3 = bcosAacosAşi p 1 = p 2 = p 3 = 1; cu relaţia (∗), după calcule de rutină, obţinem x GH =c 2 (acosA+bcosB)b 2 (acosA+ccosC) . Avem: x IG = x GH ⇔ a+ba+c = c2 (acosA+bcosB)b 2 (acosA+ccosC) ⇔ [a2 (b 2 −c 2 ) + a(b 3 − c 3 )]cosA + bc(b 2 − c 2 ) + bc2a [a2 (b − c) + b 3 − c 3 + bc(b − c)] = 0 (s-auînlocuit cosB şi cosC prin expresiile date de teorema cosinusului) şi rezultă b = c.Analog deducem a = b, deci △ABC este echilateral.Propoziţia 3. Fie Γ punctul lui Gergonne al △ABC. Dacă I G ≡ G Γ (i.e.punctul lui Spiecker coincide cu centrul de greutate al triunghiului pedal al punctuluiΓ), atunci △ABC este echilateral.Demonstraţie. Pentru Γ avem k 1 = p−bp−c , k 2 = p−cp−a , k 3 = p−ap−b , p 1 = p 2 =p 3 = 1 şi obţinem x GΓ = c b · a(p−a)+b(p−b) . Avem succesiv:a(p−a)+c(p−c)x IΓ ≡ x GΓ ⇔ a+ba+c = 2abc+ac2 +bc 2 −a 2 c−b 2 c2abc+ab 2 +b 2 c−a 2 b−bc 2 ⇔⇔ (b−c)(4abc−a 3 +ab 2 +abc+ac 2 +b 2 c+bc 2 ) = 0.Deoarece 2abc + ab 2 + ac 2 = a(b + c) 2 > a 3 (din inegalitatea triunghiului), rezultăb = c. Analog deducem a = b şi concluzia dorită.Propoziţia 4. Fie N punctul lui Nagel. Dacă I G ≡ G N (i.e. punctul lui Spieckercoincide cu centrul de greutate al triunghiului pedal al punctului N), atunci △ABCeste echilateral.Demonstraţie. În cazul punctului N, k 1 = p−cp−b , k 2 = p−ap−c , k 3 = p−bp−a ,p 1 = p 2 = p 3 = 1 şi, ca urmare, x GN = c(p−c)(a+b)b(p−b)(a+c) . Atunci x IG ≡ x GN ⇔a+ba+c = c(p−c)(a+b) ⇔ b(p−b) = c(p−c) ⇔ (b−c)(b+c−a)= 0, de unde b = c.b(p−b)(a+c)Analog deducem că a = b şi, deci, concluzia dorită.110


Propoziţia 5. Dacă H G ≡ G H (i.e. ortocentrul triunghiului median coincide cucentrul de greutate al triunghiului ortic), atunci △ABC este echilateral.Demonstraţie. Valoarea raportului x GH a fost stabilită în Propoziţia 2. Pentrupunctul H G avem k 1 = k 2 = k 3 = 1, p 1 = ccosBbcosC , p 2 = acosCccosA , p 3 = bcosAacosB şiobţinem x GH = ccosCbcosB . Atunci,x HG = x GH ⇔ cosCcosB = c(acosA+bcosB)b(acosA+ccosC) ⇔⇔ (abcosC −accosB)cosA+bc(cos 2 C −cos 2 B) = 0 ⇔⇔ (a 2 +b 2 −c 2 −a 2 +b 2 )cosA+2bc(cosC −cosB)(cosC +cosB) = 0 ⇔⇔ 2(b 2 −c 2 )cosA+2bca 2 +b 2 −c 2− a2 +c 2 −b 2 (cosC +cosB) = 0 ⇔2ab 2ac⇔ 2(b 2 −c 2 )cosA+ 1 (b−c)(b+c−a)(b+c+a)(cosC +cosB) = 0,adeci b = c. Analog deducem a = b şi rezultă că △ABC este echilateral.Observaţii. 1) Formula (*) este utilă în situaţiile în care laturile triunghiuluipedal se calculează relativ uşor sau nu este nevoie de ele (p 1 = p 2 = p 3 = 1). Dacăp 1 ,p 2 ,p 3 depind de laturile triunghiului şi acestea sunt greu de calculat (cum ar fi,de exemplu, cazul △A O B O C O sau al △A I B I C I ), calea descrisă mai sus nu poate fiurmată.2) Cu formula (∗) se demonstrează uşor că △ABC este echilateral în următoarelesituaţii:a) Punctul Nagel al triunghiului median este centrul cercului înscris în triunghiuldat ([1]);b) Punctul lui Lemoine al triunghiului de contact este punctul lui Gergonne altriunghiului dat ([2]);c) Izotomicul ortocentrului triunghiului median este punctul lui Lemoine al triunghiuluidat ([3]).Bibliografie1. N. Altshiller-Court – College Geometry, Barnes & Noble Books, New-York, 1968.2. G. Mihalescu – Geometria elementelor remarcabile, Editura Tehnică, Bucureşti,1957.3. F. Toma – Proprietăţi caracteristice triunghiului echilateral, Recreaţii Matematice,1/2011, 17-19.4. T. Zvonaru, B. Ioniţă – Rapoarte determinate de o ceviană şi o secantă într-untriunghi, Recreaţii Matematice, 1/2005, 15-17.111


Inegalităţi geometrice în poligoane convexe,de tip Bergström-MitrinovičDumitru M. BĂTINEŢU-GIURGIU 1 , Neculai STANCIU 2Abstract. Some Mitrinovič type inequalities for general convex polygons are presented. Themain tool in the proofs is Bergström inequality.Keywords: Mitrinovič type inequalities, Bergström inequality, convex polygon.MSC 2000: 51Mxx, 26D15.Inegalitatea lui Bergström are următorul enunţ: Dacă n ∈ N ∗ − {1}, x k ∈ R,y k ∈ R ∗ +, ∀k = 1,n, X n =nk=1x k , Y n =nk=1y k , atunci:(B)nk=1x 2 ky k≥ X2 nY n,cu egalitate dacă şi numai dacă există t ∈ R ∗ astfel încât x k = ty k , ∀k = 1,n.Inegalitatea luiD.S.Mitrinovič areurmătorulenunţ: În orice triunghi de perimetru2p, circumscris unui cerc C(I;r), are loc inegalitatea:(M) p ≥ 3r √ 3,cu egalitatea dacă şi numai dacă triunghiul este echilateral.Scopul acestui articol este de a stabili unele inegalităţi geometrice (altele decâtcele din [1]) de tipul (M) în poligoane convexe, folosind inegalitatea (B).Pentru orice poligon convex A 1 A 2 ...A n , n ≥ 3, vom nota cu S aria poligonului,cu 2p perimetrul poligonului, cu a k lungimea laturii [A k A k+1 ],k = 1,n, A n+1 ≡ A 1 ,iar pentru orice punct M interior poligonului notăm T k = pr Ak A k+1M, d k = MT k ,u k = µ(∠A k MT k ), v k = µ(∠T k MA k+1 ), S k = aria[A k MA k+1 ],∀k = 1,n.Lemă. Fie A,B,A ≠ B două puncte în plan şi M /∈ AB, cu T = pr AB M ∈ [AB];atunci AB d = tgu+tgv, unde u = µ(AMT), v = µ(T<strong>MB</strong>) (în radiani), iar d estedistanţa de la M la dreapta AB.Demonstraţie. Avem următoarele situaţii:i) T ∈ (AB). Atunci tgu = ATMTii) T ≡ A (analog T ≡ B). Găsim tgu = 0 şi tgv = BTBT ABşi tgv = , deci tgu+tgv =MT MT ., deci tgu+tgv =ABMT .MTTeorema 1. Dacă A 1 A 2 ...A n , n ≥ 3, este un poligon convex şi M este un punctinterior lui astfel încât pr Ak A k+1M = T k ∈ [A k A k+1 ], ∀k = 1,n, A n+1 ≡ A 1 , atunci(1)nk=1a kd k≥ 2ntg π n .1 Profesor, Colegiul Naţional ”Matei Basarab”, Bucureşti2 Profesor, Şcoala Generală ”George Emil Palade”, Buzău112


nÈk=1Demonstraţie. Conform Lemei avem a k= tgu k + tgv k , ∀k = 1,n, de unded ka k= nÈk=1(tgu k +tgv k ).d kDeoarece funcţia f :0, π f(x) = tgx este convexă pe0,2→R, π rezultă2,că putem aplica inegalitatea lui Jensen şi obţinem nÈk=12ntg12n(u k +v k ). DinnÈk=1nÈk=1(u k +v k ) = 2π, deducem că nÈk=1a k= nÈk=1(tgu k + tgv k ) ≥d ka kd k≥ 2ntg 2π2n =2ntg π , ceea ce era de demonstrat.nObservaţia 1.1. Dacă poligonul A 1 A 2 ...A n este circumscris unui cerc C(I;r)şi M ≡ I, rezultă că d k = r, ∀k = 1,n, iar (1) devine(1 ′ )1rnk=1a k = 2pr ≥ 2ntg π n ⇔ p ≥ nrtg π n .Inegalitatea (1 ′ ) este o generalizare a inegalităţii (M).Observaţia 1.2. În cazul în care poligonul este un triunghi ABC, relaţia (1)devine(1 ′′ )Pentru M ≡ I obţinem inegalitatea (M).ad a+ b d b+ c d c≥ 6tg π 3 = 6√ 3.Teorema 2. Dacă A 1 A 2 ...A n , n ≥ 3, este un poligon convex şi M este un punctinterior poligonului, atunci(2)nk=1a kd k≥ 2p2S .Demonstraţie. Avem nÈk=1a kd k= nÈk=1a 2 ka k d k= nÈk=1a 2 k2S k. Aplicăm inegalitatea (B)( nÈk=1aşi obţinem nÈk=1k ) 2a k≥d k2 nÈk=1= 4p22S = 2p2 , ceea ce era de demonstrat.SS kObservaţia 2.1. Dacă A 1 A 2 ...A n , n ≥ 3, este circumscris cercului C(I;r),atunci S = pr, iar (2) devine nÈk=1a k≥ 2p2d k pr = 2pr ; apoi din (1′ ) deducem că(2 ′ )nk=1a kd k≥ 2ntg π n , ∀M ∈ IntA 1A 2 ...A n .113


Prin urmare, ∀M ∈ IntABC are loc inegalitatea(2 ′′ )ad a+ b d b+ c d c≥ 2pr ≥ 6tg π 3 .Teorema 3. Dacă A 1 A 2 ...A n , n ≥ 3, este un poligon convex şi M este un punctinterior lui astfel încât pr Ak A k+1M = T k ∈ [A k A k+1 ], ∀k = 1,n, A n+1 ≡ A 1 , atunci:(3)a kdnk=12 k≥ 2n2p tg2 π n .Demonstraţie. Avem nÈk=1a knÈk=1 d 2 k≥( nÈk=1a kd k) 2= 1a2pnÈk=1 knÈk=1a kd 2 k= nÈk=11a ka kd k2, unde aplicăm (B) şi obţinema k, de unde, folosind (1), deducem (3).d k2Observaţia 3.1. Dacă poligonul A 1 A 2 ...A n , n ≥ 3, este circumscris cerculuiC(I;r) şi M ≡ I, atunci (3) devine:(3 ′ ) 2p 2 ≥ 2n 2 r 2 tg 2 π n ⇔ p2 ≥ n 2 r 2 tg 2 π n .Teorema 4. Dacă A 1 A 2 ...A n , n ≥ 3, este un poligon convex şi M este un punctinterior lui astfel încât pr Ak A k+1M = T k ∈ [A k A k+1 ], ∀k = 1,n, A n+1 ≡ A 1 , atunci:(4)nk=1Demonstraţie. Avem nÈk=1a kşi obţinem nÈk=11 d k2≥nÈk=1d 2 kanÈk=12 kcont de (1), deducem concluzia.a 2 knk=11d 2 k= nÈk=11d 2 k≥4n 2 tg 2 π n .1a 2 ka kd k2, unde aplicăm inegalitatea (B)a ⇔nÈk=12 knÈk=11 a kd 2 , apoi, ţinândk≥nÈk=1 d k2Observaţia 4.1. Dacă poligonul convex A 1 A 2 ...A n , n ≥ 3, este circumscriscercului C(I;r), iar M ≡ I, atunci d k = r, ∀k = 1,n, şi relaţia (4) devine(4 ′ )nr 2nk=1a 2 k≥4n 2 tg 2 π nk=1an ⇔ 2 k ≥ 4nr2 tg 2 π n .Teorema 5. Dacă un poliedru convex are n(n ≥ 4) feţe poligoane convexe de ariiS k (k = 1,n), iar M este un punct interior poliedrului cu distanţa d k la faţa de arie114


S k şi V,S sunt volumul şi respectiv aria totală ale poliedrului, atunci:(5)nk=1S kd k≥ S23V .Demonstraţie. Avem nÈk=1S k= nÈk=1Sk2 = nÈk=1Sk2 , unde V k este volumuld k d k S k 3V kpiramidei de vârf M şi bază poligonul feţei de arie S k . Aplicând inegalitatea (B),deducem că nÈk=1S k2S k≥nÈk=1d k3 nÈk=1V k= S2, ceea ce era de demonstrat.3VObservaţia 5.1. Dacă poliedrul este circumscris unei sfere S(I;r) de centru I şirază r atunci relaţia (5) devine:(5 ′ )nk=1S kd k≥ S23V = S2Sr = S r .Dacă M ≡ I, atunci d k = r, ∀k = 1,n, şi relaţia (5 ′ ) devine o egalitate.Teorema 6. În condiţiile teoremei 5, are loc inegalitatea:1d 2 k≥ S49V 2.(6) Snk=1knk=12Demonstraţie. Avem nÈk=11d 2 = nÈk=1 kS kdeducem nÈk=11 d k2≥nÈk=1. Dacă ţinem seama de nÈk=1d 2 kSknÈk=12şi aplicăm din nou (B), atunci S obţinemnÈk=1knÈk=12ceea ce era de demonstrat.1S 2 kS kd k2, unde aplicăm inegalitatea (B) şiS kd k= nÈk=1S 2 kd k S k= nÈk=1S 2 k3V k1 S kd 2 ≥k≥nÈk=1 d k2S49V 2,Observaţia 6.1. Dacă poliedrul este circumscris unei sfere S(I;r) şi M ≡ I,atunci relaţia (6) devine n rnÈk=12 Sk 2 ≥ S49V 2 ⇔ nÈk=1Sk 2 ≥ r2 S 49nV 2, apoi, ţinând seama cărS = 3V, obţinem(6 ′ )nk=1S k 2 ≥ r2 S 4nr 2 S 2 = S2n .Bibliografie1. M. Dincă, M. Bencze – Trip in world of geometrical inequalities (2), OctogonMathematical Magazine, vol 11, No. 1, april 2003, 45-76.115


În legătură cu o problemă de concursDan Ştefan MARINESCU 1 , Ioan ŞERDEAN 2Abstract. Proposition 2, the main result of the paper, extends some problems which haveappeared in journals or contests of elementary mathematics.Keywords: derivatives, convex functions.MSC 2000: 26A51, 26A42.1. Introducere. La Olimpiada Naţională de Matematică din anul 2005 autoriiacestei note au propus următoarea problemă:P1. Fie f : [0,1] → R o funcţie care verifică condiţiile |f(x) − f(y)| ≤ |x − y|,(∀)x,y ∈ [0,1] şi1f(x)dx = 0.a) Să se arate că |xb) Dacă12000f(t)dt| ≤ 1 x(1−x), (∀)x ∈ [0,1],2f(x)dx = 1 , să se determine f.8Enunţul acestei probleme a apărut în Lista scurtă a Olimpiadei Naţionale deMatematică 2005 şi în R.M.C. 2005. În anul 2007 la cunoscutul Concurs studenţescde matematică Putnam din U.S.A. s-a propus problemaP2. Fie f : [0,1] → R o funcţie derivabilă cu derivata continuă şi1f(x)dx = 0.Atunci, pentru orice α ∈ [0,1] are loc inegalitatea |αf(x)dx| ≤ 10 8 max0≤x≤1 |f′ (x)|.În mod certP2se deduce din P1. Într-adevăr,fie M = max0≤x≤1 |f′ (x)|. DacăM = 0,atunci f este constantăpe [0,1]şi cum1f(x)dx = 0 deducem că f este funcţia nulă,0iar inegalitatea este evidentă în acest caz. Dacă M ≠ 0, atunci funcţia g : [0,1] → R,g(x) = 1 f(x)verificăipotezeleproblemeiP1,deunde|1g(x)dx| ≤ 1 M 2 α(1−α) ≤ 1 8 ,(∀)α ∈ [0,1] şi, în concluzie, |α0f(x)dx| ≤ 1 8 max0≤x≤1 |f′ (x)|.Tot legat de problema P1, să remarcăm faptul că în numărul din martie 2011 alrevistei americane The College Mathematics Journal apare sub semnătura lui DuongViet Thong următoarea problemă:P3. Fie f : [a,b] → R o funcţie derivabilă cu derivata continuă şibf(x)dx = 0.Demonstraţi că |baf(t)dt| ≤ M (x−a)(b−x) , (∀)x ∈ [a,b],unde M = max2x∈[a,b] |f′ (x)|.1 Profesor dr., Colegiul Naţional ”Iancu de Hunedoara”, Hunedoara2 Profesor, Colegiul Naţional ”Aurel Vlaicu”, Orăştie00a116


2. Rezultatul principal. Pentru scopul propus avem nevoie de câteva rezultatecunoscute relativ la funcţiile convexe. O funcţie f : I → R, I ⊂ R interval nedegenerat,esteconvexădacăf(ta+(1−t)b)≤ tf(a)+(1−t)f(b), (∀)a,b ∈ I,(∀)t ∈ [0,1].Propoziţia 1. (i) f : I → R este convexă dacă şi numai dacă (∃)m,n ∈ R astfelîncât funcţia g : I → R definită prin g(x) = f(x)+mx+n, (∀)x ∈ I, este convexă.(ii) Dacă f este convexă, a,b ∈ I, a < b, şi f(a) = f(b) = 0, atunci f(x) ≤ 0,(∀)x ∈ [a,b].(iii) Dacă f convexă şi există a,b ∈ I, a ≠ b şi t 0 ∈ (0,1) astfel încât f(t 0 a+(1−t 0 )b) = t 0 f(a)+(1−t 0 )f(b), atunci f(ta+(1−t)b) = tf(a)+(1−t)f(b), (∀)t ∈ [0,1].(iv) Dacă f este derivabilă, atunci f este convexă dacă şi numai dacă f ′ estecrescătoare.Demonstraţie. (i) Se aplică faptul că orice funcţie afină (de gradul întâi)verificădefiniţia funcţiei convexe cu egalitate.(ii) Fie x ∈ [a,b]. Atunci există t ∈ [0,1] astfel încât x = ta+(1−t)b şi, cum feste convexă, deducem că f(x) = f(ta+(1−t)b) ≤ tf(a)+(1−t)f(b) = 0.(iii) Din f convexă şi din faptul că orice funcţie afină verifică definiţia convexităţiicu egalitate deducem că funcţia ϕ : [0,1] → R, ϕ(t) = f(ta+(1−t)b)−tf(a)−(1−t)f(b), este convexă, iar ϕ(0) = ϕ(t 0 ) = ϕ(1) = 0 şi ϕ(t) ≤ 0, (∀)t ∈ [0,1]. Dacă arexista t ′ ∈ (0,1)\{t 0 } astfel încât ϕ(t ′ ) < 0, din convexitate s-ar obţine o contradicţie.Într-adevăr, avem: (a) dacă 0 < t ′ < t 0 , atunci există s ∈ (0,1) astfel ca t 0 =st ′ +(1−s)·1, de unde ϕ(t 0 ) ≤ s·ϕ(t ′ )+(1−s)ϕ(1) = s·ϕ(t ′ ) < 0, ceea ce este fals.(b) dacă t 0 < t ′ < 1 atunci există s 1 ∈ (0,1) astfel ca t 0 = s 1 ·0+(1−s 1 )t ′ şi seobţine iarăşi contradicţie.(iv) Fie x,y ∈ I cu x < y. Conform unui rezultat cunoscut ([1],[2]), pentru oriceu,v ∈ I cu x < u < v < y avem f(u)−f(x) ≤ f(v)−f(u) ≤ f(v)−f(y) , de undeu−x v −u v −ydeducem că f ′ (x) ≤ f ′ (y). Reciproc, fie a,b ∈ I, cu a < b şi t ∈ (0,1). Aplicândteorema lui Lagrange pe [a,ta+(1−t)b] şi pe [ta+(1−t)b,b], monotonia funcţiei f ′ne conduce la concluzie.Cititorii interesaţi de problematica funcţiilor convexe pot consulta excelentelemonografii [1] şi [2]. Rezultatul principal este cuprins înPropoziţia 2. Fie a,b ∈ R, a < b şi f : [a,b] → R o funcţie derivabilă cuproprietatea că (∃)L ≥ 0 astfel ca |f ′ (x) − f ′ (y)| ≤ L|x − y|, (∀)x,y ∈ [a,b] şif(a) = f(b) = 0. Atunci:(i) |f(x)| ≤ L (x−a)(b−x), (∀)x ∈ [a,b];2(ii) |f ′ (x)| ≤ L (b−a), (∀)x ∈ [a,b].2Demonstraţie. (i) Din |f ′ (x) − f ′ (y)| ≤ L|x − y|, (∀)x,y ∈ [a,b], deducem căfuncţiile f 1 ,f 2 : [a,b] → R definite prin f 1 (x) = Lx − f ′ (x) şi f 2 (x) = Lx + f ′ (x),(∀)x ∈ [a,b] sunt crescătoare, de unde, conform cu Propoziţia 1, (i) şi (iv), avemcă funcţiile F 1 ,F 2 : [a,b] → R definite prin F 1 (x) = L (x − a)(x − b) − f(x) şi2F 2 (x) = L 2 (x − a)(x − b) + f(x) sunt convexe. Cum F i(a) = F i (b) = 0, i ∈ {1,2},117


din Propoziţia 1, (ii) deducem că F 1 (x) ≤ 0, F 2 (x) ≤ 0, (∀)x ∈ [a,b], de unde− L 2 (x−a)(b−x) ≤ f(x) ≤ L 2 (x−a)(b−x), (∀)x ∈ [a,b], deci |f(x)| ≤ L 2 (x−a)(b−x),(∀)x ∈ [a,b].(ii) Dacă x = a sau x = b, atunci din punctul (i) deducem că |f ′ (x)| ≤ L 2 (b−a).Dacă x ∈ (a,b) definim funcţia h : [a,x] → R prin h(t) = f(t)− f(x)(t−a) , (∀)t ∈x−a[a,x]. În mod evident, h(a) = h(x) = 0, h este derivabilă şi |h ′ (t 1 ) − h ′ (t 2 )| =¬f ′ (t 1 )− f(x)x−a −f′ (t 2 )+x−a¬=|f f(x) ′ (t 1 ) − f ′ (t 2 )| ≤ L|t 1 − t 2 | şi, în consecinţă,h verifică ipotezele propoziţiei. Conform cu punctul (i), |h(t)| ≤ L (t − a)(x − t),2(∀)t ∈ [a,x], de unde¬h(t)−h(x)t−x¬≤ L 2 (t−a), ceea ce conduce la |h′ (x)| ≤ L 2 (x−a).Cum h ′ (x) = f ′ (x)− f(x)x−a , deducem că (∀)x ∈ [a,b] avem |f′ (x)| ≤ |h ′ (x)|+ |f(x)|x−a ≤L2 (x−a)+ L 2(x−a)(b−x)x−a= L 2 (b−a), adică |f′ (x)| ≤ L (b−a), (∀)x ∈ [a,b].2Observaţii. 1) Dacă există x 0 ∈ (a,b) astfel încât în Propoziţia 2 relaţia (i)are loc cu egalitate, atunci conform cu Propoziţia 1, (iii) una dintre funcţiile F 1 şiF 2 este nulă pe [a,b] şi, în consecinţă, f(x) = L (x − a)(b − x), (∀)x ∈ [a,b] sau2f(x) = − L (x−a)(b−x), (∀)x ∈ [a,b].22)Dacăexistăx 0 ∈ (a,b)astfel încât înPropoziţia2relaţia(ii) areloccuegalitate,atunci analizând demonstraţia găsim că în acelaşi x 0 are loc egalitate şi la (i) şiconform cu cele din observaţia precedentă găsim forma funcţiei f. Dacă |f ′ (a)| =L2 (b − a) sau |f′ (b)| = L 2 (b − a), ajungem imediat la concluzia că F 1 sau F 2 dindemonstraţia punctului (i) sunt funcţiile căutate.Observaţie. În fiecare din problemele P1, P2 funcţia g : [0,1] → R definită pring(x) =xf(t)dt, (∀)x ∈ [0,1], verifică condiţiile din Propoziţia 2 cu a = 0 şi b = 10şi, în concluzie, P1 şi P2 sunt consecinţe ale acesteia. În acelaşi mod se observă căfuncţia g : [a,b] → R definită prin g(x) =xf(t)dt este funcţia din P3 şi, cumaceastă funcţie verifică condiţiile din Propoziţia 2, deducem că P3 este o consecinţăa acesteia.Bibliografie1. W.W. Breckner, T. Trif - Convex Functions and Related Functional Equations,Cluj, 2008.2. C.P. Niculescu, L.-E. Persson-Convex Functions. Basic Theory and Applications,Editura Universitaria Craiova, 2003.a118


Aplicaţii ale inegalităţii mediilorîn rezolvarea unor probleme de minimLucian TUŢESCU, Mihai DICU 1Abstract. In this note some inequalities are presented; they are obtained by the inequalityof the arithmetical mean and geometrical mean. Moreover, every of the inequalities from 2) to 9)represents a generalisation of the previous inequality.Keywords: AM-GM inequality, extremum problems.MSC 2000: 52A40.În propoziţia ce urmează, ultima inegalitate generalizează pe toate cele care opreced. Am prezentat gradat acest tip de inegalităţi din motive didactice lesne deînţeles.Propoziţie. Au loc următoarele inegalităţi:(1) 2x n + n ≥ n+2, (∀)x ∈ (0,∞),(∀)n ∈ N;x2 (2) (x+y) n +2 n−1 n xy ≥ 2n−1 (n+2), (∀)x,y ∈ (0,∞), (∀)n ∈ N;(3)(x i +y i )+2ni=1ni=1n−1 1≥ 2 n−1 (n+2), (∀)x,y ∈ (0,∞), (∀)n ∈ N;x i y i(4) 3x n + n ≥ n+3, (∀)x ∈ (0,∞), (∀)n ∈ N;x3 (5) (x+y +z) n +3 n−1 nxyz ≥ 3n−1 (n+3), (∀)x,y,z ∈ (0,∞), (∀)n ∈ N;(x i +y i +z i )+3ni=1ni=1n−1 1(6)≥ 3 n−1 (n+3),x i y i z i(∀)x i ,y i ,z i ∈ (0,∞),(∀)n ∈ N;(7) kx n + n ≥ n+k, (∀)x ∈ (0,∞),(∀)k, n ∈ N;xk (8) (x 1 +x 2 +···+x k ) n + nkn−1≥ k n−1 (n+k),x 1 x 2···x k(9)(∀)x 1 ,x 2 ,··· ,x k ∈ (0,∞), (∀)k,n ∈ N;(x i1 +x i2 +···+x ik )+kni=1ni=1n−1 1≥ k n−1 (n+k),x i1 x i2···x ik(∀)k,n ∈ N, (∀)x ij ∈ (0,∞), i = 1,n, j = 1,k.Demonstraţie. Pentru justificarea fiecăreia dintre aceste inegalităţi vom folosiinegalitatea mediilor, în diverse moduri, grupând corespunzător.1 Profesori, Colegiul Naţional ”Fraţii Buzeşti”, Craiova119


1) Scriem membrul întâi desfăşurat şi apoi utilizăm inegalitatea mediilor:2x n + n x 2 = xn +x n + 1 x 2 + 1 x 2 +···+ 1 x 2 ≥ (n+2) n+2Öx n x n 1 x 2 1x 2 ··· 1x 2 = n+2,cu egalitate pentru x = 1 sau n = 0.2) Împărţind cu 2n−1 , inegalitatea se scrie sub forma (x+y)n2 n−1 + n xy ≥ n + 2.n+2ÖApoi, (x+y)n2 n + (x+y)n2 n + 1xy + 1xy +···+ 1xy√ 1(n+2) xy¡2n(xy)3) Procedând în mod analog, vom avea≥ (n+2) n+2x+y22n1(xy) n ≥n= n+2. Egalitate avem pentru x = y = 1 sau n = 0.ni=1x i +y i2+ni=1x i +y i2+ni=11≥ (n+2) n+2ni=1x i +y i 1≥x i y i 22 ni=1x i y i≥ (n+2) n+2ni=1√ 1xi y i2= n+2,nÉni=1x i y icu egalitate pentru x 1 = x 2 = ··· = x n = y 1 = y 2 = ··· = y n = 1 sau n = 0.Observaţie. Pentru n = 3 se obţine Problema 26315 [1].4) şi 7) se dovedesc adaptând demonstraţia dată pentru 1), inegalităţile 5) şi 8)ca şi 2), iar 6) ca şi 3).9) Dăm în detaliu demonstraţia acestei inegalităţi, care cuprinde ca nişte cazuriparticulareinegalităţile1)-8),aplicândînaceeaşimanierăinegalitateamediilor. NotămP = i1 +x i2nÉ+···+x ik ) şi S =i=1(x nÈi=11. Să demonstrăm că P+k n−1 S ≥x i1 x i2···x ik(n+k)k n−1 PP Psau +S ≥ n+k. Avem +S =kn−1 kn−1 kÈi=1 k n + nÈi=11≥x i1 x i2···x ik(n+k) n+kPnk 1. Ţinând seama căk i=1 x i1 x i2···x ikPk n = x 11 +x 12 +···+x 1k x 21 +x 22 +···+x 2k x n1 +x n2 +···+x nk····≥kkk≥ k√ x 11 x 12···xk 1k · √ x 21 x 22···xk 2k ··· √ x n1 x n2···x nk ,Pobţinem că +S ≥ (n+k)n+knÉnÉ 1xkn−1 i1 x i2···x ik . = n+k. Egalitateavem pentru x ij = 1,i = 1,n,j =i=1 i=1 x i1 x i2···x ik1,k.Bibliografie1. D. Săvulescu, L. Tuţescu – Problema 26 315, G.M.-B, nr. 6/2010, pag. 324.120


Asupra unei inegalităţi ”aproape clasică”Dan Dănăilă 1Abstract. In this note we present two inequalities of Cauchy-Buniakovsky-Schwarz type andsome of their consequences.Keywords: Cauchy-Buniakovski-Schwarz, cardinal number, matrix.MSC 2000: 26D15.Vom prezenta două inegalităţi de tip Cauchy-Buniakovski-Schwarz şi câteva aplicaţiiale lor.Propoziţie. Fie numerele m,n ∈ N, a 1 ,a 2 ,...,a m ∈ R + , b 1 ,b 2 ,...,b n ∈ R + ,x 1 ,x 2nÈ,...,x n ∈ R, y 1 ,y 2≤mÈ,...,y m ∈ R şimulţimile finite A 1 ,A 2 ,...,A m , B 1 ,B 2 ,...,B n .Au loc inegalităţile:jnÈ(1)mÈi=1i ∩B j¬x i y j2i ∩A j |x i x i ∩B j¬y i y j.(2)mÈi=1 j=1¬Ai,j=1¬Ai,j=1¬BDemonstraţie. (1)Fiemulţimea{c 1 ,c 2 ,...,c r } = {a 1 ,a 2 ,...,a m ,b 1 ,b 2 ,...,b n },cu 0 ≤ c 1 < c 2 < ... < c r , l 1 = c 1 ,l 2 = c 2 − c 1 ,l 3 = c 3 − c 2 ,...,l r = c r − c r−1 şimatricea D = diag( √ l 1 , √ l 2 ,..., √ l r ). Fie vectorii d i = (d i1 ,d i2 ,...,d ir ) şi e i =≤ÈnÈj=1min(a i ,b j )x i y j21≤i,j≤mjÈmin(a i ,a j )x i x min(b i ,b j )y i y j;1≤i,j≤na i ≥ c j b i ≥ c j(e i1 ,e i2 ,...,e ir ), definiţi prin d ij =1,şi e ij =1,, şi vectorii0, a i < c j 0, b i < c jd ′ i = x id i D, e ′ i = y ie i D. Cu aceste notaţii, inegalitatea (1) se rescrie astfel:mÈi=1≤ÈnÈj=1d ′ i t e j2 ′ 1≤i,j≤mjÈd ′ i t d ′ e ′ i t e ′ j⇔1≤i,j≤nd ⇔mÈi=1′ inÈj=1d ≤mÈi=1′ imÈi=1t d ′ e inÈi=1′ inÈi=1t e ′ i.t e ′ j2Dacă notăm cu u = mÈi=1d ′ i şi v = nÈi=1e ′ i , inegalitatea devine:(u t v) 2 ≤ (u t u)(v t v) u i v i2⇔ri=1≤ri=1u 2 iri=1v 2 i.Observând că am ajuns la inegalitatea CBS aplicată numerelor u 1 ,u 2 ,...,u r şiv 1 ,v 2 ,...,v r , demonstraţia se încheie.(2) Fie mulţimeaC = A 1 ∪A 2 ∪...∪A m ∪B 1 ∪B 2 ∪...∪B n , cuC = {c 1 ,c 2 ,...,c r }.Considerăm vectorii d i = (d i1 ,d i2 ,...,d ir ) şi e i = (e i1 ,e i2 ,...,e ir ), unde d ik =1 Elev, cl. a XI-a, Colegiul Naţional ”V. Alecsandri”, Galaţi121


1, c k ∈ A i c k ∈ B işi e ik =1,şi d ′ i0, c k /∈ A i 0, c k /∈ B = x id i , e ′ i = y ie i . Folosind aceste notaţii,iinegalitatea se rescrie în formanÈ1≤i,j≤n11,j=1mi=1nj=1d ′ i t e ′ j2≤1≤i,j≤mjd ′ i t d ′ e ′ i t e ′ j,1≤i,j≤njmin(a i ,a j )x i xcu care ne-am întâlnit mai sus. Continuând în acelaşi fel, deducem că (2) esteadevărată.Consecinţa 1. Fie a 1 ,a 2 ,...,a n > 0 şi x i ,y i ∈ R,i = 1,n. Atuncimin(a i a j ,1)x i y j2≤1≤i,j≤n≤nÈ j¬≤È jnÈ1≤i,j≤nmin(a i ,a j )x i x1,j=11,j=1min(a i a j ,1)x i x1≤i,j≤n1≤i,j≤nmin(a i ,a j )y i y j.Demonstraţie. Considerăm m = n şi şirurile a 1 ,a 2 ,...,a n ∈ R ∗ +, b 1 = 1 , b 2 =a 1,...,b n = 1 , x 1 ,x 2 ,...,x n ∈ R, y 1 a 1 ,y 2 a 2 ,...,y n a n ∈ R. Aplicând inegalitateaa 2 a n2.¬È(1), obţinem:mina i , 1 i y j a j2min1, 1 i y j a i a j,a jxa i a jyde unde rezultă imediat inegalitatea dorită.Consecinţa min(a i ,a j )x i x j (a i > 0,x i ∈ R, i = 1,n).nÈi,j=1Consecinţa 3. Matricele (min(a i ,a j )) i,j=1,n±(min(a i a j ,1)) i,j=1,n) sunt pozitivsemidefinite.Demonstraţie. Se deduce din Consecinţa 2. Acesta este un rezultat importantşi nebanal!nÈConsecinţa 4.≤nÈmin(x i y j ,x j y i )(x i ,y i ∈ R + , i = 1,n).(USAMO 2000, Gheorghe Zbăganu).Demonstraţie. Fără a restrânge generalitatea putemjnÈconsidera x i ,y i ∈ R ∗ + ,i = 1,n. Aplicând inegalitatea (1) pentru m = n, a i = y i, b i = x i y i , x i ,y i ∈ R ∗ +x i(i = 1,n), obţinemiminy, x jii y j2miny, y jii x minx, x ji y j,i,j=1 x i y jxi,j=1 x i x jxi,j=1 y i y jyde unde inegalitatea de demonstrat. Astfel am obţinut o rezolvare elegantă şi rapidăa cunoscutei şi dificilei inegalităţi propusă de Gheorghe Zbăganu la USAMO 2000.nÈ min(x i x j ,y i y j ) ≤i,j=1122


nÈ ≤nÈConsecinţă min(a i ,b j )25.nÈi=1 j=1i,j=1a 1 ,a 2 ,...,a m ∈ R ∗ + , b 1,b 2 ,...,b n ∈ R ∗ + (Don Zagier). Când are loc egalitatea?Demonstraţie. Dacă considerăm m = n, a i ,b i ∈ R ∗ +, x i = y i = 1(i = 1,n) şiaplicăm inegalitatea (1), obţinem inegalitatea lui Don Zagier.Partea cea mai interesantă a inegalităţii este cazul de egalitate, care se stabileşteuşor apelând la demonstraţia dată pentru inegalitatea (1). Anume, folosind condiţiile)nÈmin(a i ,a j min(b i ,b j ),i,j=1în care avem egalitate în inegalitatea CBS, deducem că egalitatea are loc când u 1=v 1u 2= ... = u r; dar u 1 = v 1 = n √ l 1 , ceea ce implică că u i = v i ,i = 1,n. Rezultăv 2 v rcă în fiecare din şirurile (a i ) i=1,n şi (b i ) i=1,n, c k apare de acelaşi număr de ori (c k deu k+1− Ôl u k√ , k = 1,r−1 ori, iar c r de u r√ ori). Deci egalitatea are loc când şirulk+1 lk lr(b i ) i=1,neste o permutare a şirului (a i ) i=1,n.≤mÈi,j=1|A i ∩B j |2Consecinţa6.mÈi=1nÈj=1n,m ∈ N şi A 1 ,A 2 ,...,A m ,B 1 ,B 2 ,...,B n sunt mulţimi finite.|nÈ|A i ∩A ji,j=1|B i ∩B j |, undeDemonstraţie. Se aplică inegalitatea (2) pentru mulţimile A 1 ,A 2 ,...,A m ,B 1 ,B 2 ,...,B n şi şirurile x 1 = x 2 = ... = x n = 1,y 1 = y 2 = ... = y m = 1 şi seobţine inegalitatea de mai sus.Observaţie. Toate aceste inegalităţi se bazează pe aplicarea inegalităţii CBSîntr-un mod ingenios şi pe observarea faptului că sumele din dreapta sunt pozitive înciuda faptului că x 1 ,x 2 ,...,x n ∈ R şi y 1 ,y 2 ,...,y m ∈ R.Premiu pe anul 2011 acordatde ASOCIAŢIA ”RECREAŢII <strong>MATEMATICE</strong>”Se acordă un premiu în bani în valoare de 200 lei elevuluiDĂNĂILĂ Dan – Colegiul Naţional ”V. Alecsandri”, Galaţipentru nota Asupra unei inegalităţi ”aproape clasică” apărută în acest număr alrevistei Recreaţii Matematice.123


Autour du cardinal d ′ un ensemblede matrices binairesAdrien REISNER 1Abstract. We here study a couple of algebraic and analytic properties of certain binary matricesin the spaces M n(R). In particular, we study power series whose coefficients are functions of thecardinal of these matrices, as well the rank of this family of matrices.Keywords: binary bistochastic matrices, rank, power series.MSC 2000: 15A03, 15A18.On considère l ′ ensemble U n des matrices binaires de taille n comportant exactementdeux 1 dans chaque ligne et exactement deux 1 dans chaque colonne (pour toutematrice A ∈ U n la matrice 1 2 A est bistochastique). On désigne par u n = CardU n eton pose u 0 = 1, u 1 = 0. J n est la matrice de M n (R) dont tous les coefficients sontégaux à 1.1Exemples. Pour n = 2, la seule matrice de U 2 est la matrice J 2 =11 1.Donc u 2 =1. Pour n=3,U 3 est formé des 6 matrices A j ,j : 1,...,6 (suivant la position1 1du seul élément nul (a 1i ) i:1,2,3 de la première ligne): (a 11 = 0) : A 1 =01 1 01 0 1,1 10 1 0 1A 2 =01 0 1 (a 12 = 0) : A 3 =11 1 0 A 4 =10 1 1 (a 13 = 0) :1 1 0;0 1 1,1 1 0;1 0 1 0A 5 =11 0 1 A 6 =10 1 1 Donc u 3 = 6.0 1 1,1 0 1.1. Étude de u n . On désigne par H n le sous-ensemble de U n comportant un 1 enposition (1,1) et h n = CardH n . En outre, K n est le sous-ensemble de H n comportantun 1 en position (1,2) et un 1 en position (2,1) et k n = CardK n . X 0 = [1] désignantle vecteur de R n dont tous les coefficients sont égaux à 1 on a leThéorème 1. a) Pour toute matrice A ∈ U n , 2 est valeur propre de A et X 0 estun vecteur propre associé à cette valeur propre. b) On a:ÈA = h n J n .A∈U nDémonstration. a) Pourtoute matrice A ∈ U n il est immédiat que le vecteur X 0est vecteur propre associéà la valeur propre λ = 2 (la matrice 1 A est bistochastique):2AX 0 = 2X 0 , ∀A ∈ U n .1 TELECOM ParisTech; e-mail: Adrien.Reisner@telecom-paristech.fr124


) Pour (i,j) fixé soit (U n ) ij l ′ ensemble (U n ) ij = {A ∈ U n ;a ij = 1} pour toutematrice A de U n désignons par A ′ la matrice obtenue à partir la matrice A enéchangeant les lignes 1 i puis les colonnes 1 j. Il est immédiat que cette matriceA ′ appartient à l ′ ensemble H n que l ′ application (U n ) ij → H n , A ↦→ A ′ est bijective.Par suite pour tout i,j : 1,...,n : Card(U n ) ij = CardH n = h n , d ′ où l ′ b).Corollaire. On a u n = n 2 h n.Démonstration. Comptetenudesdeuxassertionsprécédentesilvient:ÈAX 0A∈U n= h n J n X 0 soit 2u n X 0 = nh n X 0 et le corollaireest ainsi démontré,le vecteurX 0 étantvecteur propre de la matrice J n pour la valeur propre n.Théorème 2. On a: a) h n = (n−1) 2 k n ,n ≥ 2; b) k n = u n−2 +h n−1 pour n ≥ 4.Démonstration. a) Pour tout (i,j),2 ≤ i, j ≤ n, soit Hn ij l′ ensembles deséléments de H n nÈayant un 1 dans la position (i,1) et un 1 en position (1,j). Cesensembles Hn ij,2 ≤ i, j ≤ n, constituent une partition de H n et il y a (n − 1) 2 detels ensembles. Remarquons que Hn 22 = K n. De plus, l ′ application ϕ i,j nÈ: Hn ij → K n,A ↦→ A ij , où la matrice A ij est obtenue à partirde la matrice A par échangedes lignes2 et i et des colonnes 2 et j (lorsque i = 2 ou j = 2 on ne fait pas d ′ échange) est defaçon évidente bijective, soit pour tout (i,j), 2 ≤ i, j ≤ n, CardHn ij = CardK n = k n .On en déduit alors pour n ≥ 2 : h n = CardH n = CardHn ij = CardK n =i,j=2 i,j=2(n−1) 2 k n .b) Pour n ≥ 4, K n est réunion disjointe des deux parties suivantes: 1) K n1ensemble des éléments de K n ayant un 1 dans la position (2,2). L ′ application de K n1dans U n−2 qui à chaque matrice A fait associer la matrice obtenue à partir de A ensupprimantlesdeux premièresligneset lesdeux premièrescolonnesest manifestementbijective; donc: CardK n1 = CardU n−2 = u n−2 . 2) K n2 ensemble des éléments deK n ayant un 0 dans la position (2,2). Dans ce cas on considère l ′ application deK n2 dans H n−1 qui à chaque matrice A fait associer la matrice obtenue de celle-cien remplaçant le 0 de la position (2,2) par 1 puis en supprimant la première ligneet la première colonne. On définit ainsi une application bijective. Il vient alors:CardK n2 = CardH n−1 = h n−1 . Finalement, pour n ≥ 4 on a: k n = CardK n =CardK n1 +CardK n2 = u n−2 +h n−1 .On pose pour tout n ∈ N: w n = u n(n!) 2.Théorème 3. a) w n vérifie la relation de récurrence w n = 12n w n−2+ n−1n w n−1,n ≥ 2, avec w 0 = 1 et w 1 = 0.b) w n ∈ [0,1] pour tout n ∈ N.c) La série de terme général w n diverge.d) La série ∞Èn=0w n x n converge pour x ∈]−1,1[.Démonstration. a) Pour n ≥ 4 il vient, compte tenu du théorème précédent et125


du corollaire:u n = n 2 h n = n 2 (n−1)2 (u n−2 +h n−1 ) = n 2 (n−1)2 u n−2 +n(n−1)u n−1 .A l ′ aide des conventions u 0 = 1 et u 1 = 0 cette relation est encore vérifiée pour n = 2et n = 3. On a alors pour n ≥ 2: w n = u n(n!) 2 = 12n w n−2 + n−1n w n−1.b) Pour tout n,u n ≥ 0, donc w n ≥ 0. Montrons par récurrence que w n ≤ 1,n ∈ N. D ′ abord, w 0 = 1 ≤ 1 et w 1 = 0 ≤ 1. Soit n ≥ 2 et supposons que w n−2 ≤ 1 etw n−1 ≤ 1; alors,comptetenudel ′ assertionprécédente: w n = 12n w n−2+ n−1n w n−1 ≤12n + n−1 = 2n−1 ≤ 1.n 2nc) Il est clair que w n > 0, ∀n ≥ 2. Compte tenu de l ′ assertion a), il vient: w n ≥n−1n w n−1 et par suite: w n ≥ 2 n w 2 = 12n u 2 = 1 , pour n ≥ 3. Ceci montre que la2nsérieÈw n diverge.d) 0 ≤ w n ≤ 1 (voir b)) implique |w n x n | ≤ |x n |. La série géométrique x n étantconvergente pour −1 < x < 1, on déduit que ∞Èn=0w n x n est absolument convergente,donc convergente pour |x| < 1.Théorème 4. Pour x ∈]−1,1[ on a: W(x) = ∞Èn=0w n x n = e−1 2 x√ . 1−xDémonstration. Soit x ∈]−1,1[. D ′ après l ′ assertion a) du Théorème 3,2nw n = 2(n−1)w n−1 +w n−2 , n ≥ 2.En multipliant les deux membres de cette égalité par x n−1 , puis en sommant, onobtient alors:2nw n x∞n=2n−1 = 2x(n−1)w n−1 x∞n=2n−2 +xw n−2 x∞n=2n−2 .Il vient: 2(W ′ (x)−w 1 ) = 2xW ′ (x)+xW(x) soit puisque w 1 = u 1 = 0:W ′ (x) =D ′ ici et du fait que W(0) = 1, on obtient alorsxW(x), ∀x ∈]−1,1[.2(1−x)W(x) = e−1 2 x√ 1−x, ∀x ∈]−1,1[.Remarque. Une étude plus approfondie permet de trouver, en utilisant la fonctionΓ ainsi que la formule de Stirling, un équivalent de u n lorsque n → ∞, à savoir:u n ∼ 2ne2n+ 1 2 √ π, mais cette étude dépasse le niveau de cet article.126


2. Étude de rang. On se propose dans cette partie de déterminer le rang r n dusystème constitué des u n , matrices de U n considérées comme éléments de M n (R).Proposition. Avec ces notations on a : r 2 = 1 et r 3 = 5.Démonstration. Pourn = 2, U 2 contenantla seule matrice non nulle J 2 : r 2 = 1.Pour n = 3, U 3 contient les 6 matrices A i , i : 1,...,6, trouvées page 124. Désignonspar J le sous - espace vectoriel de M n (R) engendré par ces 6 matrices. D ′ aprèsl ′ assertion b) du Théorème 1, la matrice J 3 appartient J et par suite J est aussiengendré par les 6 matrices J − A i , i : 1,...,6. Ces 6 matrices sont dans l ′ ordre:0 0 0 0 1 0J − A 1 =10 0 1 J − A 2 =10 1 0 J − A 3 =00 0 1 J − A 4 =0 1 0,0 0 1,1 0 0,0 1 0 0 1 0 11 0 0 J −A 5 =00 1 0 J −A 6 =01 0 0 Soient λ i , i : 1,...,6,0 0 1,1 0 0,0 1 0.1 +λ 2 λ 3 +λ 4 λ 5 +λ 6des réels donnés. Alors6Èi=1λ i (J − A i ) = 0 ⇔λλ 4 +λ 6 λ 2 +λ 5 λ 1 +λ 3λ 3 +λ 5 λ 1 +λ 6 λ 2 +λ 4=0 0 00 0 0 1 = −λ 2 = −λ 3 = λ 4 = λ 5 = −λ 6 . On en déduit r 3 = 5.0 0 0⇔λSoit V n l ′ espace vectoriel des matrices A ∈ M n (R) telles que X 0 = [1] soit àla fois vecteur propre de A et de sa transposée t A. [V n est effectivement un sousespacevectoriel de M n (R) car 0 ∈ V n et si (α,β) ∈ R 2 et (A,B) ∈ (V n ) 2 alors:(αA+βB)X 0 ∈ Vect(X 0 ) et (α t A+β t B)X 0 ∈ Vect(X 0 ).]Théorème 5. U n ⊂ V n . Pour A ∈ V n si AX 0 = λX 0 , alors t AX 0 = λX 0 .Démonstration. Si A ∈ U n , sa transposée t A appartient aussi à U n nÈ. Comme X 0est vecteur propre de toute matrice de U n on a: U n ⊂ V n .Soit alors A = (a ij ) ∈ V n , λ la valeur propre de A associée au vecteur propre X 0 ,i.e. AX 0 = λX 0 et µ celle de la transposée t A associée à X 0 : t AX 0 = µX 0 . Pourtout i,j : 1,...,n, on a: a ik = λ et a kj = µ. Par suite: nλ =nÈk=1nÈk=1nÈi=1( a ij ) =j=1nÈj=1( nÈi=1a ij ) = nµ soit λ = µ.Théorème 6. a) dim V n = (n−1) 2 +1; b) r n ≤ (n−1) 2 +1.Démonstration. a) On munit M n,1 (R) de sa structure euclidienne canoniqueet on note B 0 = {e 1 ,...,e n } la base canonique de M n,1 (R). Soit e ′ 1 = 1 √ nX 0 . Oncomplète la famille orthonormée (e ′ 1 ) en une base orthonormée B 1 = {e ′ 1 ,...,e′ n}de M n,1 (R). On note P la matrice de passage de B 0 à B 1 et pour A ∈ M n (R)soit f l ′ endomorphisme de M n,1 (R) canoniquement associé à A. Compte tenu du127


théorème 5:A ∈ V n ⇔ ∃λ ∈ P/Ae ′ 1 = t Ae ′ 1 = λe ′ 1 ⇔⇔ ∃λ ∈ P,∃A ′ ∈ M n−1 (R)/Mat(f,B 1 ) =λ 00 A ′⇔⇔ ∃λ ∈ P, ∃A ′ ∈ M n−1 (R)/A = Pλ 00 A ′P −1 .Il vient alors, l ′ application M ↦→ PMP −1 étant un automorphisme de M n (R),dimV n = dim{λ 00 A ′;λ ∈ R,A ′ ∈ M n−1 (R)} = 1+(n−1) 2 .b) On en déduit puisque U n ⊂ V n : r n = dim(Vect(U n )) ≤ dimV n = (n−1) 2 +1.Pour n ≥ 3 soit A = (a ij ) une matrice de U n comportant des 1 en positions (1,1)et (2,2) et des 0 en positions (1,2) et (2,1). La matrice B = (b ij ) définie par b ij = a ijsi i > 2 ou j > 2, b ij = 1 − a ij si i ≤ 2 et j ≤ 2 est une matrice binaire ayant lesmêmes lignes et les mêmes colonnes que A à partir de la troisième ligne ou colonne.La définition des éléments (1,1),(2,2),(1,2) et (2,1) de cette matrice B montre alorsque B ∈ U n . Pour tout i,j : 1,...,n : a ij −b ij = 0 si i > 2 ou j > 2; a ij −b ij = 1 sii = j, i ≤ 2 et j ≤ 2; a ij −b ij = −1 si i ≠ j, i ≤ 2 et j ≤ 2. La matrice A−B necomporte donc que des éléments nuls sauf en positions (i,j) avec i ≤ 2 et j ≤ 2.Soit r ′ n le rang du système constitué de toutes les matrices U −U ′ où U,U ′ ∈ U n .Théorème 7. a) r ′ n ≥ (n−1)2 ; b) r n = n 2 −2n+2.Démonstration. a) Pour tout i,j : 1,...,n notons A ij la matrice obtenue àpartir de la matrice A par l ′ échange des lignes i et 2 et les colonnes j et 2 et par B ijla matrice obtenue à partir de la matrice B par les mêmes échanges. Ces matrices A ijet B ij appartiennent à U n . De plus, A ij −B ij est une matrice à coefficients nuls saufen positions (1,1), (i,j), (1,j) et (i,1) et telle que l ′ élément de la position (i,j) est1. On remarque que par suppression de la première ligne et de la première colonnede A ij − B ij on obtient la matrice E i−1,j−1 de la base canonique de M n−1 (R). Lafamille {A ij −B ij } i,j:2,...,n est donc libre, et par suite: r ′ n ≥ (n−1)2 .b) D ′ après l ′ assertion b) du Théorème 6 et de l ′ assertion a) précédente on a lesinégalitéssuivantes: (n−1) 2 ≤ r ′ n ≤ r n ≤ (n−1) 2 +1.Montronsquel ′ inégalitér ′ n ≤ r nest stricte. En effet, pour tout U,U ′ ∈ U n on a (U −U ′ )X 0 = 0. Or J n X 0 = nX 0 etpar suite la matrice J n ne peut être combinaison linéaire des éléments U −U ′ avecU,U ′ ∈ U n , mais elle est combinaison linéaire des éléments de U n (voir Théorème 1b)). Donc on a l ′ inégalité stricte: r ′ n < r n.On en déduit finalement l ′ égalité: r n = (n−1) 2 +1.128


Comentarii pe marginea unei problemeNeculai ROMAN 1Abstract. The author shows how a simple problem of elementary geometry may be a source ofnew problems and results.Keywords: equilateral triangle, orthocenter, incenter.MSC 2000: 51M04.La Concursul ”Al. Myller”, ed. a VIII-a, 2010, elevii din cl. a VII-a au fost puşiîn faţa problemei (v. Recreaţii Matematice, 2/2010, p.134):(P): Se consideră triunghiul ABC în care AB = AC şi fie M mijlocul segmentului[BC]. Punctele D şi E sunt picioarele perpendicularelor din M pe dreptele AC,respectiv AB, iar H este mijlocul segmentului [DE]. Fie punctele P,Q ∈ BC astfelîncât MQ = MP = MD şi P ∈ (BM). Arătaţi că punctul H este ortocentrultriunghiului APQ.Constatămcăproblema(P) aretreielementeconstitutive: 1)un triunghiABC, 2)un punct M ∈ BC şi 3)un triunghiAPQ(construitastfel: D,E fiind proiecţiile luiMpe AC, respectiv AB, atunci punctele P,Q ∈ BC cu P ∈ (BM) sunt determinateprinMP = ME şi MQ = MD), iar rezultatul problemei (P) are structura: în anumiteipotezerelativlatriunghiulABC şipunctulM, urmeazăoanumităproprietatepentrutriunghiul APQ.Astfelprivită,(P)devineosursădenoiproblemeşirezultate. Într-adevăr,punctulM, care în (P) este mijlocul laturii [BC], poate fi luat picior de înălţime, bisectoareetc. Relativ la triunghiul APQ putem avea în vedere proprietăţi ca: este isoscel, areotrocentrul sau alt punct important pe DE, AM ⊥ DE etc. În această configuraţieexistă suficient loc pentru curiozitatea şi imaginaţia oricui, iar pentru descoperireaproprietăţilor ei este nevoie de curaj, muncă, răbdare.Pregătim rezolvarea problemei (P) cu următoareaLemă. Fie ABC un triunghi oarecare şi M piciorul înălţimii din A pe BC.Atunci, un punct H ∈ AM este ortocentrul triunghiului ABC dacă şi numai dacă<strong>MB</strong> ·MC = MH ·MA.Demonstraţie. Dacă H este ortocentru, avem △H<strong>MB</strong> ∼ △CMA şi, deci,HMMC = <strong>MB</strong> , de unde rezultă relaţia dorită.MAAFie, acum, B ′ ∈ BH ∩AC. Din AM ⊥ BC şi <strong>MB</strong> ·MC =MH ·MA rezultă că △H<strong>MB</strong> ∼ △CMA, deci unghiurile marcateH Bsunt congruente şi BB ′ ⊥ AC, i.e. H esteortocentru.1 Profesor, Şcoala ”V. Alecsandri”, Mirceşti (Iaşi)129BMC


Demonstraţia afirmaţiei problemei (P). Din simetria figurii rezultă că DE‖BCşi H ∈ AM. Cu teorema catetei obţinem MD 2 = MA·MH, de unde MP ·MQ =MA·MH. Conform Lemei, deducem că H este ortocentrul triunghiului APQ.Observaţie. Rezolvarea dată problemei (P) diferă de cea oficială (din barem).În continuare, vom prezenta câteva reciproce şi o generalizare a problemei (P).AReciproca 1. Fie M piciorul înălţimii din vârful A al triunghiuluiABC, D şi E proiecţiile lui M pe AC, respectiv ABşi punctele P,Q ∈ BC astfel încât MP = ME, MQ = MDşi P ∈ (BM). Dacă △APQ este isoscel de vârf A, atunci şi△ABC este isoscel.Demonstraţie.B P M Q CÎn condiţiile enunţului, AM ⊥ BC implică faptul că [MP] ≡[MQ]. Deci [ME] ≡ [MD] şi [AM] este bisectoarea unghiuluiBAC. Cum aceastaeste şi înălţime în △ABC, rezultă că triunghiul este isoscel.Reciproca 2. Considerăm un punct M ∈ (BC) şi fie punctele D,E,P,Q construiteca în Reciproca 1. Dacă △APQ este isoscel de vârf A şi AM ⊥ DE, atuncişi △ABC este isoscel.Demonstraţie. În cercul circumscris patrulaterului AEMD diametrul [AM] şicoarda [DE] sunt perpendiculare, deci [ME] ≡ [MD]. Ca urmare, [MQ] ≡ [MP],deci AM ⊥ BC. Conform Reciprocei 1, rezultă că △ABC este isoscel.Reciproca 3. Considerăm un punct M ∈ (BC) şi fie punctele D,E,P,Q construiteca în Reciproca 1. Dacă punctul H = AM ∩DE este ortocentrul triunghiuluiAPQ, atunci △ABC este isoscel.Demonstraţie. Evident, [AM] este înălţime în △ABC. Ca urmare, DE esteantiparalelă cu BC (fapt cunoscut şi uşor de dovedit!). Fie P ′ = PH ∩ AQ. Întriunghiul dreptunghic PP ′ C avem: tgP ′ QP ·tgP ′ PQ = 1. DartgP ′ QP = AMMQ = AMMD = 1cosC şitgP ′ PQ = HMMP = HMME = sinMEHsinMHE =Ca urmare,cosCsin(90 ◦ −B +C) =EHPcosCcos(B −C) .1cosC · cosC= 1, de unde cos(B −C) = 1 şi, deci, B = C.cos(B −C)În rezultatele precedente punctul M ∈ (BC) apărea (de la bun început sau peparcurs!) ca fiind piciorul înălţimii din A. La începutul notei au fost sugerate câtevadirecţii de investigare a configuraţiei în discuţie. Cititorul atras şi dornic de aventuriîn câmpul geometriei este îndemnat să-şi croiască propriul traseu. Noi vom încheiacu un rezultat în care punctului M i se atribuie o altă poziţie pe latura [BC].Fie ABC un triunghi oarecare şi M ∈ BC piciorul bisectoarei unghiuluiA. Seconsideră punctele D,E,P,Q ca în reciprocele de mai sus şi punctul H ca intersecţie130D


a dreptei DE cu perpendiculara din A pe BC. Atunci H este ortocentrul triunghiuluiAPQ.Demonstraţie. Presupunem că unghiurileB şiC sunt ascuţite (alte cazuri setratează similar sau sunt triviale). Fie A 1 proiecţia punctului A pe BC şi N mijloculsegmentului [DE].Din faptul că [AM] este bisectoare, deducem că △AED esteAisoscel. Cum [AN] este mediană în △AED isoscel, rezultă că[AN] este bisectoare şi, deci, N ∈ AM.Avem cu teorema catetei,E N HD(1) MP 2 = MD 2 = MA·MN;de asemenea, scriind în două moduri puterea punctului A 1 faţă de cercul circumscrispatrulaterului PEDQ, are loc egalitatea(2) A 1 P ·A 1 Q = MP 2 −MA 2 1.Din (1) şi (2) rezultă că(3) A 1 P ·A 1 Q = MA·MN −MA 2 1.Dar AN · AM = AA 1 · AH (puterea lui A faţă de cercul circumscris patrulateruluiA 1 HNM) sauAM(AM −MN) = AA 1 (AA 1 −A 1 H) ⇔ AM 2 −MA·MN = AA 2 1 −A 1 A·A 1 H ⇔(4)B P⇔ AA 2 1 +MA 2 1 −MA·MN = AA 2 1 −A 1 A·A 1 H ⇔⇔ MA 2 1 = MA·MN −A 1A·A 1 H.Din (3) şi (4) rezultă că A 1 P · A 1 Q = A 1 A · A 1 H, deci, conform Lemei, H esteortocentrul triunghiului APQ.Observaţie. Acest rezultat reprezintă o generalizare a problemei (P).MA 1Q CVizitaţi pagina web a revistei Recreaţii Matematice:http://www.recreatiimatematice.ro131


Proprietăţi ale triunghiurilor în care 3a = b + cIon PĂTRAŞCU 1Abstract. The aim of this paper is to point out some properties of the triangles for which therelation 3a = b+c holds.Keywords: incenter, centroid, Nagel point.MSC 2000: 51M04.Vom pune în evidenţă câteva proprietăţi ale triunghiurilor în care suma a douălaturiestetriplulceleide-atreia. Vomvedeacăaceastăparticularitateimpuneanumitorpuncte importante ale triunghiului (cum ar fi G−centrul de greutate, N−punctullui Nagel) să aibă poziţii speciale faţă de elementele triunghiului.Câteva proprietăţi imediate sunt date dePropoziţia 1. Într-un triunghi ABC următoarele afirmaţii sunt echivalente:1) 3a = b + c, 2) p = 2a, 3) S = 2ar, 4) r = h a4 , 5) S = ar a, 6) r a = 2r,7) 3sinA = sinB +sinC, 8) tg B 2 tg C 2 = 1 2 .Demonstraţie. Vom utiliza formule cunoscute, ca: S = ah a2 = pr = (p−a)r a,tg A =(p−b)(p−c) etc.2 p(p−a)Evident, 1)⇔2)⇔3). Pentru 3)⇔4) scriem: S = 2ar ⇔ ah a= 2ar ⇔ h a2 4 = r.Din S = (p−a)r a , deducem uşor că 2)⇔5). Pentru a dovedi că 2)⇔6), observăm maiîntâi că pr = (p−a)r a (= S); atunci 2)⇔ p = 2(p−a) ⇔ pr = 2(p−a)r ⇔ (p−a)r a =2(p−a)r ⇔ r a = 2r ⇔6). Evident, 1)⇔7) (teorema sinusurilor). În sfârşit, 2)⇔8),căci tg B 2 tg C 2 = p−ap2)⇔ p−apîn orice triunghi (cu formulele pentru tg A 2 şi tg B ) şi atunci2= 1 2 ⇔ tg B 2 tg C 2 = 1 2 ⇔b).Propoziţia 2. Fie ABC un triunghi oarecare şi E,F punctele de tangenţă acercului înscris cu laturile AC, respectiv AB. Atunci, relaţia 3a = b+c are loc dacăşi numai dacă G ∈ (EF).Demonstraţie. Fie 3a = b + c. Notăm cu T intersecţia dreptelor EF şi BC(figura corespunde cu cazul b > c). Menţionăm faptul că EF‖BC ⇔ AB = AC şi căexcludem cazul triunghiului isoscel ca banal. Fie M mijlocul segmentului [BC] şi G ′intersecţia dreptelor AM şi EF.1 Profesor, Colegiul Naţional ”Fraţii Buzeşti”, Craiova132


Se ştie că avem relaţiile AE = AF =Ap − a, care, în ipoteza noastră, se scriuAE = AF = a. Cu teorema lui Menelausaplicată △ABC şi transversalei T −F −ETBFG Eobţinem:.TC = p−b . Ca urmare,p−c ITB= p−bT B M Ca b−c sau TB a= 2(p−b) sau,2b−cTBîncă,TM = 2(p−b) . Pe de altă parte, aplicând teorema lui Menelaus la △ABMaşi transversala T −F −G ′ , obţinem G′ AG ′ M = TBTM · ap−b , de unde G′ AG ′ = 2, adicăMG ′ ≡ G.Reciproc, dacă G ∈ (EF), obţinem ca mai sus TBTM = 2(p−b) . Cum GA =a2GM, teorema lui Menelaus aplicată la △ABM şi transversala T − F − G ne dăTBTM = 2(p−b)p−a . Egalând valorile obţinute pentru raportul TB, avem a = p − a,TMadică 3a = b+c. Propoziţia este complet demonstrată.Notăm cu D a ,E b ,F c punctele de tangenţă a cercurilor A-exînscris, B-exînscris şiC-exînscris cu laturile BC,CA şi, respectiv, AB. Se ştie că BF c = CE b = p − a,CD a = AF c = p−b şi AE b = BD a = p−c. Cevienele AD a , BE b , CF c se numescceviene Nagel şi sunt concurente într-un punct N−punctul lui Nagel (concurenţa sedovedeşte cu teorema lui Ceva, pe baza egalităţilor precedente).Propoziţia 3. Un triunghi are suma a două laturi egală cu triplul celei de-a treiadacă şi numai dacă punctul N aparţine cercului său înscris.Demonstraţie. Este cunoscută formulaIN 2 = p 2 −16Rr+5r 2 .Faptul că N aparţine cercului înscris se exprimă prin IN = r. Ţinând seama deformulele R = abc4S , r = S p şi S2 = p(p−a)(p−b)(p−c) şi forţând apariţia factorilorp−2a, p−2b, avem (cu calcule de rutină!):IN = r ⇔ p 2 −16Rr+5r 2 = r 2 ⇔ p 3 −4abc+4(p−a)(p−b)(p−c) = 0⇔ [(p−2a)+2a] 3 −4abc+4[(p−2a)+a](p−b)(p−c) = 0⇔ (p−2a)[p 2 −2ap+4(p−b)(p−c)] = 0⇔ (p−2a){[(p−2b)+2b] 2 −2a[(p−2b)+2b]+4[(p−2b)+b](p−c)} = 0⇔ (p−2a)(p−2b)[(p−2b)−2a+4(p−c)] = 0⇔ (p−2a)(p−2b)(p−2c) = 0⇔ (b+c−3a)(c+a−3b)(a+b−3c) = 0.Prin urmare, N este pe cercul înscris dacă şi numai dacă b+c = 3a sau c+a = 3bsau a+b = 3c, ceea ce încheie demonstraţia.133


Colegiul Naţional ”Gheorghe Roşca Codreanu”din BârladComisul Gheorghe Roşca Codreanu (Codrianu) s-a născut pe 10 martie 1805şi s-a stins din viaţă la Viena, foarte tînăr, pe 19 noiembrie 1837 (multă vreme s-acrezut că de tuberculoză; cercetări mai recente par să indice un cancer). Din motivefinanciare, nimeni nu s-a învrednicit să-l aducă în ţară, aşa că a rămas până azi îngroapa comună.Testamentul scris cu o zi înaintea morţii face dovada unei lucidităţi extreme (prinindicaţiilepecareledăpentruadministrareaaverii,platadatoriilorşiaşamaideparte)precum şi pe cea a unei mărinimii paşoptiste (de sorginte iluministă): cu limbă demoarte, Gheorghe Roşca Codreanu cerea înfiinţarea unei şcoli pentru educarea ao sută de copii de pe una din moşiile sale. Totul urma a se realiza doar din averea sa,daradministratorulacesteiaveriatărăgănatlucrurile, astfelcădorinţacomisuluiaveasă se împlinească abia la 1846, când s-a înfiinţat la Bârlad Clasul Real al fundaţieiCodrianului ce avea să funcţioneze în această formă până în 1858. Aici se studiaugramatica, aritmetica, geografia, catehismul şi, după dorinţa expresă a fondatorului,limba latină.Condiţiile materiale de care dispunea şcoala erau precare, dar pasul înainte făcutprin înfiinţarea ei este unul de necontestat. Trebuie să menţionăm că Neculai RoşcaCodreanu, fratele lui Gheorghe, a lăsat şi el, tot prin testament (la 30 ianuarie 1854)o sumă importantă de bani pentru creşterea învăţământului bârlădean: şase mii de134


galbeni pentru înfiinţarea unei şcoli de fete (după acumulareaunui capitaldin dobînzi- remarcăm din nou luciditatea şi chibzuinţa acestor oameni luminaţi) şi două mii degalbeni ”din al cărora venit să ţie doi profesori cu două catedre, una de limba francezăşi una de limba italiană” la ”Clasul Codrian” fondat de către Gh. Roşca Codreanu.”Binefacerea şi dorinţa de a vedea patria mea pusă pe o cale de fericire mai statornicăau fost, din frageda mea copilărie şi va fi, până în cel din urmă minut al vieţii mele,singura ideie ce mă va îndeletnici” scrie N. RoşcaCodreanu întestamentul său. Căci:”averea, ranguri şi măriri, toate se şterg şi pier ca sclipirea fulgerului în nouri”.Imperativuleconomic(înprimulrând)cereadezvoltarepetoateplanurile; Bârladulera la acea vreme un puternic centru comercial, cu oameni care înţelegeau că şcolimai bune înseamnă, finalmente, o viaţă mai bună.La trecerea sa prin Bârlad, în 1849, după ce ascultă o alocuţiune a profesoruluiClasului Codrian, Iosif Popescu-Patriciu, domnitorul Grigore Ghica a promisînfiinţarea unui gimnaziu la Bârlad. Promisiunea se va materializa destul de târziu,gimnaziul a fost pornit abia la 1858 şi a durat până în 1864. În 1859 avea să ia fiinţăşi la Botoşani un aşezământ asemănător, care va deveni Colegiul Naţional ”A. T.Laurian”. Există referinţe din acea vreme care arată că elevii acestor două gimnaziierau silitori şi primeau pentru aceasta burse din partea Ministerului Instrucţiunii şiCultelor. Merită să observăm că gimnaziul (ca şi clasul, ca şi liceul şi colegiul care auurmat) a fost unul din primele din ţară.În 1860, prin ordonanţă a domnitorului Alexandru Ioan Cuza se încuviinţeazăca Gimnaziul să poarte numele Codreanu, al părinţilor săi fondatori. Ca să ne facemo idee, iată ce se studia la matematică în cei patru ani ai gimnaziului: aritmetica înclasaI,algebra înclasaaII-a, algebra şigeometria plană într-aIII-a, iar înclasaaIV-astereometria (geometria în spaţiu) şi trigonometria (inclusiv trigonometria sferică).Legea instrucţiunii publice din 1864 transformă gimnaziul în liceu (şapteclase, spre deosebire de patru). Materia la matematică se continua în clasa a V-a:repetiţii generale cu aplicarea algebrei în geometrie, a VI-a: teoria funcţiunilor,geometria analitică cu două dimensiuni şi a VII-a: geometria practică.Din nou necesitatea reformării şi înnoirii se făcea simţită: se dorea ca profesoriisă nu mai predea mai multe discipline şi ca elevii să nu mai stea înghesuiţi în clase,în condiţii precare. Doctorul Constantin Codrescu (membru corespondent al Societăţiide Medicină din Paris, de al cărui nume se leagă şi construirea a ceea ceavea să fie multă vreme Spitalul de adulţi din Bârlad) a fost cel care a prezentat, în1881, Camerei Deputaţilor memoriul care solicita o clădire nouă pentru liceu. Memoriulnu a fost aprobat iniţial, dar în cele din urmă s-a trecut la construcţia actualeiclădiri, în 1885. La 27 aprilie 1886 aceasta a fost inaugurată, în prezenţa ministruluiînvăţământului D. A. Sturza şi a secretarului general Spiru Haret. Construcţia acostat 230000 de lei.Dumitru Bagdasar, Nicolae Bagdasar, Constantin Hamangiu, Garabet Ibrăileanu,Vasile Pârvan, Alexandru Philippide, Ştefan Procopiu, Theodor Angheluţă, Ioan Barbălată,Vasile Cruceanu, Mihai Botez, Cezar Coşniţă, Anton Davidoglu, Nicolae Donciu,Bogdan Ionescu, Emanoil Gaiu, George Teiler, Valeriu Alaci, Virgil Claudian,Radu Miron, Gheorghe Luca, Dorian Spulber -iatăoînşiruireaunei(foarte)micipărţia mulţimii marilor oameni care au trecut prin Liceul ”Gheorghe Roşca Codreanu” în135


această vreme (ca elevi sau profesori). (M-am rezumat, din pricina lipsei de spaţiu,doar la menţionarea numelor câtorva academicieni şi/sau profesori universitari dindomeniul matematicilor. Scriitorul Alexandru Vlahuţă şi pictorul Nicolae Tonitza aufost, de asemenea, elevi ai liceului. Monografia [1] menţionează - în 1971 - 24 deacademicieni şi membri corespondenţi ai Academiei, şi cam două sute de profesoriuniversitari, scriitori, oameni de artă, generali, alte personalităţi.) Se vede dar că în1946, cu ocazia centenarului, liceul şi-a primit pe merit numele şi titlul de ColegiulNaţional ”Gheorghe Roşca Codreanu”.Din păcate nu pentru mult timp. Tăvălugul comunismului a măturat totul încalea sa: nu numai numele, dar şi statutul şcolii a fost radical schimbat. Între 1948şi 1989 şcoala aceasta (cu o tradiţie de necontestat) a fost pe rând: Liceul de băieţipână în 1952, Şcoala medie de băieţi nr. 2 între 1954-1956, Şcoala medie mixtă nr. 1între 1956-1958, Şcoala generală de 7 ani nr. 1 din 1958 până în 1962 (perioadă încare toate şcolile medii din oraş au fost ”unificate” sub titulatura Complexul şcolar,probabil prin analogie cu complexele agrozootehnice). În fine, din 1962 redevineŞcoala medie nr. 2, în 1965 dând iar promoţie de liceu. Prin anii ′ 80, când eramelev al şcolii, se numea Liceul de matematică-fizică ”Gheorghe Roşca Codreanu”. Din1996s-arevenitla denumireadeplin meritată, dobândită în1946, de Colegiul Naţional”Gheorghe Roşca Codreanu”.”[...] Singure numai virtutea şi binefacerea sunt netrecătoare, căci ele ne urmeazăşi dincolo de mormânt. [...] Singura mea plăcere era de a vorbi adeseaori cu oameniiînvăţaţi şi experienţi; a le face mii de întrebări asupra acelor lucruri pe care, dinnenorocirea mea şi întâmplările venite asupra casei părinteşti nu le cunoşteam defel;căci, amar omului care trăind în lume nu cunoaşte ceea ce ar trebui să cunoascăfieşte care om, şi vai de acei părinţi ce nu se îngrijesc de buna creştere a copiilorlor. [...] Adeseori când auzeam vorbindu-se de popoare şi drepturile omenirii, despreinstituţiile şi civilizaţia ţărilor străine, gândeam la nenorocita stare a naţiei mele,dorind îmbunătăţirea soartei ei. Ideile acestea reînviindu-mi nădejdea că, poate, încurând şi Moldova să se bucure de o soartă mai ferice, mă făcură a mă ocupa cumai multă seriozitate despre planul ce de mult timp hrăneam în inima mea, de-a-miregularisi averea în folosul obştesc al naţiei mele.”Am citat iarăşidin testamentul lui Neculai Roşca Codreanu, avânddoar regretulde a nu putea cuprinde în acest colţ de revistă mai mult din ideile şi de a nu puteasurprinde mai mult din sufletul celor doi fraţi atât de generos întemeietori ai uneişcoli uimitoare prin vechime, tradiţie şi calitatea celor ce s-au perindat în timp prinspaţiul său cultural. Pe toţi aceştia am face bine să nu-i uităm niciodată; începând,desigur, cu Gheorghe şi Neculai Roşca Codreanu.Bibliografie1. T. Nicola - Liceul ”Gheorghe Roşca Codreanu” Bârlad. Monografie, Iaşi, 1971.Prof. Marian TETIVAColegiul Naţional ”Gheorghe Roşca Codreanu”136


Clasa a VII-aConcursul de matematică ”Al. Myller”Ediţia a IX-a, Iaşi, 2 aprilie 2011Problema 1. Fie p un număr natural prim. Determinaţi numărul perechilor(x,y) de numere naturale care verifică egalitateaÔx 2 +p 4 = y.Problema 2. Fie mulţimileA =(x;y) | 0 < x ≤ y < 1;x+y ≥ 1;xy ≤ 1 4,B =(x;y) | 0 < x ≤ y < 1;x+y ≤ 1;xy ≥ 1 4.a) Să se arate că mulţimea A are cel puţin 2011 elemente.b) Să se determine mulţimea B.Problema 3. a) Fie ABCD un trapez cu bazele [AB] şi [CD], în care {O} =AC∩BD şi {Q} = AD∩BC. Demonstraţi că dreapta OQ conţine mijloacele bazelortrapezului.b) Se consideră triunghiul dreptunghic ABC cu m(∢BAC) = 90 ◦ şi AB < AC.PuncteleM şiN suntsituate pe laturile[AB] şi respectiv[AC] astfel încâtAM·AB =AN ·AC. Paralela prin punctul M la dreapta BC intersectează [AC] în punctul P.Dacă {O} = BP ∩CM, arătaţi că AO⊥MN.Problema 4. În interiorul unui pătrat de latură 1 se află un patrulater convexde arie 1 . Demonstraţi că există o dreaptă d paralelă cu una dintre laturile2pătratului, care intersectează patrulaterul şi determină cu laturile acestuia un segmentcu lungimea mai mare sau egală cu 1 2 .Clasa a VIII-aProblema 1. Determinaţi numerele reale a cu proprietatea căa 2 +2a−33 +a 2 −2a−153 = 8a 2 −93 .Problema 2. Se consideră pătratul ABCD de latură 1. Punctele M,N,P şiQ sunt situate pe laturile (AB),(BC),(CD) şi respectiv (DA). Demonstraţi căperimetrul patrulaterului MNPQ este mai mare sau egal cu 2 √ 2.Problema 3. Fie x un număr iraţional. Se ştie că 36 este cel mai mic numărnatural nenul cu proprietatea că numărul x 36 este raţional. Determinaţi numărul deelemente raţionale din mulţimea A = {x ab | a+b = 36,a,b ∈ N\{0}}.137


Problema 4. Fie ABCA ′ B ′ C ′ o prismă triunghiulară dreaptă. Arătaţi că, dacădreptele A ′ B,B ′ C şi C ′ A sunt perpendiculare două câte două, atunci AB = BC =CA = AA ′ ·√2.Clasa a IX-aProblema 1. Fie K,L,M,N mijloacele laturilor (AB), (BC), (CD), respectiv(DA) ale patrulaterului ABCD înscris într-uncerc de centru O. Notăm H A , H B , H C ,H D , ortocentrele triunghiurilor AKN, BLK, CML, respectiv DNM.a) Arătaţi că 2 −−−→ OH A = 2 −→ OA+ −→ −−→ OB + OD.b) Arătaţi că H A H B H C H D este paralelogram.Problema 2. Determinaţi numerele întregi a,b pentru care sistemulx= 2 −2ax−a−2 = 0y 2 −2by−x = 0are exact trei soluţii în mulţimea numerelor reale.Problema 3. Fie (a n ) n∈N un şir de numere reale definit prin a 0 ∈ (0,1) şi0 , dacă a n = 0a n+1Òn+1 .dacă a n ≠ 0a nÓ,Arătaţi că a 0 este iraţional dacă şi numai dacă şirul nu are termeni nuli.Problema 4. Determinaţi toate funcţiile f : R → R care au proprietatea că(x 2 +xy +y 2 )(f(x)−f(y)) = f(x 3 )−f(y 3 ), pentru orice x,y ∈ R.Clasa a X-aProblema 1. Să se determine numerele complexe z cu proprietatea că |z|+|z−25|+|z −18−24i|+|z +7−24i| = 70.Problema 2. Considerăm un punct M în interiorul paralelogramului ABCD. Săse arate că MA·MC +<strong>MB</strong> ·MD AB ·AD.Problema 3. Un şir de numere reale (a n ) n1 se numeşte convex dacă2a n a n−1 +a n+1 , oricare ar fi n ∈ N,n 2.a) Să se dea un exemplu de şir convex neconstant.b) Fie (b n ) n1 un şir de numere reale pozitive astfel încât, oricare ar fi a > 0, şirul(a n b n ) n1 să fie convex. Să se demonstreze că şirul (lnb n ) n1 este convex.Problema 4. Să se determine funcţiile f : N → N care verifică relaţia f(f(n))+f(n) = 6n, oricare ar fi n ∈ N.138


Clasa a XI-aProblema 1. Fie n un număr natural nenul şi fie A o matrice pătratică de ordinn cu elemente întregi, având proprietatea că I n +A+A 2 +···+A 10 = O n .i) Să se arate că matricea I n +A+A 2 este inversabilă.ii) Să se arate că det(I n +A+A 2 ) = 1.Problema 2. Considerăm un şir (a n ) n≥1 de numere reale, strict descrescător şiconvergent la 0, cu proprietatea că a n+1 a n +a n+2á. Să se arate că lim2n(a n+1 −n→∞a n ) = 0.Problema 3. Fie n un număr natural nenul. Să se arate că există numerele realex 1 , x 2 ,...,x n astfel încât matricea1 sin2x 1 sin3x 1 ... sinnx 12 sin2x 2 sin3x 2 ... sinnx 2A n = (sinpx q ) 1p,qn =sinx. . . .. . . ....sinx n sin2x n sin3x n ... sinnx nsă fie nesingulară.Problema 4. Fie f : [0,∞) → R o funcţie crescătoare şi convexă, cu proprietateacă lim (f(x) − x) = 0. Să se demonstreze că pentru orice număr x ∈ [0,∞) avemx→∞2x f(f(2x)) 2f(x).Clasa a XII-aProblema 1. Fie I ⊂ R un interval şi fie f : I → R o funcţie integrabilă pe=yorice interval [a,b] ⊂ I. Să se arate că mulţimea A f(t)dt | x,y ∈ Ieste unxinterval, eventual degenerat.Problema 2. Fie p un număr prim, p > 2. Să se arate că polinomul cu coeficienţiîntregi f = (X −1)(X −2)···(X −p)+X +p este ireductibil în Z[X].Problema 3. Se consideră funcţia continuă f : [0,1] → R cu proprietăţile:a) f(0) < 0 < f(1);b) există un unic număr c ∈ (0,1) pentru care f(c) = 0;c)1f(x)dx 1xf(x)dx.00Să se demonstreze că pentru orice număr n natural nenul avem1x n f(x)dx > 0.Problema 4. Fie A un inel finit cu proprietăţile:i) orice divizor al lui zero este element nilpotent;ii) numărul elementelor inversabile din A şi numărul automorfismelor inelului Asunt relativ prime.Să se arate că:1 ◦ Pentru orice element inversabil a ∈ A, funcţia f a : A → A, f a (x) = axa −1 esteun automorfism al inelului A;2 ◦ Inelul A este comutativ.1390


Concursul de matematică ”Florica T. Câmpan”Ediţia a XI-a, Iaşi, 2011Etapa judeţeană, 20 februarie 2010Clasa I1. Găsiţi regula şi completaţi cu numerele care lipsesc:10 6 5 3 ? 4 9 ?a)4 2 3 4b)0; 2; 4;2; 4; 6;4; 6; 8;□; □; □;□; □; □.c) 1,2,4,3; 5,6,8,7; □,□,□,□.2. Bogdan are cu 3 timbre mai mult decât Radu, iar Radu are cu 10 mai puţinedecât Mihai, care are 20 de timbre. Câte timbre are Bogdan?3. Cătălin se joacă în scara blocului astfel: urcă 3 trepte, coboară două, urcă apoi4 trepte, coboară una, urcă 5 trepte şi coboară două. Câţi paşi a făcut Cătălin şi câtetrepte a urcat?Clasa a II-a1. a) Completaţi în toate modurile posibile dreptunghiul alăturat cu numerediferite de 0, astfel încât suma numerelor pe fiecare linie şi pe fiecare3coloanăsăfieegalăcunumărulindicatîndreptulliniei, respectivcoloanei.5b) Într-o şcoală, elevii intră pe baza unui cod <strong>format</strong> din trei cifrenenule distincte, fără a conta ordinea în care sunt introduse cifrele. Întrunadin zile, Claudiu uită cele trei cifre care îi trebuie pentru a intra în 4 106şcoală, însă ţine minte că suma celor trei cifre este 12. Aflaţi toate valorile posibilepe care le poate avea codul de acces al lui Claudiu în şcoală.2. a) După ce Diana a mâncat 6 mere, iar Elena a mâncat 7 mere, le-au mairămas fiecăreia atâtea mere câte au mâncat la un loc. Câte mere a avut fiecare?b) Rareş a desenat 24 de figuri geometrice, pătrate şi cercuri, astfel încât fiecarepătrat să conţină trei cercuri. Ştiind că în fiecare cerc desenează câte un trandafir şiîn fiecare pătrat câte două garoafe, aflaţi câte flori a desenat Rareş.3. Vlăduţ aruncă şase săgeţi la ţinta din figura alăturată şi nimereşte de fiecare140


dată într-unuldincercurileei, câştigândatâteapuncte câtesuntscrise în fiecare cerc.a) Care este cel mai mic scor pe care îl poate obţine Vlăduţ?b) Care este cel mai mare scor pe care îl poate obţineVlăduţ?c) Indicaţi toate modurile posibile prin care poate obţinescorul 10.Clasa a III-a1. a) Aflaţi numărul de două cifre căruia îi aparţine replica: ”Diferenţa dintreprima şi a doua mea cifră este 1 iar suma lor este9”. Justificaţi!b) La petrecerile din Împărăţia Numerelor iauparte doar numerele majore, adică cele mai maridecât 50. Cifra 7 doreşte insistent să participe lapetrecere; Împăratul Zero îi dă o şansă şi o supuneunei probe: ”În circuitul din figura alăturată trebuiesă porneşti de la START, să efectuezi câtevadintre operaţii, în ordinea arătată de săgeţi şi săobţii un rezultat mai mare de 50. Doar astfel tevoi primi la petrecere!” Astfel, cifra 7 intră în circuit, alege un drum şi obţine laSTOP rezultatul 55. A câştigat! Puteţi voi găsi la ce operaţii a fost supusă cifra 7când a parcurs circuitul?2. a) La o masă pătrată pot sta exact 4 copii, câte unul pe fiecare latură. La oserbare s-au aşezat şapte mese, lipite una lânga alta, astfel încât să formeze o masălungă, dreptunghiulară. Câţi copii se pot aşeza la masă <strong>format</strong>ă?b) Copiii din familiile Apostol şi Bălan s-au întâlnit într-o cofetărie, pentru asărbători o aniversare. Este posibil ca fiecare băiat să aibă exact trei fraţi şi fiecarefată să aibă exact o soră în acea cofetărie? Câţi copii ar fi atunci, de toţi? (Fiecarefamilie are drept copii şi băieţi şi fete.)3. Bancomatul jucărie din şcoala mea îmi permite să extrag ”sume” doar înjetoane de 2 euro şi de 5 euro. Pot scoate cel mult 10 jetoane de 2 euro şi cel mult 5jetoane de 5 euro.a) Care este ”suma” cea mai mare pe care o pot extrage din bancomat?b) Descrieţi un mod de a extrage exact 17 euro.c) Este posibil să scot din bancomat, dintr-o singură extragere, exact 42 euro?Clasa a IV-a1. Moş Martin avea în toamna anului 2010 o greutate de 350 de kilograme. Ştiindcă vara se îngraşă cu 20 kg, iar iarna slăbeşte cu 10 kg, aflaţi ce greutate avea MoşMartin în toamna anului 2007.Doina Nechifor2. Monograma unei persoane este reprezentată de trei litere: iniţiala numeluide familie, iniţiala primului prenume şi iniţiala celui de al doilea prenume (literele141


alfabetului se consideră a fi A,B,C,D,E,F,G,H,I,J,K,L,M,N,O,P,Q,R,S,T,U,V,W,X,Y,Z). Domnul şi doamna Câmpan doreau să pună un nume bebeluşuluiCâmpan astfel încât literele din monograma lui să fie diferite şi în ordine alfabetică.Câte astfel de monograme există?Alexandru Negrescu3. Pe planeta Nintendo, pokémonii sunt de patru feluri: de apă, de pământ, defoc şi de noapte. Ei pot avea de la 5 la 7 aripi şi de la 4 la 21 antene. Împăratullor, Lucian-Georges, vrea să pornească un război împotriva lui Katalin, cumplitulsău inamic. Care este numărul minim de pokémoni din armata ce atacă împărăţialui Katalin, dacă Lucian-Georges vrea să aibă certitudinea că va putea selecta uncomando <strong>format</strong> din 21 de pokémoni identici? (Doi pokémoni se consideră identicidacă sunt de acelaşi fel, au acelaşi număr de aripi şi acelaşi număr de antene.)Clasa a V-a1. Cătălin şi Doru au fost în vacanţă în Egipt. În ziua în care au sosit înapoiei s-au îmbolnăvit de gripă nouă (AH1N1), contaminând multe dintre persoanele cucare au intrat în contact. În fiecare zi, numărul oamenilor care s-au îmbolnăvit estede trei ori mai mare decât în ziua precedentă.a) Câţi oameni se vor îmbolnăvi în primele 100 de zile?b) Arătaţi că numărul total al bolnavilor din primele 100 de zile este mai mare ca2 150 .Ciprian Baghiu2. La o lucrare de control, cei 29 de elevi ai unei clase primesc spre rezolvare unset de trei întrebări. Dacă răspund corect la întrebarea I primesc 1 punct, pentru aII-a întrebare primesc 2 puncte, pentru a III-a întrebare primesc 3 puncte, iar dacănu dau niciun răspuns corect primesc 0 puncte.a) Arătaţi că cel puţin cinci elevi au obţinut acelaşi punctaj.b) Se păstrează concluzia dacă în acea clasă sunt 28 de elevi?Cătălin Budeanu3. Suma a 30 de numere naturale pare şi nenule este 328.a) Daţi un exemplu de numere ce îndeplinesc condiţiile de mai sus, printre care săexiste exact o grupă cu patru termeni egali.b) Arătaţi că oricumam alege numerecare satisfactoate condiţiile problemei, vomavea cel puţin patru numere egale între ele.Ciprian BaghiuClasa a VI-a1. Păcalăşi Tândalăau primit bonuri valoricede 11 lei bucata. Într-un supermarketPăcală a cumpărat două pâini, a mâncat 7 cârnciori şi a băut un pahar cu mustiar Tândală a cumpărat 3 pâini, a mâncat 5 cârnăciori şi a băut 7 pahare cu must.Păcală a plătit cu un număr întreg de bonuri valorice, fără a primi rest. Arătaţi că şiTândală poate achita plata totală la fel.142


2. Ana desenează pe tablă figura alăturată. Maria alege zece numere dinmulţimea {0;1;2;...;14} şi le scrie în cercurile din desen. Luizascrie pe fiecare segment din desen diferenţa numerelor din cercurilepe care acesta le uneşte (diferenţele sunt numere naturale). Esteposibil ca numerele de pe segmente să fie distincte? Justificaţi!Andrei Nedelcu3. Elevii Andrei, Bogdan şi Costel joacă ping-pong. Cel carepierde un set lasă loc la masă celui care s-a odihnit. În final Andreia jucat 13 seturi iar Bogdan 27. Câte seturi a jucat Costel?Gheorghe IureaClasa a VII-a1. La un club sportiv sunt înscrişi mai puţin de 70 de elevi. O treime din numărulfetelor reprezintă un sfert din numărul băieţilor. Unii copii joacă volei, ceilalţi joacăhandbal. Un sfert din numărul celor care joacă volei reprezintă o cincime din numărulcelor care joacă handbal. Ştiind că 17 fete joacă handbal, aflaţi câţi băieţi joacă volei.Gazeta Matematică 11/20102. Despre un număr întreg a vom spune că are valenţa n dacă există exact ntripletedenumereîntregi(x,y,z)astfelîncât−5 ≤ x ≤ 10,−5 ≤ y ≤ 10,−5 ≤ z ≤ 10şi x−2y +3z = a. Determinaţi valenţele numerelor 50, −50 şi 0.Claudiu Ştefan Popa şi Gabriel Popa3. Un copil de clasa a VII-a găseşte un document îngălbenit de vreme care aratălocul unde a fost îngropată o comoară pe o insulă: ”Caută turnul bisericii T, cascadaC, stejarul bătrân S şi stânca diavolului D. Înfige un ţăruş M la mijlocul drumuluidrept dintre T şi D şi încă unul N la mijlocul drumului drept dintre C şi S. Uneşteprin linii drepte M cu N şi T cu S, marcând locul în care aceste linii se întâlnesc (E).Mergi de la M la E, numărându-ţi paşii, apoi numără tot atâţia paşi în prelungirealiniei MN, începând din N, şi înfige un ţăruş în locul P în care ai ajuns. Comoarase află acolo unde linia dreaptă prin C şi D întâlneşte linia dreaptă prin T şi P.”Ajuns pe insulă, copilul află de la băştinaşi că stejarul bătrân a fost doborât de untrăsnet, cu mulţi ani în urmă. Găseşte însă o hartă veche pe care se vede că TCSDera un trapez cu baza mare TC şi DS = SO, unde {O} = ST ∩CD.a) Arătaţi că ST = DS +TC.b) Demonstraţi că unghiul SPT este drept.c) Ajutaţi-l pe elev să găsească locul comorii, chiar în absenţa stejarului bătrân!Claudiu Ştefan Popa şi Gabriel PopaClasa a VIII-a1. Ali-Baba şi ai lui 40 de hoţi au strâns de-a lungul timpului o sumă imensă debani: abcde256 euro. Pentru că Ali-Baba se temea de hoţi, a pus suma într-un seif lao bancă în Elveţia. Un agent sub acoperire, infiltrat de foarte multă vreme în bandalui Ali-Baba, era cunoscut ca rapper de succes cu numele M-One. Acesta a aflat înscurt timp combinaţia de la seif şi a scris următorul e-mail către colegii din poliţie:143


∗∗∗ cde < 256∗ ∗ ∗ În fiecare căsuţă este un pătrat perfect şi numărul din fiecare căsuţă estesuma numerelor peste care se află∗∗∗ La vârf este codul:Sunt pătrat de pătratNatural, adevăratCel mai mare din o mieToată lumea mă ştie.Cei din poliţie nu s-au descurcat cu mesajul şi l-au trimis la Concursul FloricaT. Câmpan, sperând ca unul dintre elevi să dezlege enigma. Aflaţi pentru ei codulseifului şi suma din seif.Marian Panţiruc2. În vârfurile unui cub se aşează numere naturale şi pe fiecare muchie se aşeazămedia aritmetică a numerelor din capete, de asemenea număr natural.a) Pot fi cele 20 de numere pare şi distincte două câte două?b) Pot fi cele 20 de numere impare şi distincte două câte două?c) Demonstraţi că nu putem aşeza în acest fel 20 de numere consecutive.Julieta Grigoraş3. Un chioşc de ziare are forma unei prisme patrulatere regulate DIFUZARE,cu latura bazei de 2m şi înălţimea de 3m. Chioşcul estealimentat cu electricitate printr-un fir ce uneşte vârfulV al unui stâlp VO înalt de 4m cu mijlocul laturii DU(evident, fără a străpunge chioşcul). Ştiind că puncteleD,I,O sunt coliniare şi IO = 4m, aflaţi lungimea celuimai scurt fir care poate alimenta chioşcul cu energie electrică.Marian PanţirucClasa a IV-aEtapa interjudeţeană, 26 martie 20111. În timpul vacanţei, Claudiu îşi ajută părinţii lucrând la magazinul familiei.Într-una din zile, la magazin se aduc cutii de compot care au înălţimea de 10 centimetri.Claudiu trebuie să le aranjeze pe o masă unele peste altele: pe primul rândde jos pune 12 cutii, pe rândul al doilea 11 cutii, pe următorul rând 10 cutii şi aşamai departe.144


a) De câte cutii de compot are nevoie Claudiu pentru aranjamentul de pe masă?Ce înălţime va avea acest aranjament?b) Pentru o altă aranjare a cutiilor, Claudiu se hotărăşte să pună 12 cutii peprimul rând de jos, 10 cutii pe al doilea rând şi aşa mai departe, până ce pune douăcutii pe ultimul rând de sus. De câte cutii are nevoie Claudiu? Ce înălţime va aveaacest nou aranjament?2. În pătratul alăturat, produsul numerelor de pe fiecare linie, de pe fiecarecoloană şi de pe fiecare dintre cele două diagonale este acelaşi şi nenul.2 a ba) Aflaţi b.9 6 cb) Dacă, în plus, 9f +2d = 144, aflaţi şi celelalte numere.d e f3. Un număr care nu se împarte exact la niciuna din cifrele sale se numeşte numărcivilizat (precizăm că niciun număr nu se împarte la 0).a) Arătaţi că numerele 52 şi 354 nu sunt civilizate.b) Claudiu şi Diana au găsit două numere civilizate careînmulţite dau ca rezultat tot un număr civilizat.Reconstituiţi înmulţirea găsită de cei doi copii (steluţele înlocuiesc cifre).Clasa a V-a∗ 2 3 ∗∗ 9∗ ∗ ∗ ∗1. O carte ciudată are paginile numerotate astfel: 16,23,30,37,44,...,2011.a) Aflaţi câte foi are cartea ciudată.b) Determinaţi câte cifre s-au folosit pentru numerotarea paginilor cărţii ciudate.c) Calculaţi suma numerelor înscrise pe foaia din mijlocul cărţii.2. Monica este o învăţătoare foarte serioasă şi iubită de copiii pe care îi învaţă. Înclasalacarepredăare10copiipregătiţipentruconcursuri,dintrecare7lamatematicăşi 6 la limba română. La sfârşitul unei săptămâni au loc trei concursuri: sâmbătăunul de matematică şi unul de limba română (la aceeaşi oră), iar duminică unul de ßÞ Ð2011 cifre de 0matematică. La fiecare concurs ea trimite câte un singur copil. În câte moduri poatealege participanţii la cele trei concursuri?Ciprian Baghiu3. a) Arătaţi că numărul 1006009 este pătrat perfect.b) Demonstraţi că există măcar 2011pătrate perfecte care nu au ultima cifră egalăcu 0 şi care conţin un număr impar de cifre de 0.c) Arătaţi că, oricare ar fi cifra b nenulă, numărul b 000...00 bnu este pătratperfect.Cristian LazărClasa a VI-a1. Construim succesiv dreptunghiuri vecine,alternativ spre dreapta şi în jos, ca în figuraalăturată. (Două dreptunghiuri se numesc vecinedacă au o latură comună.) Laturile dreptunghiurilorau consecutiv lungimile 1 1 , 1 2 , 1 3 , 1 4 , 1 5 ,....1 /21 1/31/41/51/6...145


Arătaţi că există o grupă de două sau mai multe dreptunghiuri vecine două câtedouă astfel încât suma ariilor acestora să fie 1 3 .2. Pe o tablă de şah 8×8 se aşează 8 turnuri astfel încât niciunul să nu le atacepe celelalte.a) Arătaţi că un astfel de aranjament este posibil.b) Pentru un astfel de aranjament, arătaţi că în orice pătrat 5×5 există cel puţindouă turnuri.3. Demonstraţicăpotfialesecelmult671numeredinmulţimeaA = {1,2,...,2011}astfel încât diferenţa oricăror două numere alese să nu dividă suma acestora.Clasa a VII-a1. Ioana desenează pe monitorul calculatorului, cu ajutorul unui program degrafică pe computer, un triunghi isoscel ABC şi ia punctele D pe baza (BC) şi E pelatura (AB) astfel încât ∡ADE ≡ ∡ACB. Programul folosit îi permite Ioanei să selectezeoricetriunghicareapareîndesen,să-lmăreascăsausă-lmicşoreze(păstrându-iforma) cu funcţia zoom, să-i schimbe poziţia sau să-l rotească, după dorinţă. Numimtransformare o succesiune oarecare de astfel de operaţii.a) Demonstraţi că Ioana poate găsi o transformare înurma căreiatriunghiul DBEsă se suprapună exact peste triunghiul ACD.b)Observândaltetransformăriurmatedesuprapuneridetriunghiuriînconfiguraţiadesenată, Ioana redescoperă relaţia lui Stewart pentru triunghiul isoscel: AB 2 =AD 2 +BD ·CD. Demonstraţi şi voi această relaţie!Claudiu Ştefan Popa2. Se consideră numerele raţionale strict pozitive a,b,c,d astfel încât a b = c d .a) Arătaţi că √ ac ∈ Q dacă şi numai dacă √ bd ∈ Q.b) Demonstraţi că(a+b)(c+d) ∈ Q dacă şi numai dacă √ ac ∈ Q.c) Este adevărată implicaţia(a+b)(c+d) ∈ N ⇒ √ ac ∈ N? Dar reciproca?Claudiu Ştefan Popa şi Gabriel Popa3. Considerăm mulţimea M = {−1,;−2;−3;...;−2011}. Pentru fiecare submulţimenevidă A a lui M, calculăm produsul P A al tuturor elementelor sale; dacăA = {a}, atunci P A = a.a)Găsiţi treisubmulţimidistincte A,B şiC alelui M,pentrucareP A = P B = P C .b) Care este valoarea cea mai mare pe care o poate lua un astfel de produs? Darcea mai mică?c) Calculaţi suma tuturor produselor care se obţin. (Dacă valoarea unui produsse repetă pentru mai multe submulţimi ale lui M, respectiva valoare se va repeta deacelaşi număr de ori şi în sumă.)Clasa a VIII-a1. Baronul de Münchhausen spune o obişnuită poveste gogonată despre cum azburat el pe Lună călarepe o ghiulea trasădintr-un tun. Oricine şi-ar putea da seama146


că este o uriaşă minciună, însă puţini puteau şi demonstra acest lucru. În cele dinurmă, un copil isteţ de vreo15 ani a reuşit să afle, studiind diversespecificaţii tehnice,că viteza unei ghiulele ar fi de 200m/s şi, conform legilor fizicii, după t > 0 secundede mişcare, ghiuleaua se află la înălţimea h(t) = −5t 2 +200t. Aflaţi şi voi înălţimeamaximă la care ar fi putut ajunge baronul Münchhausen şi timpul total al călătorieisale călare pe ghiulea.Marian Panţiruc2. Fie prisma triunghiulară regulată A 1 A 2 A 3 B 1 B 2 B 3 având toate muchiile, precumşi diagonalelefeţelor laterale, coloratecu roşu sau cu albastru. În fiecare triunghicare se formează, exceptând bazele, există atât o latură roşie cât şi o latură albastră.Arătaţi că cele şase muchii ale bazelor sunt colorate toate cu aceeaşi culoare.Gheorghe Iurea3. Lucian şi Cătălin au vândut împreună, la piaţă, n kilograme carne de curcan,cu valoarea de n euro kilogramul. Ei acţionează succesiv: primul îşi laudă marfaLucian (până vinde carne în valoare de 10 euro), apoi Cătălin (până vinde şi el învaloare de 10 euro), din nou Lucian etc. Cătălin câştigă la ultima strigare o sumăîntreagă, mai mică de 10 euro. Determinaţi acestă sumă.Doru Buzac şi Gabriel MîrşanuNume de români date unor corpuri cereştisau unor forme de relief pe acesteaDenumirea corpurilor cereşti (stele, planete, asteroizi, comete etc.) şi a oricăreiforme de relief pe acestea este decisă de Uniunea Astronomică Internaţională(UAI), cu sediul la Paris. În timp ce cometele poartă numele descoperitorilor lor,corpurile şi formele de relief poartă numele unor personalităţi din lumea artelor şiştiinţei.1. Nume româneşti pe planete• Craterul Haret (Spiru Haret, 1851-1912) pe faţa invizibilă a Lunii, cudiametrul de 29,77 km;• Craterul Eminescu pe Mercur, cu diametrul de 125 km;• Craterul Văcărescu (Elena Văcărescu, 1866-1947) pe Venus, cu diametrulde 315,5 km;• Patera Darclée (Hariclea Darclée, 1860-1939) pe Venus, formaţiune vulcanicănumită ”patera” cu diametrul de 15 km;(continuare la pag. 75)147


Soluţiile problemelor propuse în nr. 2/2010Clasele primareP.196. Mioara aranjează patru mărgele, două albe şi două galbene, una lângă alta,pe o aţă. În câte feluri poate aranja Mioara mărgelele?(Clasa I)Inst. Maria Racu, IaşiSoluţie. Mioara poate aranja mărgelele în şase moduri: AAGG, GGAA, GAGA,AGAG, AGGA, GAAG.P.197. Dan împarte o ciocolată astfel: o linie întreagă de 4 pătrăţele pentrufratele său, o coloană întreagă de 4 pătrăţele pentru sora sa, iar restul pentru sine.Câte pătrăţele de ciocolată i-au revenit lui Dan?(Clasa I)Maria Ursu, elevă, IaşiSoluţie. Iniţialciocolataavea4coloaneşi 5linii. Danarămascu 3+3+3+3= 12pătrăţele de ciocolată.P.198. Număraţi figurile geometrice din desenul alăturat şi scrieţi:a) numărul triunghiurilor;b) numărul figurilor geometrice care sunt pătrate sau dreptunghiuri.(Clasa a II-a) Andreea Amarandei, studentă, IaşiSoluţie. a) 5 triunghiuri; b) 5 pătrate şi 4 dreptunghiuri formează 9 figuri geometricecare sunt pătrate sau dreptunghiuri.P.199. Dacă a−15 = 15−b, poate fi diferenţa a−b un număr impar?(Clasa a II-a)Andreea Bîzdîgă, elevă, IaşiSoluţie. Din a−15 = 15−b se obţine a+b = 30, deci a−b este număr par, căcisuma şi diferenţa a două numere naturale au aceeaşi paritate.P.200. Dintr-un număr de 12 bile, 11 au mase egale, iar una are o masă maimare decât celelalte. Care este cel mai mic număr de cântăriri prin care se poatedepista bila cu masa mai mare, dacă avem la îndemână o balanţă cu două talere?(Clasa a III-a)Ana Cojocaru, IaşiSoluţie. Formăm trei grupe: 5b, 5b, 2b. Aşezăm câte 5 bile pe fiecare taler.După prima cântărire determinăm grupa care conţine bila mai grea. În continuaremai avem nevoie de maximum două cântăriri pentru a determina bila cea mai grea.În concluzie, numărul cel mai mic de cântăriri este 3.P.201. Diferenţa dintre suma vârstelor a două persoane şi diferenţa lor este 20de ani. Triplul sumei vârstelor este egal cu de 7 ori diferenţa vârstelor. Care estevârsta fiecărei persoane?(Clasa a III-a)Valeria Avasîlcei, IaşiSoluţie. Din (a +b)−(a −b) = 20 se obţine b+b = 20, deci b = 10. Utilizânda doua informaţie găsim 3(a + 10) = 7(a − 10), de unde 3a +30 = 7a−70, aşadar30 = 4a−70 şi a = 25. Vârstele sunt: 25 ani, 10 ani.148


P.202. Aflaţi numerele ab şi c, cu b ≠ 9 şi c ≠ 0, astfel încât, dacă mărim cifrelea şi b cu câte o unitate, atunci produsul ab·c se dublează.(Clasa a III-a)Amalia Munteanu, elevă, IaşiSoluţie. Dacă cifrele a şi b se măresc cu câte o unitate, atunci numărul ab devineab +11 şi condiţia problemei ne dă (ab + 11)c = 2 · ab · c, de unde 11·c = ab · c şiobţinem ab = 11, c = 1,2,3,··· ,9.P.203. Cei doi tigri de la Zoo au suficientă hrană pentru 3 săptămâni. Au maifost aduşi, însă, încă 5 tigri. Dacă fiecare mănâncă aceeaşi cantitate de hrană pe zi,câte zile le va mai ajunge acum hrana?(Clasa a IV-a)Inst. Laura Chirilă, IaşiSoluţie. Dacă 2 tigri au suficientă hrană 3 săptămâni, atunci un singur tigru aresuficientă hrană 6 săpţămâni, adică 42 zile. 7 tigri vor avea suficientă hrană numai42 : 7 = 6 zile.P.204. Aflaţi toate numerele naturale n astfel încât triplul predecesorului lui n nudepăşeşte dublul succesorului lui n.(Clasa a IV-a)Andreea Simion, elevă, IaşiSoluţie. Trebuie să avem3×(n−1) ≤ 2×(n+1), adică 3×n−3×1 ≤ 2×n+2×1,ceea ce este echivalent cu n ≤ 5 şi obţinem n = 1,2,3,4,5.P.205. Fie S suma a zece numere naturale nenule.a) Dacă S = 54, arătaţi că există cel puţin două numere egale.b)Aflaţi zece numere naturale nenule distincte pentru care S = 57. Câte posibilităţisunt? Justificaţi!(Clasa a IV-a)Constanţa Tudorache şi Nelu TudoracheSoluţie. a) Suma celor mai mici zece numere naturale nenule distincte este 1+2+3+··· +9+10 = 55. Micşorând cu o unitate unul dintre termeni, vom obţinedouă numere egale.b) În suma 1+2+3+···+9+10 = 55 putem să înlocuim pe 9 cu 11 sau pe 10cu 12 şi obţinem 10 numere naturale nenule şi distincte care au suma 57. Avem douăposibilităţi.Clasa a V-aV.123. Un spiriduş îi şopteşte Ioanei: ”Pentru a salva lumea de rău, prepară opoţiune din praf de aur şi praf de stele. În camera albastră este un dulap uriaş, cunumeroase rafturi, fiecare conţinând câte 20 de sticluţe numerotate: de la 1 la 20 peprimul raft, de la 21 la 40 pe al doilea etc. Numărul raftului cu sticluţa cu praf de aureste egal cu numărul sticluţei cu praf de stele. Suma dintre numărul sticluţei cu prafde aur şi numărul sticluţei cu praf de stele este 243”. Care sunt numerele sticluţelorpe care trebuie să le aleagă Ioana?Cristina Timofte, IaşiSoluţie. Fie x numărul sticluţei cu praf de stele; numărul sticluţei cu praf de aurva fi 20(x−1)+r unde r poate fi 1,2,··· ,20. Avem:x+20(x−1)+r = 243 ⇒ 21x+r = 263, r ≤ 20.Singura soluţie convenabilă este x = 12, r = 11. Sticluţele căutate sunt numerotatecu 12 (cea cu praf de stele), respectiv 231 (cea cu praf de aur).149


V.124. Arătaţi că numărul A = 10071·2 + 72·3 +···+ 799·100+58911·3 +13·5 +···+ 1cel mai mare număr natural de trei cifre distincte.587·589esteAnca Chiriţescu, Ţigănaşi (Iaşi)Soluţie. Calculând sumele (telescopice) din paranteze, obţinem că A = 100 ·693 294+589· = 987, care este cel mai mare număr natural de trei cifre distincte.100 589V.125. Se consideră numărul a = 1+2+3+···+2010 şi mulţimea M = {n ∈N|na = k 2 ,k ∈ N}. Determinaţi cele mai mici cinci elemente ale lui M.Nicolae Ivăşchescu, CraiovaSoluţie. Calculând suma, obţinem că a = 2010·2011 : 2 = 3·5·67·2011, undefactorii ultimului produs sunt numere prime. Cum na este pătrat perfect, rezultă căn = 3 · 5·67·2011·m 2 = am 2 , cu m ∈ N. Cele mai mici cinci elemente ale lui Msunt 0,a,4a,9a şi 16a.V.126. Se consideră mulţimea A = {1,2,3,··· ,2010}. Scrieţi mulţimea A careuniune a trei mulţimi disjuncte două câte două, având acelaşi cardinal şi aceeaşisumă a elementelor.Mirela Marin, IaşiSoluţie. Avem că A = B∪C ∪D, unde B = {6k+1|0 ≤ k ≤ 334}∪{6k+6|0 ≤k ≤ 334}, C = {6k+2|0 ≤ k ≤ 334}∪{6k+5|0 ≤ k ≤ 334} şi D = {6k +3|0 ≤ k ≤334}∪{6k+4|0 ≤ k ≤ 334}. Fiecare dintre mulţimile B,C şi D are cardinalul 670 şisuma elementelor egală cu 673685, deci sunt îndeplinite cerinţele problemei.V.127. Determinaţi numerele naturale A pentru care A + S(A) = 2010. (Amnotat cu S(A) suma cifrelor numărului A.)Cătălin Budeanu, IaşiSoluţie. Numărul A este de cel mult patru cifre, cu cifra miilor cel mult egalăcu 2, deci S(A) ≤ 2+9+9+9 = 29. Rezultă că 1981 ≤ A ≤ 2010. Verificând celetreizeci de posibilităţi, obţinem că A ∈ {1986,2004}.Evident, numărul verificărilor poate fi micşorat. De exemplu, cum A şi S(A) dauacelaşi rest la împărţirea prin 3, iar suma 2010 se divide cu 3, rezultă că A . .3 şi astfelrămân de făcut doar zece verificări.V.128. Reconstituiţi o împărţire, ştiind că împărţitorul, câtul şi restul sunt cifreale deîmpărţitului.Ioan Săcăleanu, HârlăuSoluţie. Fie D = I ·C +R, cu R < I. Cum I,C,R sunt cifre, rezultă că D ≤ 89,iar din R < I, urmează că D nu poate fi număr de o cifră. Astfel, D = ab şiI,C,R ∈ {a,b}; deducem că I = C > R sau I > C = R. În cazul în care a > b,obţinem că I = C = a, R = b, respectiv I = a, C = R = b; prima situaţie conduce la10a+b = a 2 +b ⇔ a = 10, imposibil, iar a doua conduce la 100+b= ab+b ⇔ b = 10,din nou impisibil. Rămâne că a < b, deci I = C = b, R = a sau I = b,C = R = a.În primul caz, 10a+b = b 2 +a ⇔ 9a = b(b−1), cu unica soluţie a = 8, b = 9, iar încel de-al doilea, 10a+b = ab+a ⇔ 9a = a(b−1) ⇔ b = 10, imposibil. În concluzie,împărţirea căutată este 89 = 9·9+8.150


V.129. Fie a,b,c trei numere impare, iar A = 2 a+b2 ·3 a+c2 ·5 b+c2 . Ştiind că 30A nueste pătrat perfect, arătaţi că măcar unul dintre numerele a,b,c nu este pătrat perfect.Andrei Nedelcu, Iaşia+bSoluţie. Întrucât a,b,c sunt impare, numerele x =2 , y = a+c şi z = b+c2 2sunt naturale, având suma egală cu a + b + c, deci impară. Rezultă că x,y,z suntfie toate impare, fie unul impar şi două pare. Prima situaţie nu convine, deoarecenumărul 30A = 2 x+1·3y+1·5z+1 ar avea toţi exponenţii pari, deci ar fi pătrat perfect.Rămâne că există un număr par printre numerele x,y,z (de fapt, chiar două) şi atuncidouă dintre numerele impare a,b,c dau resturi diferite (1 şi 3) la împărţirea prin 4.Acela dintre numerele a,b,c care este de forma M 4 +3 nu poate fi pătrat perfect.Clasa a VI-aVI.123. Stabiliţi în câte moduri îl putem scrie pe 2010 ca sumă de trei numerenaturale nenule, direct proporţionale cu trei numere naturale consecutive.Mirela Obreja şi Ioan Lungu, VasluiSoluţie. Fie a,b,c,n ∈ N ∗ astfel încât a+b+c = 2010 şi (a,b,c)D.P.(n,n+1,n+2); folosind proprietatea fundamentală a şirului de rapoarte egale, avem căan = bn+1 = cn+2 = a+b+cn+(n+1)+(n+2) = 20103(n+1) = 670n+1 .nAtuncib = 670,a = 670· , iarc = 670n+2. Cum(n,n+1) = 1şi(n+1,n+2)=n+1 n+11, rezultă că n + 1 este un divizor al lui 670, cel puţin egal cu 2 (deoarece n ≠ 0).Otinem că n ∈ {1,4,9,66,133,334,669}, prin urmare 2010 poate fi descompus înşapte moduri cu respectarea cerinţelor problemei.VI.124. Într-o duminică, bunica face clătite pentru nepoţi; 40% dintre clătitesunt cu gem, iar restul cu ciocolată. În duminica următoare, bunica face cu 10% maimulte clătite cu gem şi cu 5% mai puţine clătite cu ciocolată. În care dintre duminicia făcut bunica mai multe clătite?Doru Turbatu, IaşiSoluţie. Fie n numărul clătitelor făcute de bunică în prima duminică. Dintreacestea, 40% · n = 2n 5 sunt cu gem, iar 3n sunt cu ciocolată. În a doua duminică,5avem 2n 5 + 100·2n 5 = 11n25 clătite cu gem şi 3n 5 − 100·3n 55 = 57n clătite cu ciocolată,100în total 101n clătite. Rezultă că în a doua duminică bunica a făcut mai multe clătite100(cu 1% mai mult decât în prima duminică).VI.125. Determinaţi restul împărţirii prin 2010 a numărului A = 1 2011 +2 2011 +···+2011 2011 .Andrei Paşa, elev, IaşiSoluţie. Pentruexponeţiimpari,a n +b n = M(a+b),oricarearfia,b ∈ N. Încazulnostru, 1 2011 +2009 2011 = M 2010 , 2 2011 +2008 2011 = M 2010 ,··· ,1004 2011 +1006 2011 =M 2010 . Evident că 2010 2011 = M 2010 , iar 2011 2011 = (2010 + 1) 2011 = M 2010 + 1.151


Numărul 1005 2011 este impar şi se divide cu 1005, prin urmare 1005 2011 = M 2010 +1005. În concluzie, A = M 2010 +1006.VI.126. Pe tablă sunt scrise numerele 1,2,3,··· ,2010. Andrei şterge de pe tablădouă numere, înlocuindu-le cu media lor aritmetică şi procedează astfel în mod repetat,până când pe tablă rămâne un singur număr. Este posibil ca acest ultim număr să fie2009,25?Gabriel Popa, IaşiSoluţie. Media aritmetică a două numere distincte este strict mai mică decâtnumărul mai mare. Pentru ca în final să rămână pe tablă numărul 2009,25,la ultimulpas trebuie să facă media numerelor 2010 şi 2008,5. Pentru a obţine 2008,5, lapenultimul pas Andrei va face media numerelor 2009 şi 2008. Astfel, în primii 2007paşi, ar trebui ca din numerele 1,2,··· ,2008, să rămână pe tablă 2008, fapt imposibilconform observaţiei iniţiale.VI.127. Considerăm punctele coliniare distincte A,B,C,D şi E astfel încâtAB = b, AC = a, BD = b − a (2a < b < 3a), E este simetricul lui B faţă deD, iar mijlocul segmentului [AC] este punctul E. Aflaţi numerele a şi b, ştiind căBD = 6.Matei Hăvârneanu, elev, IaşiSoluţie. ConsiderândpedreaptaAB sensuldeparcursdinspreAcătreB, primelepatru puncte se pot afla într-una dintre ordinileA−B −C −D, C −A−B −D saua b-aC E A D BC−A−D−B. În primele două situaţii, mijlocul segmentului AC nu poate coincidecu simetricul lui B faţă de D. Rămâne de studiat doar cazul C −A−D −B.Cum E este mijlocul lui [AC], obţinem că AE = a . Însă D este mijlocul lui [BE],2prin urmare a+ a 2 = b−a, deci b = 5a . Condiţia BD = 6 conduce la a = 4, b = 10.2VI.128. Se consideră triunghiul isoscel ABC cu m(A) = 120 ◦ . Perpendiculara înA pe AC intersectează bisectoarea unghiuluiC în F şi latura BC în E. Paralela prinE la AB taie CF în M şi notăm {P} = AM ∩BC. Determinaţi măsura unghiuluiAPB.Gabriela Popa, IaşiSoluţie. Observăm că AB = AC, m(B) = m(C) = 30 ◦ , iar m(BAE) = 120 ◦ −90 ◦ = 30 ◦ . Cum AB‖EM, deducem căm(AEM) = 30 ◦ . Însă m(AEC) = 180 ◦ − 90 ◦ −30 ◦ = 60 ◦ , prin urmare EM este bisectoareaunghiuluiAEC. Rezultă că M este centrul cerculuiînscris în △AEC, deci m(EAP) = 1 2 ·90◦ =45 ◦ . Din triunghiul AEP obţinem că m(APB) =180 ◦ −60 ◦ −45 ◦ = 75 ◦ .VI.129. Se consideră triunghiul ABC. Determinaţi un punct M pe latura [AC],aflat la egală distanţă de vârful A şi de dreapta BC.Mihaela Cianga, Iaşi152Ba2a2EFAPMC


Soluţie. Fie O punctul de intersecţie dintreBC şi perpendiculara în A pe dreapta AC.Din ipoteza problemei, rezultă că M este egaldepărtat de dreptele OA şi OC, prin urmare seaflă situat la intersecţia dintre AC şi bisectoareaunghiuluiAOC.Clasa a VII-aVII.123. Determinaţi numerele abc, scrise în baza 10, pentru care numărul A = abc−Ôabc−28 este natural.Vasile Chiriac, BacăuSoluţia 1. Observăm căÔabc−28 = abc−A 2 ∈ N, deci numărul abc−28 estepătrat perfect. Rezultă că abc = k 2 +28, unde k ∈ {9,10,··· ,31}. Verificând cele 22de cazuri posibile, obţinem că A este număr natural doar când abc ∈ {109,812}.Soluţia 2. Mai general, determinăm numerele naturale x pentru care A = x− √ x−28este număr natural. Vom aveax = k 2 +28, k ∈ N, iark 2 −k+28 = A 2 ,adică (2k −1) 2 −4A 2 = −111. Deducem că (2k −2A−1)(2k +2A−1) = −111 şi,studiind cazurile posibile, regăsim exact soluţiile de mai sus. Remarcăm astfel faptulcă nu este esenţial ca numărul natural x să fie de trei cifre.VII.124. Demonstraţi că nu există numere naturale n pentru care numărul A =√ n+√ n+2010 să fie pătrat perfect.Neculai Stanciu, BuzăuSoluţie. Dacă n,m sunt numere naturale astfel încât √ n+ √ m ∈ Q ∗ , atunci nşi m sunt pătrate perfecte. Într-adevăr, am obţine că √ n − √ m = n−m √ n+√ m∈ Q,deci √ n = 1 2 [(√ n+ √ m)+( √ n− √ m)] ∈ Q şi √ m = ( √ n+ √ m)− √ n ∈ Q. Cumn,m ∈ N, deducem că √ n, √ m ∈ N, prin urmare n şi m sunt pătrate perfecte.Săpresupunem căarexistan ∈ NpentrucareA ∈ N; atuncin = a 2 ,n+2010= b 2 ,cu a,b ∈ N. Rezultă că b 2 −a 2 = 2010, deci (b−a)(b+a) = 2010. Cum A = a+beste pătrat perfect, ar trebui ca a+b = 1, b−a = 2010, situaţie care nu convine. Înconcluzie, nu există n ∈ N pentru care A să fie pătrat perfect.VII.125. Demonstraţi că numărul A = 2009·2011 2010 −2010·2011 2009 +1 estedivizibil cu 2010 2 .Tamara Culac, IaşiSoluţie. Întrucât (a + b) n = ma 2 + nM a + b n , avem că 2011 2010 = M 2010 2 +2010 · M 2010 + 1 = M 2010 2 + 1, iar 2011 2009 = M 2010 2 + 2009 · M 2010 + 1. AtunciA = 2009(M 2010 2 + 1) − 2010·(M 2010 2 + 2009·M 2010 + 1) + 1 = M 2010 2 + 2009−M 2010 2 −2010+1 = M 2010 2.VII.126. Dacă a,b,c sunt lungimile laturilor unui triunghi, iar p este semiperimetrulacestuia, demonstraţi căap−a + bp−b + cp−c ≥ 6.153OBAIonel Nechifor, IaşiMC


aSoluţie. Avem:p−a + bp−b + c =p −1+p −1+pp−c p−a p−b p−c −1=p1p−a + 1p−b + p−c−3 1 = ((p−a)+(p−b)+(p−c))1p−a + 1p−b + p−c−13 ≥ 9−3 = 6, cu egalitate în cazul triunghiului echilateral.Notă. S-au primit alte trei soluţii ale acestei probleme din partea d-lui TituZvonaru, Comăneşti.VII.127. Se consideră triunghiul ABC, punctulAD pe latura (BC) şi romburile BDEF şi CDGH cuE,G ∈ (AD), astfel încât A,F şi H sunt de aceeaşi parte FEa dreptei BC, iar AD separă F şi H. Demonstraţi căG Hdreptele AD,BH şi CF sunt concurente..Dan Popescu, SuceavaOSoluţie. Fie {O} = BH∩CF; cum BF‖CH, rezultăBD Ccă HOOB = HCBF = CD . Cu reciproca teoremei lui Thales în △BCH, obţinem căDBDO‖CH şi atunci O ∈ AD, deci dreptele AD,BH şi CF sunt concurente.VII.128. Pe latura (AB) a triunghiului ascuţitunghic ABC se consideră punctulM, iar pe segmentul (CM) punctul N, astfel încât △BNM ∼ △CAM. Demonstraţică AN ⊥ BC.Constantin Apostol, Rm. SăratSoluţie. Avem căN<strong>MB</strong> ≡CMA şi, cum cele Adouă unghiuri sunt suplementare, ele vor fi drepte, deciCM ⊥ AB. Fie {P} = BN ∩ AC; deoareceNBM ≡PACM şi m(ACM)+m(A) = 90 ◦ , rezultă că m(ABP)+m(A) = 90 ◦ , prin urmare BP ⊥ AC. Astfel, N se află Mla intersecţia a două înălţimi ale triunghiului, adică este Nortocentrul acestuia. Deducem că AN este tot înălţime,deci AN ⊥ BC.BCVII.129. Se consideră triunghiul ABC cu BC > AC > AB şi punctele D,E pelatura (BC) şi F pe (AC), astfel încât AB = BD = AF, iar AC = CE. DreaptaBF intersectează AD şi AE în G, respectiv H. Arătaţi că punctele D,E,G şi H suntconciclice.Daniela Brumă, Deleni (Iaşi)Soluţie. Observăm că, pe BC, ordinea punctelor este B − E − D − C,deoarece altfel ar rezulta că BC > AB + AC, imposibil.Din triunghiurile isoscele ABF şi EAC, obţinemAcăm(ABF) = 90 ◦ − 1 2 m(A), iar m(EAC) = 90 ◦ − 1 2 m(C),deci m(BAH) = m(A) + 1 2 m(C) − 90 ◦ . Deducem căm(AHB) = 180 ◦ −m(ABF)−m(BAH) = 180 ◦ −m(A),prin urmare m(EHG) = 180 ◦ − m(A) = 180 ◦ − m(D)(datfiindfaptulcătriunghiulBAD esteisoscel). Rezultăastfelcăm(EHG)+m(D) =154BH. .EG..D.FC


180 ◦ , de unde concluzia problemei.Clasa a VIII-aVIII.123. Rezolvaţi în R ecuaţia 2x 2 +2y 2 −2xy +2x−10y+14 = 0.Ionica Marcovschi, PaşcaniSoluţie. Ecuaţia datăse poate scriesub forma(x−y+2) 2 +(x−1) 2 +(y−3) 2 = 0.Cum pătratele numerelorreale sunt nenegative, obţinem că x−y+2 = x−1 = y−3 =0, de unde x = 1, y = 3.VIII.124. Demonstraţi ca numărulA = (24 +2 2 +1)(3 4 +3 2 +1)···(2010 4 +2010 2 +1)3(2010 2 +2010+1)este pătrat perfect.Bianca Maria Filip, elevă, IaşiSoluţie. Folosind descompunerea n 4 +n 2 +1 = (n 2 +1) 2 −n 2 = [n(n−1)+1]·[n(n+1)+1], obţinem că A = (2 2 +2+1) 2 (3 2 +3+1) 2···(20092 +2009+1) 2 , deciA este pătrat perfect.äÔn 2 +3n+9ç=Ôn 2 +3n+9.Ionuţ Stroe, student, IaşiVIII.125. Determinaţi numerele întregi n pentru careSoluţie. Partea întreagă fiind număr întreg, rezultă că n 2 + 3n+9 = k 2 , undek ∈ N. Obţinem că (2n + 3) 2 + 27 = 4k 2 , deci (2k + 2n + 3)(2k − 2n − 3) = 27,cu ambele paranteze numere întregi. Analizând situaţiile posibile, găsim soluţiilen ∈ {−8,−3,0,5}.VIII.126. Fie a,b,c,p ∈ N ∗ cu a,b,c distincte şi √ p ∈ R\Q, iar A = {a √ p,b √ p,c √ p}. Funcţia f : A → N este definită prin f(x) = x 2 +(2c−a−b)x−b−1. Dacăx,y ∈ A, x < y, demonstraţi că f(x) < f(y).Cosmin Manea şi Dragoş Petrică, PiteştiSoluţie. Cum f(a √ p) ∈ N, obţinem că (a 2 p−b−1)+a(2c−a−b) √ p ∈ N, deunde a(2c−a−b) = 0. Ştim că a ≠ 0, deci 2c−a−b= 0 şi atunci f(x) = x 2 −(b+1).Dacă x,y ∈ A, x < y, numerele x şi y vor fi pozitive, prin urmare x 2 < y 2 şi de aicirezultă că f(x) < f(y).VIII.127. Determinaţi numerele reale x,y şi z, ştiind că4(x−1)y 2 z 2 +4(y −1)z 2 x 2 +4(z −1)x 2 y 2 = 3x 2 y 2 z 2 .Lucian Tuţescu şi Mariana Mărculescu, CraiovaSoluţie. Dacă unul dintre numerele x,y sau z este zero, vom avea şi un aldoilea număr nul, prin urmare ecuaţia dată are soluţiile (α,0,0);(0,β,0) şi (0,0,γ),cu α,β,γ ∈ R. Dacă xyz ≠ 0, împărţim prin 4x 2 y 2 z 2 şi obţinem că x−1x 2 +y −1y 2+ z −1z 2= 3 . Fiecare dintre fracţiile din stânga este însă cel mult egală cu4155


14 (x−1 x 2 ≤ 1 4 ⇔ (x − 2)2 ≥ 0, evident adevărat, cu egalitate doar dacă x = 2).Deducem că ecuaţia din enunţ, mai admite, în plus, soluţia (2,2,2).VIII.128. Dacă cercurile înscrise în trei feţe ale unui tetraedru sunt tangentedouă câte două, atunci cercul înscris în a patra faţă este tangent primelor trei.Mihály Bencze, BraşovSoluţie. Considerăm cercurile înscrise triunghiurilor ABC,ACD şi ABDtangente muchiilor în T 1 ,T 2 ,··· ,T 6 . Avem că AT 1 = AAT 2 = AT 3 = a (deoarece sunt tangente duse din A lacercurile înscrise în triunghiurile ABC,ACD). La fel,T 3T 1 . .BT 1 = BT 4 = BT 6 = b; CT 4 = CT 2 = CT 5 = c şiTDT 25 = DT 3 = DT 6 = d. Dacă M,N,P sunt punctele deDtangenţăalecercului înscristriunghiuluiBCD culaturileT 6.B ..BC,CD respectiv BD, cum BM = 1 T 52 P BCD − CD =T 4 C12 (2b+2c+2d)−(c+d) = b = BT 4, rezultă că M = T 4 . La fel, N = T 5 şi P = T 6 .VIII.129. Să se arate că din feţele unui cub de muchie l, confecţionat din carton,putem construi, fără resturi, feţele a şase cuburi de muchii l 1 ,l 2 ,··· ,l 6 astfel încâtl 2 = l1 2 +l2 2 +···+l2 6 .Petru Asaftei, IaşiSoluţie. Împărţim fiecare faţă a cubului dat în şase pătrate de laturi l 1 = 2 3 l şil 2 = l 3 = ··· = l 6 = 1 3 l. Evident, l2 1 +l2 2 +···+l2 6 = 4 9 l2 +5 1 9 l2 = l 2 .Clasa a IX-aIX.111. Let a,b,cbe positive real numbers. Prove that a2 +1 +1 +1b+c +b2 c+a +c2 a+b ≥3.Pedro H.O. Pantoja, BrazilxSoluţia 1. Conform inegalităţii lui Nesbitt, avem căy +z + yz +x + zx+y ≥ 3 2 .Luând x = a 2 + 1, y = b 2 + 1, z = c 2 + 1, obţinem căa 2 +1b 2 +c 2 +2 ≥ 3 . Pe de2altă parte, cum b 2 + 1 ≥ 2b, c 2 + 1 ≥ 2c, rezultă că b 2 + c 2 + 2 ≥ 2(b + c), decia 2 +1b+c ≥ 2a2 +1b 2 +c 2 şi de aici urmează inegalitatea cerută. Egalitatea se+2atinge pentru a = b = c = 1.Soluţia 2. AvemÈa 2deciÈa 2 +1b+c ≥ a+b+c +2b+c ≥ (a+b+c)22(a+b+c) = a+b+c292(a+b+c) ≥ 2·Öa+b+c ·2şiÈ1b+c ≥ 992(a+b+c) = 3.2(a+b+c) ,IX.112. Arătaţi că pentru fiecare număr natural n ≥ 2, există numerele naturale1x 1 < x 2 < ··· < x n astfel încât + 1 +···+ 1 = 1x 1 x 2 x n 2010 . Radu Sava, Iaşi156


Soluţie. Vom demonstra afirmaţia prin inducţie matematică. Pentru n = 2, putemconsidera x 1 =2011 şi x 2 = 2010·2011.Presupunem că există x 1 ,x 2 ,··· ,x n ca înenunţ şi să arătăm că există y 1 < y 2 < ··· < y n+1 astfel încât 1 + 1 +···+ 1 =y 1 y 2 y n+112010 . Cum 1 1=x n x n +1 + 1x n (x n +1) , putem lua y 1 = x 1 ,··· ,y n−1 = x n−1 ,y n = x n +1, y n+1 = x n (x n +1) şi toate cerinţele sunt îndeplinite.IX.113. Coardele AB şi CD ale unui cerc de centru O sunt perpendiculare şise intersectează în P. Dacă E şi F sunt mijloacele segmentelor AC, respectiv BD,arătaţi că PE = OF.Petru Asaftei, IaşiSoluţia 1. Dacă R,S sunt mijloacele segmentelor AB, respectiv CD, atunci2 −→ −→ −→ −→ −→ −→ −→PO = 2PR+2PS = ( PA+ AR)+(PB+ BR)+( PC+ D−→CS)+( −−→ PD+ −→ DS) = ( −→ PA+ −→ −→ −−→ −→ −→PB+ PC+ PD)+( AR+ BR)+( −→ CS+ −→ DS) = ( −→ PA+ −→ −→ −−→ −→ −→PC)+( PB+ PD) = 2PE+2PF =2 −→ −→ −→ −→ −→ −→ PE+2PO+2OF. Deducem că 2PE+2OF = 0 , prinOFurmare −→ −→ PE = FO; în particular, PE = OF.SSoluţia 2. Fie {M} = PF ∩ AC. CumAPM ≡FPB ≡FBP ≡ACP, rezultă că m(APM) + PAR Bm(PAM) = 90 ◦ , deci m(AMP) = 90 ◦ . Astfel, FP ⊥ EAC şi, cum OE ⊥ AC, deducem că PF‖OE. Analog se Carată că OF‖PE şi atunci PEOF este paralelogram, de unde concluzia problemei.IX.114. a) Dacă O este punctul de intersecţie a diagonalelor neperpendiculareale patrulaterului convex ABCD, atunci AB‖CD dacă şi numai dacă AO 2 +DO 2 +BC 2 = BO 2 +CO 2 +AD 2 .b) Presupunem că AB‖CD, m(AC,BD) ≠ 90 ◦ şi notăm cu r 1 ,r 2 razele cercurilorînscrise în triunghiurile AOD, respectiv BOC şi cu R 1 ,R 2 razele cercurilor circumscriseacestor triunghiuri. Arătaţi că AD = BC ⇔ r 1 = r 2 ⇔ R 1 = R 2 .Claudiu Ştefan Popa, IaşiSoluţie. a) CumAOD ≡BOC (opuse la vârf), atunci cosAOD = cosBOC, deciAO 2 +DO 2 −AD 2= BO2 +CO 2 −BC 2(∗). Deducem D C2AO·DO 2BO·COcă AB‖CD ⇔ AOOC = BO ⇔ AO · DO = BO · CO ⇐⇒OD (∗)OAO 2 +DO 2 −AD 2 = BO 2 +CO 2 −BC 2 ⇔ AO 2 +DO 2 +BC 2 = BO 2 +CO 2 +AD 2 .b) Cum AB‖CD, are loc egalitatea de la punctul a). ABÎn plus, AO · DO = BO · CO şi S AOD = S BOC (= S). Atunci R 1 = R 2 ⇔AD ·DO·AO= BO·CO·BC ⇔ AD = BC, iar r 1 = r 2 ⇔ S = S ⇔ p 1 =4S 4Sp 1 p 2p 2 (= p) ⇔ p 2 1 = p2 2 ⇔ AO2 + DO 2 +BC 2 +2(AO ·DO +AO · BC + DO · BC) =BO 2 + CO 2 + AD 2 + 2(BO · CO + BO · AD + CO · AD) ⇔ BC(AO + DO) =AD(BO +CO) ⇔ BC(p−AD) = AD(p−BC) ⇔ BC = AD.157


permutare a mulţimii {1,2,··· ,n}. Demonstraţi căax 1 + bx σ(1)···ax n + bx σ(n)≤ax 1 + bx 1···ax n + bx n.Dan Nedeianu, Drobeta Tr. SeverinSoluţie. Eliminând numitorii, avem de demonstrat că(ax 1 x σ(1) +b)(ax 2 x σ(2) +b)···(ax n x σ(n) +b) ≤ (ax 2 1 +b)(ax2 2 +b)···(ax2 n +b).Observăm că (ax 1 x σ(1) + b) 2 ≤ (ax 2 1 + b)(ax2 σ(1)+ b), întrucât această inegalitateeste echivalentă cu ab(x 1 − x σ(1) ) 2 ≥ 0. Scriind şi celelalte inegalităţi similare şiînmulţindu-le membru cu membru, obţinem ceea ce dorim.X.114. Fie z 1 ,z 2 ,z 3 ∈ C ∗ astfel încât |z 1 | = |z 2 | = |z 3 | ̸= 1 şi z 1 −z 2 z 3 , z 2 −z 3 z 1şi z 3 −z 1 z 2 sunt numere reale. Demonstraţi că (z 1 −z 2 )(z 2 −z 3 )(z 3 −z 1 ) = 0.Ionuţ Ivănescu, CraiovaSoluţie. Fie |z 1 | = |z 2 | = |z 3 | = r. Cum z 1 − z 2 z 3 = z 1 − z 2 z 3 şi z 2 − z 1 z 3 =z 2 −z 1 z 3 , prinînmulţiremembrucumembru, obţinem(dupăreduceri)că(1−r)(z 1 z 2 −z 2 z 1 ) = 0. Întrucât r ≠ 1, deducem că z 1z 2 = a ∈ R ∗ . Analog se arată că z 2 z 3 = b ∈R ∗ şi z 3 z 1 = c ∈ R ∗ . Din z 1 z 2 = a, z 1 z 3 = c = c obţinem, prin împărţire membru cu∈ R, cu |α| =|a||c| = 1, aşadar z 2 = ±z 3 . Similar,membru, că z 2= α, unde α = a z 3 cz 2 = ±z 1 şi atunci măcar două dintre cele trei numere vor fi egale, de unde cerinţaproblemei.X.115. Fie ABCD un patrulater convex şi punctele P,Q ∈ AC, R,S ∈ BD astfelîncât PAPC = QAQC = ABCD = RBRD = SB . Dacă M şi N sunt mijloacele segmentelorSDPQ, respectiv RS, demonstraţi că 2MN < PQ+RS.Titu Zvonaru, ComăneştiSoluţie. NotămAC = α,BD = β, AB = k şiputempresupunek > 1(încazulînCDcare k = 1, unul dintre punctele S, P sau Q ynu există). Alegem un reper cartezian în raportQ. .cu care A(0,0), B(a,0), C(b,c), D(d,e), cu b >CS .d. Ţinând cont de formula care dă coordonatele Dpunctului care împarte un segment într-un raport ..PE.Rdat, obţinem că Pbk1+k , ckQ−bk1+k,1−k , 1−k,−ckxABRa+dk1+k , ekSa−dk1+kşi1−k , 1−k, −ek prin urmare2kα 2kβPQ =k 2 , iar RS =−1 k 2 −1 . Coordonatele punctelor M şiN sunt Mbk 2bk 2k 2 −1 − dk2 −ak −122k 2 −1 , ck 2k 2 −1, respectiv Ndk 2 −ak 2 −1 , 2k 2k 2 −1, deci MN 2 =+ck 2k 2 −1 − ek2k 2 −12159=k 4(k 2 −1) 2¢(b−d) 2 +(c−e) 2£+


a 2(k 2 −1) 2 + 2ak2 (b−d)(k 2 −1) 2 . Cum CD2 = (b − d) 2 + (c − e) 2 şi b − d ≤ CD, rezultăk 2că MN 2 ≤(k 2 −1) 2åk2 ·CD 2 +AB 2 1 +2AB ·CDè=k2 k(AB +CD)2AB ·CD), de unde MN ≤k 2 .−1Pentru a încheia rezolvarea, ar fi destul să demonstrăm căk 2(k 2 −1) 2(AB2 + CD 2 +2k(AB +CD)k 2 −12k(AC +BD)k 2 , adică AB + CD < AC + BD. Această inegalitate rezultă imediat−1din inegalitatea triunghiului: dacă {E} = AC∩BD, atunci AB+CD < AE+EB+CE +ED = (AE +EC)+(BE +ED) = AC +BD.Clasa a XI-aXI.111. Demonstraţi că 4(x 3 +y 3 ) ≥ 9xy|x−y|, ∀x,y ∈ [0,∞).Lucian Tuţescu, CraiovaSoluţie. Dacă y = 0, avem de arătat că 4x 3 ≥ 0, adevărat. Fie y > 0; dacăx = y, inegalitatea este evidentă, iar când x ≠ y putem presupune că x > y (datorităsimetriei). Împărţind prin y3 , cu notaţia t = x > 1, avem de arătat că f(t) ≥ 0, undeyf : [1,∞) → R, f(t) = 4t 3 −9t 2 +9t+4. Avem că f ′ (t) = 3(4t 2 −6t+3) > 0, ∀t ∈ Rşi, cum f(1) = 8 > 0, rezultă că f(t) > 0, ∀t ∈ [1,∞). Egalitatea în inegalitatea dinenunţ se atinge când x = y = 0.1XI.112. Fie şirurile (w n ) n≥1 şi (p n ) n≥1 , unde w n =å(2n)!!·(2n−1)!!è22n+1 , ∀n ∈N ∗ (şirul luiWallis), iar p n este al n-leanumăr prim. Calculaţi limn→∞2w np nlnä1+2w nπ −1ç=π pn 0, ∀n ∈ N ∗ . Apoi, (1+x) α > 1+αx, ∀x ∈ (0,∞) (întrucâtfuncţia f : [0,∞) → R, f(x) = (1+x) α −αx−1 are derivata pozitivă şi f(0) = 0).Rezultă că 1+αx n = (1+x n+1 ) α > 1+αx n+1 , de unde x n+1 < x n , ∀n ∈ N ∗ . Astfel,există lim x n = l ∈ [0,∞) şi (1+l) α = 1+αl. Însăn→∞ (1+l)α > 1+αl, dacă l ∈ (0,∞),după cum am demonstrat anterior şi rămâne că l = 0.Dacă α ∈ (0,1), se arată similar că (1+x) α < 1+αx, ∀x ∈ (0,∞). Deducem căşirul este strict crescător şi, dacă ar fi mărginit superior, ar avea o limită L ∈ (0,∞)160


astfel încât(1+L) α = 1+αL, ceea cear intra încontradicţie cu inegalitateaanunţată.Rămâne că şirul este nemărginit superior, deci limn→∞ x n = +∞.XI.114. Fiind date a,b,α,β ∈ R, determinaţi funcţiile f : R → R cu proprietatea:¬x−α y −α∀x,y ∈ R.f(x)−β f(y)−β¬=af(x)+bf(y),Marius Tiba, elev, BucureştiSoluţie. Observăm că schimbând x şi y între ele, semnul determinantului seschimbă, ceea ce conduce la af(x) + bf(y) = −af(y) − bf(x), ∀x,y ∈ R, adică(a+b)(f(x)+f(y)) = 0, ∀x,y ∈ R.Dacă a + b ≠ 0, atunci f(x) + f(y) = 0, ∀x,y ∈ R. Luând y = 0, obţinem căf(x) = c, ∀x ∈ R (unde c = −f(0)). O astfel de funcţie verifică egalitatea din enunţdacă şi numai dacă c = β = 0. Atunci, dacă β ≠ 0, ecuaţia dată nu are soluţii, iardacă β = 0, singura soluţie este funcţia nulă.Dacă a + b = 0, definim g : R → R prin g(x) = f(x) − β, ∀x ∈ R. Ecuaţiay −αfuncţională din enunţ devine¬x−a− g(y)], ∀x,y ∈ R, de undeg(x) g(y)¬=a[g(x)xg(y)−yg(x) = (a−α)[g(x) −g(y)], ∀x,y ∈ R. Dacă a = α, pentru y = 1 obţinemg(x) = xg(1), ∀x ∈ R, deci f(x) = mx+β, cu m ∈ R. Dacă a ≠ α, luăm y = 0 şiobţinem că g(x) = m(x+a−α), ∀x ∈ R, de unde f(x) = m(x+a−α)+β, ∀x ∈ R.Se constată uşor că funcţiile afine obţinute în acest caz verifică ecuaţia din enunţ.Problema 26043 din G.M. 9/2008 se obţine din aceasta, în cazul particular α =β = 0, a = −b.XI.115. Determinaţi matricele X,Y,Z ∈ M 2 (Z), având determinantul 1, ştiindcă X 4 +Y 4 +Z 4 = X 2 +Y 2 +Z 2 +6I 2 .Florin Stănescu, GăeştiSoluţie. Dacă notăm cu x urma matricei X, cum detX = 1, rezultă că X 2 =xX−I 2 . De aici, X 3 = xX−X = (x 2 −1)X−xI 2 , iar X 4 = (x 2 −1)X 2 −xX = (x 3 −2x)X−(x 2 −1)I 2 . Trecândlaurmă,obţinemcăTrX 2 = x 2 −2şiTr(X 4 ) = x 4 −4x 2 +2şi relaţii analoage au loc pentru matricele Y şi Z. Trecem acum la urmă în egalitateadinenunţ; rezultăcăx 4 −4x 2 +2+y 4 −4y 2 +2+z 4 −4z 2 +2 = x 2 −2+y 2 −2+z 2 −2+12,de unde (x 2 −1)(x 2 −4)+(y 2 −1)(y 2 −4)+(z 2 −1)(z 2 −4) = 12 (∗). Cum x,y,zsunt numere întregi, produsele (x 2 −1)(x 2 −4), (y 2 −1)(y 2 −4) şi (z 2 −1)(z 2 −4)sunt sigur nenegative. Dacă, de exemplu, |x| ≥ 3, atunci (x 2 −1)(x 2 −4) ≥ 40 şi seajunge la o contradicţie. Rezultă că |x|,|y|,|z| ∈ {0,1,2}; pentru a putea avea loc(*), va trebui ca |x| = |y| = |z| = 0, prin urmare X 2 = Y 2 = Z 2 = −I 2 . În concluzie,a bX,Y şi Z vor fi matrice arbitrare de forma−1−a 2 cu a,b ∈ Z şi a−a, 2 +1bdivizibil cu b.Clasa a XII-aXII.111. Fie x,y,z numere reale nenule. Dacă x+y +z = 0 şi x 5 +y 5 +z 5 =x 7 +y 7 +z 7 , calculaţi valoarea expresiei A = x 2 +y 2 +z 2 .Mihai Crăciun, Paşcani161


Soluţie. Fie xy + yz + zx = a, xyz = b; atunci x,y,z sunt soluţiile ecuaţieit 3 +at−b = 0. Notând S n = x n +y n +z n , n ∈ N, obţinem că S n+3 = −aS n+1 +bS n .Cum S 0 = 3, S 1 = 0, S 2 = −2a, deducem: S 3 = 3b; S 4 = 2a 2 ; S 5 = −5ab; S 7 = 7a 2 b.Din condiţia S 5 = S 7 , rezultă că ab(7a+5) = 0. Cum ab ≠ 0, obţinem că a = − 5 7 şiA = S 2 = 107 .XII.112. Calculaţi limita şirului (a n ) n∈N definit prin a n =1∀n ∈ N.0Ôx n +x n+2 dx,Bogdan Victor Grigoriu, FălticeniSoluţia 1. Vom aplica inegalitatea CBS, forma integrală:a n =10√xn ·Ô1+x 2 dx ≤10Cum evident a n > 0, ∀n ∈ N, rezultă că limn→∞ a n = 0.10x n dx·1(1+x 2 )dx =023(n+1) .Soluţia 2 (Moubinool Omarjee). Observăm că 0 < a n =1x n 2Ô1+x 2 dx ≤x n 2 √ 22 ·√2dx = , ∀n ∈ N, prin urmare limn+2 a n = 0.n→∞XII.113. Calculaţi limx→∞1x1x+1ctgt 2 dt.Soluţie. Din formula de medie,1xx 21x+1t 2 xsin 2 ·cos 2 1t x ·t x (xt x ) 2. Cumx+1 < t x < 1 xobţinem că limita cerută este egală cu 1.Silviu Boga, Iaşictgt 2 dt =1x − x+1ctgt 1 2 x = 1x(x+1) ·0, atunci limx→∞ t x = 0 şi limx→∞ (xt x) = 1;XII.114. Se consideră funcţia continuă f : [0,1] → (0,1]. Demonstraţi că pentrunorice n ∈ N ∗ , există un unic x n ∈ (0,1) astfel încâtxf(t)dt = n1f(t)dt.x nCalculaţi limita şirului (x n ) şi arătaţi că lim n(1−x n) = 1 f(t)dt.n→∞ f(1)10Florin Stănescu, GăeştiNotă. Problema a apărut şi în G.M. 6/2010, cu numărul C.O: 5130. Soluţia sapoate fi găsită în G.M. 1/2011.XII.115. Fie F mulţimea funcţiilor de două ori derivabile pe [a,b] cu derivatade ordin doi continuă, pentru care f(a) = α şi f(b) = β, unde α şi β sunt constantefixate. Notăm I(f) =b[f ′ (x)] 2 dx, ∀f ∈ F şi J(f) =b[f ′ (x)(1 + 2f(x))] 2 dx,a∀f ∈ F. Determinaţi min{I(f)|f ∈ F} şi min{J(f)|f ∈ F}.Adrian Corduneanu, Iaşi1620a


Soluţie. Fie f 0 ∈ F astfel încât I(f) ≥ I(f 0 ), ∀f ∈ F. Orice f ∈ F poatefi scrisă sub forma f = f 0 + εϕ, unde ϕ ∈ C 2 [a,b] şi ϕ(a) = ϕ(b) = 0, iar ε este oconstantăarbitrară. Rezultă căI(f) = I(f 0 +εϕ) =b[(f 0 +εϕ) ′ ] 2 dx =b[f 0 ′ (x)]2 +2εbf 0 ′ 2b(x)ϕ′ (x)dx+ε [ϕ ′ (x)] 2 dx, expresie al cărei minim se atinge pentru ε = 0aa(întrucât I(f) ≥ I(f 0 )). Rezultă căbf 0 ′ (x)ϕ′ (x)dx = 0, de undebaaaaf ′′0 (x)ϕ(x)dx =0, ∀ϕ ∈ C 2 [a,b] cu ϕ(a) = ϕ(b) = 0. De aici, f 0 ′′(x)= 0, ∀x ∈ [a,b], deci f 0(x) =λx+µ, cu λ,µ constante care se determină din condiţiile în capete. În final, găsimcă f 0 este unic determinat: f 0 (x) = βb−a (x−a)+ α (b−x). Valoarea minimă ab−a(β −α)2lui I(f) este I(f 0 ) =b−a .Pentru a doua parte, observăm că f ′ (1+2f) = (f +f 2 ) ′ , ∀f ∈ F. Considerândfuncţia g = f +f ′ , cu g(a) = α+α 2 , g(b) = β+β 2 , avem de găsit minimul integraleib[g ′ (x)] 2 dx. Ţinând seama de a), valoarea minimă a lui J(f) este (g(b)−g(a))2 =ab−a(β −α) 2 (1+α+β) 2.b−aParabolaUn preasfânt părinte spune discipolului său:– Fiule, să nu uiţi niciodată: y = ax 2 +bx+c!– Da, preasfinte, o să ţin minte...Peste ani de zile, credinciosul îl întreabă pe mentorul său:– Mă chinuie o întrebare, părinte, ce este y = ax 2 +bx+c?La care preasfântul părinte îi răspunde:– Fiule, asta e o parabolă !163


Soluţiile problemelor propuse în nr. 1/2011Clasele primareP.206. Dan a scris în ordine descrescătoare numerele de la 75 la 23. Calculaţidiferenţa dintre al zecelea şi al 32-lea număr.(Clasa I)Andreea Bîzdîgă, elevă, IaşiSoluţie. Al zecelea număr este 75−10+1 = 66, iar al 32-lea este 75−32+1 = 44.Diferenţa celor două numere este 22.P.207. Fiecărei forme geometrice îi corespunde unpreţ de cost: □ →10 lei, → 20 lei, △ → 30 lei, ○ → 20 lei. Cât costă confecţionareacăsuţei?(Clasa I)Mariana Nastasia, elevă, IaşiSoluţie. Căsuţa costă 10 lei + 10 lei + 10 lei +20 lei +30 lei +20lei = 110 lei.P.208. Pe trei rafturi sunt 75 cărţi. Dacă pe primul raft punem jumătate dincărţile de pe cel de-al doilea, atunci vom avea pe rafturi numere consecutive de cărţi.Câte cărţi erau la început pe fiecare raft?(Clasa I)Iulia Sticea, elevă, IaşiSoluţie. La început, pe primul raft erau a cărţi, pe al doilea b+b cărţi, iar peal treilea c cărţi. În final, pe primul raft sunt a+b cărţi, pe al doilea b cărţi, iar peal treilea c cărţi. Deoarece a+b > b, inseamnă că pe primul raft vor fi 26 cărţi, peal doilea 25 cărţi, iar pe al treilea 24 cărţi. La început, pe cele trei rafturi erau 1,50,respectiv 24 de cărţi.P.209. Numărul lalelelor dintr-o vază este cu 7 mai mare decât numărul trandafirilor,care reprezintă jumătate din numărul lalelelor. Câte flori sunt în vază?(Clasa a II-a)Inst. Maria Racu, IaşiSoluţie. Numărul florilor din vază este (7+7)+7 = 14+7 = 21.P.210. Ce zi nu poate fi astăzi, dacă alaltăieri nu a fost luni şi poimâine nu estesâmbătă?(Clasa a III-a)Mihaela Gâlcă, elevă, IaşiSoluţie. Astăzi nu poate fi nici miercuri, nici joi.P.211. Suma a două numere este un număr de două cifre al căror produs este 5.Care sunt cele două numere, dacă diferenţa lor este 7?(Clasa a III-a)Ana Cojocariu, IaşiSoluţie. Suma celor două numere poate fi 15 sau 51. Deoarece diferenţa lor este7, atunci numerele sunt 4 şi 11 sau 22 şi 29.P.212. Un elev a greşit la adunarea a două numere: a scris cifra zero la sfârşitulprimului număr în loc s-o scrie la sfârşitul celui de-al doilea şi astfel a obţinut suma98 şi nu 89 - suma corectă. Aflaţi cele două numere.(Clasa a III-a)Cristina Timofte, IaşiSoluţie. Dacă numerele sunt a şi b, avem a+b = 89, iar 10a+b : 10 = 98 saua+b = 89 şi 100a+b = 980, de unde 99a+89 = 980. Găsim a = 9, b = 80.164


P.213. O vilă turistică are apartamente cu 3 şi 4 camere. Ştiind că în uşa deintrare a fiecărui apartement se află câte 2 chei, iar numărul camerelor şi al cheilorla un loc este 39, aflaţi câte apartamente cu 3 camere sunt în vilă.(Clasa a IV-a)Dorel Luchian, IaşiSoluţie. Notăm cu x numărul apartamentelor cu 3 camere şi cu y numărul celorcu 4 camere. Avem (3x + 4y) + (2x + 2y) = 39 ⇔ 5x + 6y = 39, cu soluţia unicăx = 3, y = 4.P.214. La o masă rotundă stau cinci copii, fiecare având câte un jeton pe careeste scris un număr. Toţi copiii afirmă că vecinii lor au jetoane cu numere de parităţidiferite. Arătaţi că măcar un copil nu spune adevărul.(Clasa a IV-a)Iuliana Moldovan, studentă, IaşiSoluţie. Fie A,B,C,D şi E cei cinci copii şi să presupunem că niciunul dintre einu minte. Spunem că A este par dacă are un jeton cu număr par şi spunem că A esteimpar în caz contrar.Dacă A este par, avem succesiv că C este impar, E par, B impar, D par, A impar– contradicţie. Analog ajungem la contradicţie dacă A este impar. În concluzie, celpuţin un copil minte.P.215. Într-un rucsac sunt 12 şosete care pot forma şase perechi de culori diferite,iar două dintre ele sunt rupte. Câte şosete trebuie scoase la întâmplare din rucsacpentru a avea o pereche bună?(Clasa a IV-a)Petru Asaftei, IaşiSoluţie. Avem de analizat două cazuri. Primul, când cele două şosete rupte auaceeaşi culoare. Cea mai dezavantajoasă situaţie este să scoatem 7 şoşete, dintre care5 şosete bune de culori diferite şi o pereche de aceeaşi culoare, însă rupte. Dacă maiextragem o şosetă, atunci putem forma o pereche de şosete bune.Al doilea, când cele două şosete rupte au culori diferite. În această situaţie, celmai dezavantajos este să extragem 8 şosete, 6 şosete bune de culori diferite şi douăşosete rupte, tot de culori diferite. Dacă mai extragem o şosetă, aceasta este bunăşi putem forma o pereche de şosete bune. În concluzie, pentru a fi siguri că avem opereche de şosete bune, trebuie să extragem 9 şosete.Clasa a V-aV.130. Determinaţi numărul a0bb (3) , dacă a0bb (3) = bba (7) .Nicolae Ivăşchescu, CraiovaSoluţie. Trecând în baza 10, egalitatea din enunţ devine 27a+4b = 56b+a, adică26a = 52b, prin urmare a = 2b. Cum a şi b sunt cifre în baza 3, a ≠ 0, convine doarsituaţia a = 2, b = 1. În concluzie, a0bb (3) = 2011 (3) .V.131. Se consideră numărul N = 1+7+13+19+a+b+37, unde termeniisumei sunt scrişi în ordine strict crescătoare. Determinaţi numărul perechilor (a,b)pentru care N este pătrat perfect.Anca Chiriţescu, Ţigănaşi (Iaşi)Soluţie. Observăm că N min = 1+7+13+19+20+21+37 = 118, iar N max =1 + 7 + 13+19 + 35+36 + 37 = 148, deci N este pătrat perfect dacă este egal cu121 sau cu 144. Rezultă a + b poate lua valorile 44 sau 67, prin urmare (a,b) ∈165


{(20,24);(21,23);(31,36);(32,35);(33,34)}. În concluzie, există cinci perechi (a,b)pentru care N este pătrat perfect.V.132. Numerele 0,1,...,2011 sunt aranjate într-un tablou astfel:0 7 8 15 ... 20081 6 9 14 ... 20092 5 10 13 ... 20103 4 11 12 ... 2011.a) Stabiliţi care sunt elementele coloanei 211 (de sus în jos).b) Calculaţi suma elementelor de pe a treia linie.Ioana Maria Popa, elevă, IaşiSoluţie. a) Coloana n conţine numerele 4n−4,4n−3,4n−2 şi 4n−1, în ordinecrescătoare dacă n este impar şi în ordine descrescătoare dacă n este par. Atuncielementele coloanei 211 sunt (de sus în jos): 840,841,842,843.b) Suma elementelor de pe a treia linie este S = S 1 + S 2 , unde S 1 = 2 + 10 +18 + ... + 2010 = (2+2010)·252 = 253512, iar S 2 = 5 + 13 + 21 + ... + 2005 =ßÞÐ2011de9ßÞÐ2011de92(5+2005)·251= 252255. În concluzie, S = 505767.2V.133. Calculaţi sumele:ßÞÐa) S 1 = 85+985+9985+...+ 99...985;b) S 2ßÞÐ= 17+197+1997+...+199...97.Eugeniu Blăjuţ, BacăuSoluţie. a) Observăm că S 1ßÞÐ ßÞÐ= (100−15)+(1000−15)+...+(1 00...0 −15) =2013 de 011...1100−15·2012 = 11...11080920.2012de12008 de 1b) Cum S 1 = 5S 2 , atunci S 2 = S 1 : 5 = 22...2216184.2008 de 2V.134. Lucian-Georges rămâne nesupravegheat la calculator şi, din neatenţie,printează toate numerele naturale de 1 la 1000000. Drept pedeapsă, el trebuie sănumere de câte ori a fost tipărită cifra 5. Care este răspunsul corect pe care trebuiesă-l dea copilul?Andrei Nedelcu şi Cătălin Budeanu, IaşiSoluţie. Numărul apariţiilor cifrei 5 nu se modifică dacă în loc de numerele1...1000000 considerăm numerele 0...999999, iar fiecare număr se scrie cu ajutorula şase cifre, adăugând zerouri în stânga sa (de exemplu, în loc de 2011 vom scrie002011). În acest fel, numărul total de cifre va fi 6×106 şi fiecare cifră va avea acelaşinumăr de apariţii, deci cifra 5 va fi folosită de 6·10 6 : 10 = 600000 ori.V.135. Demonstraţi că 2 122 < 10 37 .166Titu Zvonaru, Comăneşti


Soluţie. Cum 2 10 = 1024 < 1029 = 3 · 343 = 3 · 7 3 , atunci 2 20 < 9 · 7 6 =9·(7 2 ) 3 < 9·50 3 = 9·2 3·5 6 , deci 2 17 < 9·5 6 . Deducem că 2 23 < 9·10 6 , prin urmare2 69 < 729·10 18 < 3·2 8·10 18 , de unde 2 61 < 3·10 18 . Obţinem că 2 122 < 9·10 36 < 10 37 .Notă (T. Zvonaru). Pe parcurs, am obţinut că 2 23 < 9 · 10 6 , deci 2 17 < 10 7 .Această inegalitate poate fi folosită pentru a demonstra că 2 100 < 2 61 , fără a calcula2 33 (a se vedea soluţia problemei V.111. din Rec.Mat. 2/2010). Într-adevăr, 223 102 , rezultă că 2 200 < 10 61 .V.136. Se consideră şirul de numere naturale 4,19,163,1945,.... Determinaţiultimele 501 cifre ale celui de-al 2011-lea termen al şirului.Mihai Crăciun, PaşcaniAvem: a 1 = 4 = 3 1 · 1! + 1, a 2 = 19 = 3 2 · 2! + 1, a 3 = 163 = 3 3 · 3! + 1,a 4 = 1945 = 3 4·4!+1. Atunci a 2011 = 3 2011·2011!+1. Numărul de zerouri cu care setermină scrierea zecimală a numărului 2011! esteå20115è+å20112è+å20115 5 3è+... =402+80+16+3+0+... = 501, deci ultimele 501 cifre ale lui a 2011 sunt 00...001.Clasa a VI-aVI.130. Fie A 1 ,A 2 ,...,A 2011 puncte echidistante pe o dreaptă d. Notăm cuB 1 ,B 2 ,...,B 2010 mijloacele segmentelor A 1 A 2 ,A 2 A 3 ,...,A 2010 A 2011 ; apoi, fie C 1 ,C 2 ,...,C 2009 mijloacele segmentelor B 1 B 2 ,B 2 B 3 ,...,B 2009 B 2010 ş.a.m.d., până cândobţinem un singur punct M. Determinaţi poziţia punctului M.Elena Iurea, IaşiSoluţie. La fiecare pas, numărul punctelor scade cu 1. În etapa 1, avem puncteleA 1 ,A 2 ,...,A 2011 . Dupăunpas, înetapa2, avempuncteleB 1 ,B 2 ,...,B 2010 . Înetapa3, punctele sunt C 1 = A 2 , C 2 = A 3 ,...,C 2009 = A 2010 şi în toate etapele cu numărde ordine impar vom obţine puncte dintre cele iniţiale: A 3 ,A 4 ,...,A 2009 în etapa 5;A 4 ,...,A 2008 în etapa 7; ...;A 1005 ,A 1006 ,A 1007 în etapa 2009; A 1006 în etapa 2011.În concluzie, M = A 1006 .VI.131. Se consideră △ABC cu AC = BC şipunctele M,N cu B ∈ [AM], N ∈ [AC]. Arătaţi căMA = MN dacă şi numai dacă<strong>MB</strong>C ≡CNM.Petru Asaftei, IaşiSoluţie. Folosind faptul căCAB ≡CBA, avem:MA = MN ⇔MAN ≡MNA ⇔CBA ≡MNA ⇔<strong>MB</strong>C ≡CNM.NCA B MVI.132. Se consideră triunghiurile isoscele ABC şi ABD cu AB = AC = AD,m(BAC) = 28 ◦ , m(BAD) = 32 ◦ , punctele C şi D fiind de o parte şi de alta a drepteiAB. Dacă E este mijlocul segmentului AC şi {M} = DE∩BC, determinaţi măsurileunghiurilor triunghiului MAB.Eugeniu Blăjuţ, BacăuSoluţie. Triunghiul ACD are AC = AD şi m(CAD) = 28 ◦ + 32 ◦ = 60 ◦ ,deci este echilateral; cum DE este mediană, rezultă că DE ⊥ AC. Atunci ME167


este mediatoarea segmentului AC şi triunghiul MAC va fi isoscel, cu m(MAC) ≡m(MCA) = 1 2 (180◦ − 28 ◦ ) = 76 ◦ şi m(AMC) = 180 ◦ − 2 · 76 ◦ = 28 ◦ . Apoi,m(MAB) = 76 ◦ −28 ◦ = 48 ◦ , iar m(ABM) = 180 ◦ −28 ◦ −48 ◦ = 104 ◦ .VI.133. Măsura unui unghi este u = ab ′ ab ′′ , unde 0 < ab < 60, iar numărulnatural nenul n are proprietatea că n·u exprimă un număr întreg de grade. Aflaţi upentru care n este minim.Marian Panţiruc, IaşiSoluţie. Observăm că u =ab60 3600◦+ ab =61·ab. Cum n · u ∈ N3600◦∗ ,0 < ab < 60, 61 este relativ prim cu 3600 iar n este minim, rezultă că ab trebuie săfie multiplu al celui mai mare divizor al lui 3600 mai mic decât 60. Astfel, ab = 45,caz în care n = 80.VI.134. Arătaţi că numărul A = 2010 2010 +2012 2012 −2 este divizibil cu 2011.Daniela Munteanu, IaşiSoluţie. Cum 2010 2010 = (2011 − 1) 2010 = M 2011 + (−1) 2010 = M 2011 + 1 şi2012 2012 = (2011+1) 2012 = M 2011 +1, rezultă că A = (M 2011 +1)+(M 2011 +1)−2 =M 2011 .VI.135. Determinaţi câte fracţii ireductibile şi subunitare au proprietatea că sumadintre numărător şi numitor este 1005.Mirela Marin, IaşiaSoluţie. Fracţiile căutate sunt de forma , cu a < 1005−a, adică a ∈1005−a{1,2,...,502}. O astfel de fracţie este reductibilă prin d dacă d|a şi d|1005−a, decicând d|1005, adică d ∈ {3,5,67,3·5,3·67,5·67}. Numărul fracţiilor reductibile este å502238,3è+å5025è+å50267è−å50215è−å502201è−å502335è=167+100+7−33−2−1=prin urmare numărul de fracţii ireductibile va fi 502−238 = 264.VI.136. După ce fiecare echipă a jucat cu fiecare dintre celelalte câte un meci,clasamentul grupei A de la Campionatul mondial de fotbal 2010 arăta astfel:1. Uruguay 3 2 1 0 4-0 72. Mexic 3 1 1 1 3-2 43. Africa de Sud 3 1 1 1 3-5 44. Franţa 3 0 1 2 1-4 1Ştiindcăîn meciul Uruguay-Franţanu s-amarcat niciun gol, aflaţi rezultatele fiecăruiadintre cele şase meciuri.Titu Zvonaru, ComăneştiSoluţie. Deoarece rezultatul meciului Uruguay-Franţa a fost 0 − 0, scorurileposibile pentru fiecare echipă sunt:168


Uruguay: (0-0,1-0,3-0) sau (0-0,2-0,2-0)Franţa: (0-0,1-2,0-2) sau (0-0,1-3,0-1)Mexic: (0-0,2-0,1-2) sau (0-0,3-0,0-2) sau (0-0,3-1,0-1)sau (1-1,2-0,0-1)Africa de Sud: (0-0,1-0,2-5) sau (0-0,2-0,1-5) sau (0-0,3-0,0-5)sau (0-0,2-1,1-4) sau (0-0,3-1,0-4) sau (0-0,3-2,0-3)sau (1-1,1-0,1-4) sau (1-1,2-0,0-4) sau (1-1,2-1,0-3).Cum printre scorurile posibile ale echipelor Uruguay, Franţa şi Mexic nu se găsesc5−2,5−1,5−0,4−1,4−0,3−2 singura posibilitate pentru Africa de Sud rămâne(1−1,2−1,0−3). Deoarece meciul egal al Africii de Sud a fost cu Mexicul, scorurileacestei echipe vor fi (1 − 1,2 − 0,0 − 1). Acum obţinem uşor scorurile echipelorUruguay: (0−0,1−0,3−0) şi Franţei: (0−0,1−2,0−2). Rezultatele complete alegrupei au fost:Uruguay-Franţa 0−0; Mexic-Africa de Sud 1−1;Uruguay-Africa de Sud 3−0; Mexic-Franţa 2−0;Uruguay-Mexic 1−0; Africa de Sud-Franţa 2−1.Clasa a VII-aVII.130. Fie C ′ mijlocul laturii AB a triunghiului AABC, iar P un punct pe segmentul CC ′ . Dacă AP ∩BC = {R}, arătaţi că AP ·CR = BC ·PR.Claudiu Ştefan Popa, Iaşi CSoluţie. Cu teoremalui Menelaus întriunghiul ABRPcu transversalaC ′ −P−C, obţinem că AC′ ·BCCR·RPC ′ B PA =BC ·PRB R C1, deci = 1, de unde cerinţa problemei.AP ·CRVII.131. Într-un triunghi ABC, lungimile laturilor verifică relaţia a b = b+ca .Dacă m(A) = α, determinaţi măsurile unghiurilorB şiC.Neculai Stanciu, BuzăuSoluţie. Considerăm punctul D astfel încât A ∈ (CD), AD = AB = c. CumCBBCA = CDCB şi unghiulC este comun, rezultă că△CAB ∼ △CBD, de unde m(ABC) = m(CDB).ÎnsăBAC este unghi exterior triunghiului isoscelABD, prin urmare m(BAC) = 2m(D)a c=2m(CBA). Deducem că m(CBA) = α 2 , iarC b A c Dm(C) = 180 ◦ − 3α 2 . (Evident, se impune ca α < 120◦ , în caz contrar neexistândtriunghiuri ca în enunţ.)VII.132. În cercul C(O,r) se consideră coarda [AB]. Notăm cu C mijlocul arculuimareAB, cu O ′ simetricul punctului O faţă de AB şi fie x = m(ACB). Determinaţivalorile lui x pentru care O ′ ∈ IntC.Geanina Havârneanu, Iaşi169


Soluţie. Avem O ′ ∈ IntC ⇔ OO ′ < r ⇔ d(O,AB) < r 2 = AO 2 ⇔ sin(OAB) 60 ◦ (unde D este mijlocularcului micAB) ⇔ m(AD) > 60 ◦ ⇔ m(AB) > 120 ◦ ⇔ 2x >OAB. .120 ◦ ⇔ x > 60 ◦ . Evident că x ≤ 90 ◦ (deoarece C se află pearcul mareAB), prin urmare valorile căutate ale lui x sunt celedin intervalul (60 ◦ ,90 ◦ ].OVII.133. Se consideră pătratul ABCD cu laturade 3 √ 5cm.DPe semidreptele (AB şi (BC se consideră punctele M, respectivN, astfel încât AM = 7 √ 5cm şi BN = 10 √ 5cm. Determinaţi măsura unghiuluiMND.Constantin Apostol, Rm. SăratSoluţie. Întrucât AM = CN = 7√ 5cm şi AD = CD = 3 √ 5cm, triunghiuriledreptunghice AMD şi CND sunt congruente (C.C.), A Dprin urmare DM = DN şiMDA ≡NDC. Însăm(ADM) + m(MDC) = 90 ◦ , deci m(NDC) + BNm(MDC) = 90 ◦ , adică m(MDN) = 90 ◦ . Deducem că C△DMN este dreptunghic isoscel şi astfel m(MND) =45 ◦ .VII.134. Comparaţi numerele reale a = 2 √ 1+2 √ 2+ M2 √ 3+...+2 √ 64 şi b = ( √ 0+ √ 2)+( √ 1+ √ 3)+( √ 2+ √ 4)+...+( √ 63+ √ 65).Cosmin Manea şi Dragoş Petrică, PiteştiSoluţie. Eliminând termenii comuni ai celor două sume, compararea numerelor aşi b revine la compararea numerelor √ 1+ √ 64 cu √ 65. Însă √ 1+ √ 64 = 9 = √ 81 >√65, prin urmare a > b.VII.135. Rezolvaţi în numere întregi ecuaţia x 3 +y 3 +3xy = 9.Cristina Ene, elevă, CraiovaSoluţie. Cum 3¬x 3 +y 3 , avem două cazuri:I. x = 3a,y = 3b, cu a,b ∈ Z; atunci 27(a 3 +b 3 +ab) = 9, imposibil.II. x = 3a+1,y = 3b−1, cu a,b ∈ Z; atunci x 3 +y 3 = 27a 3 +27a 2 +9a+1+27b 3 −27b 2 +9b−1, deci x 3 +y 3. .9. Rezultă atunci că 3xy . .9, adică xy = (3a+1)(3b−1) . .3,ceea ce este imposibil. În concluzie, ecuaţia dată nu are soluţii întregi.VII.136. Numerele întregi x,y şi z verifică relaţia 17x+5y−2z = 0. Arătaţi cănumărul(3x+y)(z −x)(2x+2y+z)(3y +3z −x)A =210este natural, pătrat perfect.Mihai Haivas, IaşiSoluţie. Cum z = 17x+5y = 8x+2y + x+y , rezultă că x +y = 2n, n ∈ Z.2 2Atunci y = 2n − x şi z = 5n + 6x, deci A = 2·3·5·7·(x+n)2 ·(3n+2x) 2=210[(x+n)(3n+2x)] 2 şi concluzia se impune.170


Clasa a VIII-aVIII.130. Date fiind punctele A,B şi C, determinaţi punctele P din spaţiu cuproprietatea că PA 2 +PB 2 −PC 2 = AB 2 −AC 2 −BC 2 .Dan Popescu, SuceavaSoluţie. Fie M mijlocul segmentului AB; folosind teorema medianei în triunghiurilePAB şi CAB, obţinem că PA 2 + PB 2 = 2PM 2 + 1 2 AB2 , iar AC 2 + BC 2 =2CM 2 + 1 2 AB2 . Înlocuind în relaţia din enunţ, găsim că 2(PM 2 + CM 2 ) = PC 2 .Cu teorema medianei în △CMP, avem că 2(PM 2 + CM 2 ) = 4MN 2 + PC 2 , undeN este mijlocul segmentului CP. Atunci 4MN 2 = 0, de unde N = M. În concluzie,singurul punct P cu proprietatea din problemă este simetricul lui C faţă de mijlocullui AB.Notă. Cazultriunghiului dreptunghic înC, situaţie încarerelaţiadin eunţ devinePA 2 + PB 2 = PC 2 , constituie obiectul uneia dintre problemele de la ConcursulNaţional de Matematică, R.D.G. 1982, problemă ce apare şi în G.M. 11/2000. A sevedea şi problema înrudită G5 din RecMat 2/2001.VIII.131. Demonstraţi că(1−x)(1+y)+(1+x)(1−y) ≤4−(x+y) 2 ,∀x,y ∈ [−1,1].Rodica Pop şi Ovidiu Pop, Satu MareSoluţie. Inegalitatea din enunţ se scrie echivalent:(1−x)(1+y)+(1+x)(1−y)+2(1−x 2 )(1−y 2 ) ≤ 4−(x+y) 2 ⇔⇔ 0 ≤ 2−x 2 −y 2 −2(1−x 2 )(1−y 2 ) ⇔ 0 ≤ (Ô1−x 2 −1−y 2 ) 2 ,inegalitate adevărată. Egalitatea are loc dacă şi numai dacă y = ±x.VIII.132. Dacă a,b,c ∈ R ∗ + şi √ a 2 +ab+b 2 + √ a 2 +ac+c 2 + √ b 2 +bc+c 2 =√3(a+b+c), arătaţi că a = b = c.D.M. Bătineţu-Giurgiu, BucureştiSoluţie. Se arată uşor că √ √3a 2 +ab+b 2 ≥ (a + b), egalitatea realizându-se2√când a = b. Scriind inegalităţile similare şi sumând, deducem căÈ√ a2 +ab+b 2 ≥3(a+b+c). Cum se atinge egalitatea, rezultă că a = b = c.VIII.133. Fie E(x,y,z) = (1+x)(1+y)1+ √ + (1+y)(1+z)xy 1+ √ + (1+z)(1+x)yz 1+ √ .zxa) Arătaţi că E(x,y,z) ≥ x+y +z +3, ∀x,y,z ∈ (0,1).b) Arătaţi că E(x,y,z) ≤ x+y +z +3, ∀x,y,z ∈ (1,∞).Ion Nedelcu, Ploieşti şi Liviu Smarandache, CraiovaSoluţie. a) Dacă x,y ∈ (0,1), atunci (1+x)(1+y)1+ √ ≥ 1+ x+y (după calcule,xy 2această inegalitate revine la ( √ x− √ y) 2 ( √ xy−1) ≤ 0). Scriind încă două inegalităţianaloage şi adunând membru cu membru, obţinem că E(x,y,z) ≥ x+y +z +3, cuegalitate când x = y = z.171


) Pentru x,y ∈ (1,∞), avem că ( √ x− √ y) 2 ( √ xy−1) ≥ 0, deci (1+x)(1+y)1+ √ ≤xy1+ x+y etc.2VIII.134. Dacă a,b,c ∈ R ∗ + sunt laturile unui triunghi, demonstraţi că +2ac aa2 ·(a+b+c)b b2 +2ab ·c c2 +2bc


Clasa a IX-aIX.116. Rezolvaţi în numere întregi ecuaţia 8x 3 +y 3 +12xy = 8.Vasile Chiriac, BacăuSoluţie. Cu notaţiile s = 2x+y, p = 2xy, ecuaţia dată se scrie succesiv:(2x+y)(4x 2 −2xy +y 2 )+12xy−8 = 0 ⇔ s(s 2 −3p)+6p−8 = 0 ⇔⇔ (s−2)(s 2 +2s+4)−3p(s−2) = 0 ⇔ (s−2)(s 2 +2s+4−3p) = 0.Dacă s = 2, obţinem soluţiile x = m, y = 2 − 2m, m ∈ Z. Dacă s ≠ 2, atunci(s+1) 2 +3−2p = 0, deci s+1.3. Rezultă că s = 3k −1, k ∈ Z, apoi p = 3k 2 +1.Numerele 2xşi y vorfi soluţiile ecuaţiei t 2 −(3k−1)t+(3k 2 +1) = 0, cu discriminantul∆ = −3(k + 1) 2 < 0, prin urmare în acest caz nu obţinem soluţii întregi. În final,ecuaţia dată are soluţiile (m,2−2m), m ∈ Z.IX.117. Determinaţi funcţiile f : N → N cu proprietatea că f(x+y) = f(x) +f(y)+3xy(x+y), ∀x,y ∈ N.2Mihály Bencze, BraşovSoluţie. O soluţie evidentă este funcţia f 0 (x) = x 3 , ∀x ∈ N. Dacă f verificăecuaţia funcţională din enunţ, funcţia g(x) = f(x) − x 3 , x ∈ N, verifică ecuaţiaCauchy g(x+y) = g(x) + g(y), ∀x,y ∈ N. Atunci g(x) = αx, unde α = g(1) ∈ N,prin urmare f(x) = x 3 +αx, ∀x ∈ N, unde α ∈ N.IX.118. Dacă x,y,z ∈ R ∗ + , demonstraţi că are loc inegalitatea√ xyx+y ≥cycliccyclicz √ xy(x+z)(y +z)Ṁarian Tetiva, Bârlad√ √ xz yzSoluţie. Relaţiax+z + y +z ≥ 4· z √ xy(x+z)(y +z) este echivalentă cu √ x(y +z)+ √ y(x+z) ≥ 4 √ xyz şi, cum y +z ≥ 2 √ yz şi x+z ≥ 2 √ xz, are loc inegalitateaanunţată. Scriem încă două inegalităţi analoage şi, prin adunare, rezultă inegalitateadin enunţ.IX.119. Fie P un punct în interiorul triunghiului echilateral ABC şi P 1 ,P 2 ,P 3proiecţiile lui pe laturile triunghiului. Dacă G este centrul de greutate al triunghiuluiABC, demonstraţi că mijlocul G 1 al segmentului PG este centru de greutate pentrutriunghiul P 1 P 2 P 3 .Petru Asaftei, IaşiSoluţie. Vom folosi următoarele proprietăţi uzuale:(1) G este centrul de greutate al △ABC ⇔ −→ GA+ −→ −→ −→ GB + GC = 0;(2) Dacă P ∈ IntABC şi G este centrul de greutate al △ABC, atunci −→ PA+ −→ PB+−→PC = 3 −→ PG;(3) Dacă △ABC este echilateral atunci, cu notaţiile din enunţ, are loc egalitatea−→PA+ −→ −→ −−→ PB + PC = 2( PP1 + −−→ PP 2 + −−→ PP 3 ).173


Întrucât −−−→ G 1 P 1 + −−−→ G 1 P 2 + −−−→ G 1 P 3 = ( −−→ G 1 P + −−→ PP 1 )+( −−→ G 1 P + −−→ PP 2 )+( −−→ G 1 P + −−→ PP 3 ) =3 −−→ G 1 P +( −−→ PP 1 + −−→ PP 2 + −−→ PP 3 ) (3)= 3 −→GP + 1 2 2 (−→ PA+ −→ −→ (2)PB + PC) = 3 −→GP + 3 −→PG = −→ 0 ,2 2din (1) rezultă că G 1 este centrul de greutate al triunghiului P 1 P 2 P 3 .IX.120. Fie P un punct în interiorul triunghiului ABC astfel încâtPAB ≡PBC ≡PCA. Dacă AB ·−→ −→ −→ −→AP +BC · BP +CA· CP = 0, arătaţi că △ABC esteechilateral.Claudiu Mândrilă, elev, TârgovişteSoluţie. Fie ϕ măsura comună a unghiurilor din enunţ. Vectorii AB · −→ AP şi−→AP · AB au acelaşi modul, deci se obţin unul din celălalt printr-o rotaţie de unghiϕ. Raţionând analog pentru ceilalţi doi termeni ai sumei, rezultă că vectorul AP ·−→ −→ −→ −→ −→ −→AB + BP · BC + CP · CA se obţine din AB · AP + BC · BP + CA · CP printr-o−→ −→ −→ −→rotaţie de unghi ϕ, prin urmare AP · AB + BP · BC + CP · CA = 0. De aici,−→ −→ −→(AP − PB) · AB + (BP − PC) · AC = 0, aşadar PA = PB = PC şi atunci Peste centrul cercului circumscris △ABC. În aceste condiţii, m(BPC) = 2m(A); însăm(BPC) = 180 ◦ −m(PBC)−m(PCB) = 180 ◦ −ϕ−[m(C)−ϕ] = 180 ◦ −m(C) =m(A) +m(B) şi deducem căA ≡B. Analog se arată căB =C, adică △ABC esteechilateral.Clasa a X-aX.116. O urnă conţine x bile roşii şi n−x bile verzi, unde x,n ∈ N ∗ , x < n. x 3bile roşii şi x bile verzi sunt marcate cu 1, iar celelalte bile sunt marcate cu 2. Din2urnă se extrage o bilă şi se consideră evenimentele A: ”obţinem o bilă roşie” şi B:”obţinem o bilă marcată cu 1”.a) Determinaţi n şi x pentru care A şi B sunt evenimente independente.b) Aflaţi P(B|A).Laurenţiu Modan, BucureştiSoluţie. Se impune ca x.3 şi x.2, deci x = 6k, k ∈ N ∗ , iar n−x > x 2 , aşadarx ∈1,2,...,å2n3è.a) Evenimentele A şi B sunt independente când P(A)·P(B) = P(A∩B), adicăxn · 5x6n = x3n . Obţinem că 5x = 2n, prin urmare x = 6k, n = 15k, k ∈ N∗ .b) AvemcăP(B|A) = P(A∩B)P(A)= 2n−3x , deci P(B|A) = 2n−3x2n 2(n−x) .n−x− x 2n, unde P(A) = 1−P(A) = n−xn , iarP(A∩B) =X.117. Demonstraţi că √ n+1− √ n > 3√ n+1− 3√ n, ∀n ∈ N ∗ .Ionuţ Ivănescu, CraiovaSoluţie. Amplificând în ambii membri cu expresiile conjugate, observăm că ar fisuficient să dovedim că √ n+1+ √ n < 3(n+1) 2 + 3(n+1)n+ 3√ n 2 ; vom arăta174


că √ n+1 ≤ 3(n+1) 2 (1) şi că √ n < 3(n+1)n + 3√ n 2 (2). Ridicând ambiimembri la puterea a şasea, (1) revine la (n+1) 3 ≤ (n+1) 4 , adevărat pentru n ∈ N ∗ .Apoi, din inegalitatea mediilor, 3n(n+1)+ 3√ n 2 ≥ 2 6n 3 (n+1) > 6√ n 3 = √ n, deciare loc şi (2) şi cu aceasta soluţia problemei este completă.X.118. Determinaţi n ∈ N pentru caresin 2n x+cos 2n x+nsin 2 xcos 2 x = 1, ∀x ∈ R.Ani Drăghici şi Ileana Mândruleanu, CraiovaSoluţie. Pentru x = π 1, relaţia din enunţ devine4 2 n + 12 n +n = 1, sau 1+(n−4)·42 n−3 = 0. Această egalitate nu poate avea loc pentru n ≥ 4. Pentru n = 0, condiţiadin ipoteză devine 2 = 1, fals. Pentru n = 1, obţinem că 1+sin 2 xcos 2 x = 1, ∀x ∈ R,imposibil. Se verifică imediat că identitatea din enunţ este adevărată pentru n = 2 şin = 3, valori care constituie soluţiile problemei.X.119. Demonstraţi că pentru orice z 1 ,z 2 ,z 3 ∈ C are loc inegalitatea|z 1 +z 2 | 2 +|z 2 +z 3 | 2 +|z 3 +z 1 | 2 ≥ |z 1 +z 2 +z 3 | 2 +Im(z 1 z 2 +z 2 z 3 +z 3 z 1 ).Dan Nedeianu, Drobeta Tr. SeverinSoluţie. Se ştie (sau se arată uşor) că |z 1 +z 2 | 2 +|z 2 +z 3 | 2 +|z 3 +z 1 | 2 −|z 1 +z 2 +z 3 | 2 = |z 1 | 2 +|z 2 | 2 +|z 3 | 2 , deci rămâne să dovedim că(∗) |z 1 | 2 +|z 2 | 2 +|z 3 | 2 ≥ Im(z 1 z 2 +z 2 z 3 +z 3 z 1 ).Pornim de la inegalitatea evidentăÈ|z 1 −iz 2 | 2 ≥ 0, care se scrie sub formaÈ(z 1 −iz 2 )(z 1 +iz 2 ) ≥ 0, sau 2È|z 1 | 2 +Èi(z 1 z 2 −z 1 z 2 ) ≥ 0. Însă z−z = 2i·Imz, ∀z ∈ C,prin urmare i(z−z) = −2Imz, ∀z ∈ C, deci obţinem că 2·È|z 1 | 2 −2ÈImz 1 z 2 ≥ 0,adică tocmai inegalitatea (*).X.120. Fie ABCD un patrulater înscris în cercul C(M,R), având m(BOC) ≤90 ◦ , unde {O} = AC ∩BD. Demonstraţi căAB 2 −BC 2 +CD 2 −DA 2 ≥ AC ·BD −4R 2 .Când se realizează egalitatea?Florin Stănescu, GăeştiSoluţie. Notăm: AB = a, BC = b, CD = c, DA = d, AC = e, BD = f,m(AB) = 2x, m(BC) = 2y, m(CD) = 2z, m(DA) = 2t. Cu teorema sinusurilorîn triunghiurile ABC şi ACD, obţinem că a = 2Rsinx, b = 2Rsiny, c = 2Rsinz,d = 2Rsint. Cum ABCD este inscriptibil, ef = ac + bd (teorema lui Ptolemeu) şiatunci inegalitatea de demonstrat devine4R 2 (sin 2 x−sin 2 y +sin 2 z −sin 2 t) ≥ 4R 2 (sinxsinz +sinysint−1) ⇔⇔ 1−cos2x2− 1−cos2y2+ 1−cos2z2− 1−cos2t2175≥ cos(x−z)−cos(x+z) +2


cos(y −t)−cos(y +t)+ −1 ⇔ (cos2y +cos2t)−(cos2x+cos2z) ≥2≥ cos(x−z)+cos(y −t)−2 ⇔ 2cos(y +t)cos(y −t)−2cos(x+z)cos(x−z) ≥≥ cos(x−z)+cos(y −t)−2 ⇔ cos(y −t)[2cos(y +t)−1]+cos(x−z)··[2cos(y +t)−1] ≥ −2 ⇔ [2cos(y +t)−1]·2cos y −t+x−z cos y −t−x+z ≥2 2≥ −2 ⇔ [2cos(y +t)−1]·sin(t+z)·sin(y +z) ≥ −1.(Pe parcurs am folosit faptul că x+y +z +t = π.) Observăm că 2cos(y +t)−1 =2cos(BOC)−1 ∈ [−1,1], întrucât cos(BOC) ∈ [0,1], apoi sin(t+z) = sinB ∈ (0,1],sin(z +y) = sinA ∈ (0,1], deci ultima inegalitate este adevărată.Se realizează egalitatea când cos(BOC) = 0, sinB = 1 şi sinA = 1, adicăBOC =90 ◦ ,B = 90 ◦ ,A = 90 ◦ , prin urmare în cazul în care ABCD este pătrat.Clasa a XI-aXI.116. a) Fie α ∈ R + şi A ∈ M 2 (R) astfel încât det(αI 2 +A 2 ) = 0; demonstraţică detA = α.b) Arătaţi că există α ∈ R ∗ − şi A ∈ M 2(R) astfel încât det(αI 2 + A 2 ) = 0 şidetA ≠ α.Dan Popescu, SuceavabSoluţie. a) Fie A =ac d∈M 2 (R), iar u = a+d = trA, v = ad−bc = detA.CumA 2 = uA−vI 2 ,atunciαI 2 +A 2 ub= uA+(α−v)I 2 =ua+(α−v)uc ud+(α−v),prin urmare det(αI 2 +A 2 ) = u 2 v +ua(α −v) +ud(α −v) +(α −v) 2 +(α −v) 2 =u 2 α+(α−v) 2 . Deoarece u 2 , α,(α−v) 2 ∈ R + , suma precedentă este nulă numai dacăα−v = 0, adică detA = α.0b) De exemplu, putem considera A =1α = −1; atunci det(αI0 2şi 2 +A 2 ) =¬0 0iar detA = 2 ≠ α.0 3¬=0,XI.117. Dacă a,b ∈ R ∗ + , arătaţi că ax ≥ ln(bx + 1), ∀x ∈− 1 b ,∞dacă şinumai dacă a = b.Dumitru Săvulescu, BucureştiSoluţie. Dacă a = b, cu notaţia y = ax ∈ (−1,∞), ar trebui să dovedim că y ≥ln(y +1), fapt binecunoscut. Reciproc, fie f : (− 1 ,∞) → R, f(x) = ax−ln(bx+1).bCum f(x) ≥ 0 = f(0), ∀x ∈− 1 b ,∞, rezultă că x = 0 este punct de minim local allui f, prin urmare f ′ (0) = 0. Dar f ′ (x) = a−că a = b.bbx+1 , deci f′ (0) = a−b şi deducemXI.118. Fie α 1 ,α 2 ,α 3 ,α 4 numere reale pentru care limita l = limx→∞ (α 1{x} +176


α 2x+ 1 2+α 3x+ 2 3+α 4 {3x}) este finită (unde {·} desemnează partea fracţionară).Arătaţi că α 1 +α 2 +α 3 +3α 4 = 0.Marius Drăgan, BucureştiSoluţie. Se ştie că o funcţie continuă şi periodică ce are limita finită la +∞este constantă; în cazul nostru, deducem că α 1 {x} + α 2x+ 1 3+α 3x+ 2 3+α 4 {3x} = c, ∀x ∈ R, unde c este o constantă. Dând lui x valorile 0, 1 3 şi 2 3 , obţinemcă α 2 + 2α 3 = α 1 + 2α 2 = 2α 1 + α 3 = 3c, de unde α 1 = α 2 = α 3 = c. Atuncic({x}+x+ 1 3+x+ 2 3−1)+α 4 {3x} = 0 şi, folosind identitatea lui Hermite,rezultă că (c+α 4 ){3x} = 0, adică α 4 = −c şi de aici concluzia problemei.XI.119. Se consideră şirul (u n ) n≥0 definit prin u 0 = 1, u 1 = 2, u n+2 = 1 +14 arctg u n+1, n ∈ N ∗ . Arătaţi că şirul este convergent şi aflaţi-i limita.u nMoubinool Omarjee, ParisSoluţie. Observăm că u n ∈1, 3 2, ∀n ≥ 2. Fie a = 1 + π 16 = 1 + 1 4 arctg1;vom demonstra prin inducţie că |u n − a|


a 1 = 1k +a − 1b−a(k +a)(k +b) , rezultă că 1=k +x nk +b =b−a(k +a)(k +b) , întrucât y n > 0, ∀n ∈ N ∗ . Deducem că x n +k k +y n(k +a)(k +b), deb−ab) Deoarece x n+1 −x n = k2 +kx n> 0 şi y n+1 −y n = k2 +ky n> 0, şirurile diny nproblemă sunt strict crescătoare, deci au limită: l = lim x n, l ′ = limy n , cu l,l ′ > 0.n→∞Şirul (x n ) n≥1 este mărginit: x n ∈a, k2 +2ka+ab, deci l ∈ R ∗b−a+. Dacă, prinabsurd, l ′ ∈ R ∗ +, prin trecere la limită în relaţia x n+1 = k2 +kx n +x n y ny nx nrezultă căl = k2 +kl+ll ′l ′ , de unde k 2 +kl = 0; atunci l = −k < 0, fals. Rămâne că l ′ = +∞.Trecând acum la limită în relaţia1k +x n=b−a(k +a)(k +b) + 1k +y n, obţinem că1k +l = b−a(k +a)(k +b) , deci l = k2 +2ka+ab.b−aClasa a XII-aXII.116. Determinaţi funcţia f : Z 8 → Z 8 cu proprietatea căf(3x)+f(5x)+f(7x) = 3x 2 +5x+6, ∀x ∈ Z 8 .Bogdan Chiriac, student, IaşiSoluţie. Înlocuim succesiv pe x cu 3x,5x şi 7x în relaţia din enunţ; adunând celetrei egalităţi obţinute şi ţinând seama de ipoteză, obţinem că3f(x)+2(3x 2 +5x+6) =x 2 +3x+2. Prin înmulţire cu 3, rezultă că f(x) = x 2 +3x+2, ∀x ∈ Z 8 .XII.117. Aflaţi n ∈ N pentru care există α ∈ R ∗ − astfel încâtα(2t 3 +3)dt = n.Soluţie. Avem căαα−1(2t 3 + 3)dt =t 4 2 +3t¬αα−1α−1Romeo Cernat, Iaşi= 2α 3 − 3α 2 + 2α + 5 2 .Ecuaţiadin enunţ sescriesubformaf(α) = 0,unde f(α) = 2α 3 −3α 2 +2α+52 −n.Întrucât f ′ (α) = 6α 2 − 6α + 2 ≠ 0, ∀α ∈ R, ecuaţia f(α) = 0 are o unică soluţiereală, iar aceasta se află în intervalul (−∞,0) dacă şi numai dacă f(0) > 0. Condiţiaf(0) = 5 −n > 0 conduce la valorile căutate ale lui n, anume n ∈ {0,1,2}.2XII.118. Fie f : [0,1] → R o funcţie continuă cu proprietatea că1f(x)dx =ln a+1a , unde a ∈ (0,1) este dat. Demonstraţi că există x 0 ∈ (0,1) astfel încât1x 0 +1 < f(x 0) < 1 x 0.Bogdan Victor Grigoriu, Fălticeni1780


1 a+1Soluţie. Cum1dx = ln , funcţia f are proprietatea că10 x+a a 0f(x)−1= 0. Fie g(x) = f(x)−x+adx 1 , x ∈ [0,1] funcţie continuă, iar G o primitivăx+asa; atunci G(1) = G(0) şi, cum G este derivabilă pe [0,1], sunt îndeplinite condiţiileteoremei lui Rolle. Obţinem că există x 0 ∈ (0,1) pentru care G ′ (x 0 ) = 0, adică1f(x 0 ) =x 0 +a . Deoarece a ∈ (0,1), rezultă că 1x 0 +1 < 1x 0 +a < 1 , de undex 0cerinţa problemei.n(n 2 +1 2 )(n 2 +2 2 )·...·(n 2 +n 2 )XII.119. Demonstraţi că limn→∞ n 2 = 2 e 2 ·e π 2.Adrian Corduneanu, IaşiSoluţie. Considerăm funcţia continuă, deci integrabilă, f : [0,1] → R, f(x) =ln(1+x 2 ). Integrând prin părţi, obţinem că I =1ln(1+x 2 )dx = xln(1+x )¬12 −210x 211+x2dx = ln2 − 21−10 1+x 2dx=ln2 + π 20− 2. Considerând şirulde diviziuni ∆ n = (x 0 = 0 n ,x 1 = 1 n ,...,x n = n ) cu şirul normelor tinzând lanzero,observămcăs n = 1 nÈk=1f(x k ) = 1 12 22 n2ln1+n n n21+ n2·...·1+ n 2=n(n 2 +1)(n 2 +2 2 )·...·(n 2 +n 2 )lnesteun şirdesume Riemannacăruilimită esten 2egală cu I. Astfel, limita şirului din enunţ este e I = 2 e 2 ·e π 2.XII.120. Fie f : R → (0,∞) o funcţie continuă cu lim f(x) = 0, astfel încâtx→∞noricare ar fi n ∈ N ∗ , există un unic x n ∈ R ∗ + pentru carexn0sinf(t) x ndt. Calculaţi limf(t) n→∞ n .Soluţie. Considerăm şirul a n = 1 nn000ln(1+f(t))dt =f(t)Florin Stănescu, Găeştisinf(t)dt, n ∈ N ∗ . Folosind lema Stolzf(t)sinf(t)n→∞n+1sinf(c n ),f(c n )Cesàroşiteoremademedie, obţinem lim a n = lim dt = limn→∞ n f(t) n→∞cu n < c n < n+1. Cum c n → ∞ şi lim f(x) = 0, rezultă că lim a n = 1.x→∞ n→∞Utilizând acum teorema lui Lagrange şi inegalitatea ln(1 + t) ≤ t, ∀t ∈ [0,∞),deducem căx n0ln(1+f(t))dt = F(x n )−F(0) = x n·F ′ (y n ) = x ln(1+f(y n))n·f(t)f(y n )x n · 1 = x n , ∀n ∈ N ∗ (unde F(t) este o primitivă a funcţieix n >x n0ln(1+f(t))dt =nf(t) 0


lim x n = ∞.n→∞1Întrucât lim f(x) = 0, avem că (1 + f(t))f(t)> 2 pentru t > t 0 convenabilales, decixx→∞ln(1+f(t))>xln2 → ∞ când x → ∞. În calculul limitei1limx→∞ xx00 f(t) 0ln(1+f(t))dt vom putea aplica regula lui l ′ Hospital (cazul ∞ ) şi obţinemf(t)∞ln(1+f(x))că limita este egală cu limx→∞În relaţiann10f(x)sinf(t)dt = x nf(t) n ·= 1.1x nx n0∞ şi, conform celor de mai sus, rezultă că 1 =limn→∞ln(1+f(t))dt facem pe n să tindă laf(t)x n x naşadar limn·1,n→∞ n = 1.(continuare de la pag. 147)2. Asteroizi cu nume de români - toţi aceştia fac parte din centura principală deasteroizi situată între Marte şi Jupiter• 2331 Parvulesco (Constantin Pârvulescu (1890-1945) şi copii săi Carinaşi Antares), cu diametrul estimat între 11 şi 24 km şi cu o rotaţie în jurulSoarelui la 3,78 ani;• 4268 Grebenikov (Eugeniu Grebenicov, n. 1932, Basarabia), cu diametrul5-12 km şi o rotaţie în jurul Soarelui în 4,28 ani;• 6429 Brâncuşi, cu diametrul de 4-9 km şi o rotaţie în 3,16 ani;• 9253 Oberth (Herman Iulius Oberth, 1894-1989), cu diametrul până la6 km;• 9403 Sanduleak (Nicolae Sanduleak, 1933-1990), cu diametrul până la6 km, o rotaţie în 4 ani;• 9493 Enescu, cu diametrul până la 9 km;• 9494 Donici (Nicolae Donici, 1875-1957, membru fondator al UAI);• 9495 Eminescu, cu diametrul până la 6 km şi o rotaţie în 3,23 ani;• 10 034 Birlan (Mirel Bîrlan, n. 1963, Giurgiu);• 12 498 Dragesco (Jean Dragesco, n. 1920), cu diametrul de cel mult10 km.3. Comete cu nume de români• Cometa Daimaca 1943c (Victor Daimaca, 1892-1969, Turnu Severin);• Cometa van Gent-Peltier-Daimaca 1943W1.180


Soluţiile problemelor pentru pregătireaconcursurilor propuse în nr. 1/2011A. Nivel gimnazialßÞÐn−1 de 0G196. Fie M mulţimea numerelor naturale nenule scrise numai cu cifre pare,care au cel mult 2011 cifre. Arătaţi că suma inverselor elementelor lui M este maimică decât 4.Cecilia Deaconescu, PiteştiSoluţie. Pentrufiecaren ∈ {1,2,...,2011},mulţimeaM conţine 4·5 n−1 elementecare se scriu cu n cifre pare şi orice astfel de element este cel puţin egal cu 2 0...0 =2·10 n−1 . Notând cu S n suma inverselor numerelor de n cifre din M, avem că S n


Dacă b = 1, atunci a 2 +ab+b 2 = a 2 +a+1 nu este pătrat când a ∈ N ∗ , întrucâta 2 < a 2 +a+1< (a+1) 2 . Dacă b = 2, atunci a 2 +ab+b 2 = a 2 +2a+4nu este pătratperfect pentru a ∈ N ∗ , deoarece (a+1) 2 < a 2 +2a+4 < (a+2) 2 . Dacă b = 4, atuncia 2 +ab+b 2 = a 2 +4a+16 are proprietatea că (a+2) 2 < a 2 +4a+16 < (a+4) 2 şia 2 +4a+16 = (a+3) 2 , deci iarăşi nu este pătrat perfect.În concluzie, valorile căutate sunt b ∈ N ∗ \{1,2,4}.a 2G200. Arătaţi că2b 3 +2c 3 +5 + b 22a 3 +2c 3 +5 + c 22a 3 +2b 3 +5 ≤ 1 3 , ∀a,b,c ∈[0,1].Dan Nedeianu, Drobeta Tr. SeverinSoluţie. Întrucât c 3 ∈ [0,1], rezultă că 2c 3 a 2+ 3 ≤ 5, deci2b 3 +2c 3 +5 ≤a 3Scriindşianaloagele,dupăsumareobţinemcăÈ2(a 3 +b 3 +c 3 )+3 . a 22b 3 +2c 3 +5 ≤a 2 +b 2 +c 22(a 3 +b 3 +c 3 )+3 . Dar 2a3 +1 ≥ 3a 2 (echivalent cu (a−1) 2 (2a+1) ≥ 0, adevăratcânda ∈ [0,1])şi, similar, 2b 3 +1 ≥ 3b 2 , 2c 3 +1 ≥ 3c 2 , prin urmare2(a 3 +b 3 +c 3 )+3 ≥3(a 2 +b 2 +c 2 a 2 +b 2 +c 3), adică2(a 3 +b 3 +c 3 )+3 ≤ 1 , şi de aici concluzia problemei.3G201. Se consideră triunghiul ABC cu AC ≠ BC > AB. Dacă există un punctD ∈ (BC) pentru care AB 2 = BD · BC şi AD 2 = BD · DC, arătaţi că − 1 3 ≤AB 2BC 2 −AC 2 ≤ 1.Ovidiu Pop, Satu MareSoluţie. Cu notaţiile uzuale în triunghi, din BD = c2 a şi AD2 = BD(a −BD)obţinem că AD =a√ c a2 −c 2 . Pe de altă parte, folosind formula lui Heron, avem căh a = 2S a = 1 √2a2 b2a2 +2b 2 c 2 +2c 2 a 2 −a 4 −b 4 −c 4 . Întrucât AD ≥ h a, rezultă că4a 2 c 2 −4c 4 ≥ 2a 2 b 2 +2b 2 c 2 +2c 2 a 2 −a 4 −b 4 −c 4 , adică (3c 2 +a 2 −b 2 )(c 2 +b 2 −a 2 ) ≤ 0.Dacă a < b, relaţia precedentă conduce la 3c 2 +a 2 −b 2 c 2≤ 0, de undeb 2 −a 2 ≤ 1 3şi atunci − 1 3 ≤ c2a 2 −b 2 < 0 < 1. Dacă a > b, obţinem că c2 +b 2 −a 2 ≤ 0, prin urmare− 1 3 < 0 < c2a 2 ≤ 1 şi cu aceasta soluţia este completă.−b2 G202. Fie ABCD pătrat, iar punctele M şi N pe laturile AB, respectiv ADsunt astfel încât AM = DN = k · AB. Notăm {P} = CN ∩ DM şi {Q} = AP ∩CD. Determinaţi valorile lui k pentru care PQ este bisectoare, respectiv mediană întriunghiul CDP.Neculai Roman, Mirceşti (Iaşi)Soluţie. Se arată uşor că DM ⊥ CN. Folosind teorema catetei în △CDN,182


obţinem că NPPC = DN2CD 2 = k2 . Cu teorema lui Menelaus în△CDN (transversalaQ−P−A) deducem că NP ·CQQD·DAPC AN =1, deci k 2 · CQQD ·DNDC1 QD= 1, prin urmarek −1 QC = k21−k .Din asemănarea △PCD ∼ △DCN obţinem că PDPC =DQ= k. Atunci PQ este bisectoare în △CDP dacă şi numai dacăQC = PDPC ,k 2adică1−k = k, de unde k = 0 (care nu convine) sau k = 1 , care este valoarea2k 2căutată. Apoi, PQ este mediană în △CDP dacă şi numai dacă = 1, cu soluţia1−kacceptabilă k = −1+√ 5.2G203. Fie date numerele reale a şi b cu a < b < 2a. Triunghiurile isosceleABC şi A ′ B ′ C ′ au aceeaşi axă de simetrie, aceeaşi dreaptă suport a bazelor, iarBC = a, AB = AC = b, B ′ C ′ = b, A ′ B ′ = A ′ C ′ = a. Dacă {M} = AB ∩ A ′ B ′ ,{N} = AC ∩A ′ C ′ , arătaţi că MN este linie mijlocie în △ABC dacă şi numai dacăa 3 +b 3 = 2a 2 b.Temistocle Bîrsan, IaşiSoluţie. Din motive de simetrie, avem că MN‖BC. Notăm cu D mijlocul comunal segmentelor BC şi B ′ C ′ şi fie h = AD, iar h ′ = A ′ D; evidentcă h =Öb 2 − a24 , h′ =Öa 2 − b2 . Aplicând teorema lui4Menelaus în △ABD, cu transversala A ′ −M −B ′ , obţinem căA<strong>MB</strong>M NMA = b−a h ′·b h−h ′ şi, cum MA + <strong>MB</strong> = b, deducem căAM = b2 (h−h ′ )B B D C Cbh−ah ′ . Din faptul că △AMN ∼ △ABC rezultă căMN = a b ·AM = ab(h−h′ )bh−ah ′ = ab(√ 4b 2 −a 2 − √ 4a 2 −b 2 )b √ 4b 2 −a 2 −a √ . Condiţia ca MN să fie4a 2 −b2 linie mijlocie în △ABC este 2MN = a, ceea ce revine la 2b( √ 4b 2 −a 2 − √ 4a 2 −b 2 ) =b √ 4b 2 −a 2 − a √ 4a 2 −b 2 , adică b √ 4b 2 −a 2 = (2b − a) √ 4a 2 −b 2 ; după ridicare lapătrat şi reduceri, se obţine a 3 +b 3 = 2a 2 b.Analog se poate arăta că MN este linie mijlocie în △A ′ B ′ C ′ dacă şi numai dacăa 3 +b 3 = 2ab 2 .G204. Despre un punct de pe o muchie a unui tetraedru vom spune că este punctbisector dacă el este piciorul a două bisectoare ale unor unghiuri ale feţelor. Arătaţică numărul punctelor bisectoare ale unui tetraedru este 0, 2 sau 6.Silviu Boga, IaşiSoluţie. Pefiecaremuchieexistăcelmultun punctbisector. DacăM ∈ (AB) estepunct bisector, atunci CM şi DM sunt bisectoare, deci MA<strong>MB</strong> = CACB = DADB . Rezultă183DNAPQMACB


că ACAD = BCBD , prin urmare bisectoarele unghiurilorCAD şiCBD se vor întâlniîntr-un punct P ∈ (CD) încât ACAD = BCBD = PC , adică P este şi el punct bisector;PDdeducem astfel că numărul punctelor bisectoare este par. Dacă un tetraedru ar aveapatru puncte bisectoare, fie acestea M ∈ (AB), P ∈ (CD), N ∈ (BC) şi Q ∈ (AD),din AC ·BD = AD ·BC şi AC ·BD = AB ·CD urmează că AD ·BC = AB ·CD,ceea ce arată că vom avea puncte bisectoare şi pe muchiile AC şi BC; astfel, numărulpunctelor bisectoare poate fi foar 0,2 sau 6.Evident, tetraedrul regulat are şase puncte bisectoare. Alegând faţa ABC culaturile de lungimi distincte şi vârful D astfel încât proiecţia sa pe planul (ABC) să fiecentrul cercului circumscris acestei feţe, obţinem un tetraedru fără puncte bisectoare.În sfârşit, un tetraedru cu AB = BC = CA şi DA ≠ DB = DC ≠ BC are exactdouă puncte bisectoare.G205. Două panouri luminoase sunt situate în plane paralele (verticale). Ele auforma a două dreptunghiuri identice, împărţite fiecare în câte zece pătrate congruentecu ajutorul câte unei linii orizontale şi a câte patru linii verticale. În cele 18 vârfuride pătrate care se formează pe fiecare panou sunt instalate beculeţe. La un moment datpe fiecare panou se aprind câte două beculeţe, la întâmplare. Care este probabilitateaca cele patru beculeţe aprinse să se afle într-un acelaşi plan?Gabriel Popa şi Cristian Lazăr, IaşiSoluţie. Două beculeţe dintre cele 18 aflate pe un panou pot fi alese în 18·17 =2153 moduri. Cele patru beculeţe pot fi alese în 153 2 moduri.Gândim o pereche de beculeţe de pe un panou ca pe un segment cu capetele înnodurile reţelei de pătrate. Două segmente de pe cele două panouri sunt coplanaredacă şi numai dacă sunt incluse în drepte paralele. Acest lucru se întâmplă când suntambele orizontale, ambele verticale sau ambele oblice, dar formând unghiuri egale cuorizontala.Pe fiecare dreptunghi avem câte 6·5 · 3 = 45 segmente orizontale, 3 · 6 = 182segmente verticale, 5 segmente de o pantă 2, 14 cu panta 1,3 cu panta 2 , 10 cu panta312 , un segment cu panta 2 5 , 6 cu panta 1 3 , 4 cu panta 1 4 şi 2 segmente cu panta 1 5 .Rezultă că numărul cazurilor favorabile evenimentului urmărit este 45 2 +18 2 +5 2 +14 2 +3 2 +10 2 +1+6 2 +4 2 +2 2 = 2736.Probabilitatea cerută este P = 3042601 ≃ 0,117.B. Nivel licealL196. Demonstraţi că în orice triunghi ascuţitunghic are loc inegalitatea(ctgA+ctgB +ctgC) 3(ctgA+ctgB)(ctgB +ctgC)(ctgC +ctgA) ≥ 83 √ 3184(1+cosAcosBcosC) 3.sinAsinBsinCGheorghe Costovici, Iaşi


Soluţie. Cum ctgA+ctgB = sin(A+B)sinAsinB = sinC, avem căsinAsinBE =(ctgA+ctgB +ctgC) 3(ctgA+ctgB)(ctgB +ctgC)(ctgC +ctgA) · sinAsinBsinC(1+cosAcosBcosC) 3 == (ctgA+ctgB +ctgC)3 (sinAsinBsinC) 3sinAsinBsinC ·(1+cosAcosBcosC) 3 ==(cosAsinBsinC +cosBsinCsinA+cosCsinAsinB)3sinAsinBsinC(1+cosAcosBcosC) 3 .Întrucât −cosC = cos(A+B) = cosAcosB −sinAsinB, rezultă căcosAsinBsinC +cosBsinCsinA+cosCsinAsinB == sinCsin(A+B)+cosC(cosC +cosAcosB) == sin 2 C +cos 2 C +cosAcosBcosC = 1+cosAcosBcosC.1AtunciE =sinAsinBsinC ≥ 27(sinA+sinB +sinC) 3 ≥ 27(3sin A+B+C3) = 13 sin 3 π =383 √ (am aplicat inegalitatea mediilor MG ≤ MA şi inegalitatea lui Jensen pentru3funcţia concavă sin :0, π 2→[0,1]).Nota autorului. Problemaeste inspiratăde cea a d-lui Marian Tetiva, apărutăîn AMM 2008, pg. 77:(1−cosA)(1−cosB)(1−cosC)cosAcosBcosC≥8(tgA+tgB +tgC) 327(tgA+tgB)(tgB +tgC)(tgC +tgA) .Asemănător rezolvă problema d-l Titu Zvonaru, Comăneşti. O altă soluţie, primităde la d-nii Mihàly Bencze, Braşov şi Ioan Viorel Codreanu, Maramureş, reduceinegalitatea de demonstrat la una cunoscută, anume, S ≤ 3√ 34 R2 .L197. Fie ABCDEFGH un paralelipiped dreptunghic, iar S o sferă prin A careintersectează segmentele AB,AD,AE şi AG în M,N,P, respectiv Q. Arătaţi căAM ·AB +AN ·AD+AP ·AE = AQ·AG.Claudiu Ştefan Popa, IaşiSoluţie. PlanuldreptunghiuluiABCD şisferaS auîncomunpunctelenecoliniareA,M şi N, deci intersectează S după cercul circumscris triunghiului AMN. DreaptaAC intersectează cercul circumscris triunghiului AMN, deci şi sfera S, într-un punctR ∈ (AC] (în caz contrar, am avea că AR > AC > diametrul sferei, fals). Conformcu L166 din RecMat 2/2009, obţinem că AM · AB + AN · AD = AR · AC. Planuldreptunghiului ACGE şi S au în comun punctele A,P şi R; repetând raţionamentul,deducem că AR·AC +AP ·AE = AQ·AG şi de aici cerinţa problemei.Aşa cum, în plan, problema L166 extinde teorema lu Pitagora, problema L197extinde cunoscuta: pătratul diagonalei unui paralelipiped dreptunghic este suma185


pătratelordimensiunilor sale (care se obţine când paralelipipedul este înscris însferă).Probabil că o relaţie asemănătoare este valabilă în R n .Notă. Soluţie corectă au dat şi d-nii I.V. Codreanu şi T. Zvonaru.L198. Fie ABC un triunghi ascuţitunghic înscris în cercul C. Cercul C ′ estetangent cercului C în punctul A şi laturii BC în punctul D. Arătaţi că AD estebisectoarea unghiuluiBAC.Titu Zvonaru, ComăneştiSoluţie. Dacă triunghiul ABC este isoscel cu b = c, concluzia este imediată.Presupunem că b > c şi fie T intersecţia cu BC a tangenteiîn A la cercul C. Cu puterea punctului T faţă deAC, obţinem că TA 2 = TB ·TC ⇔ TA 2 = TB(TB +a).Aplicând teorema cosinusului în △TAB, deducem căTA 2 = TB 2 + AB 2 − 2TB · AB · cosTBA şi atunci T B D CTB 2 + a · TB = TB 2 + c 2 + 2c · TB · cosB, de undec 2TB =a−2ccosB , iarTB+a = c2 +a 2 −2accosB=a−2ccosBb 2a−2c·cosB . Deoarece TA şi TD sunt tangente cercului C′ , avem că TA = TD,prin urmareBD = TD−TB =TB(TB +a)−TB = bc−c2a−2ccosB =c(b−c)a=a 2 −2accosB = ac(b−c)b 2 −c 2 = ac ab⇒ CD =b+c b+cşi de aici concluzia rezultă imediat.Notă. Profesorul N. Roman semnalează că problema apare în lucrarea Transformărigeometrice de D. Smaranda şi N. Soare, la p. 104, cu numărul 67 (soluţia dela p. 216 foloseşte omotetia).L199. Fie O centrul cercului circumscris triunghiului ABC Aşi {M} = OB∩AC, {N} = OC∩AB. Dacă OM = ON, arătaţică triunghiul este isoscel sau dreptunghic.N OMTemistocle Bîrsan, IaşiSoluţie (Gh. Iurea şi T. Zvonaru). Se constatăBCuşor că nici B, nici C nu poate fi obtuz sau drept. Întrucât△MOC ≡ △NOB (L.U.L.), obţinemcă<strong>MB</strong>N ≡MCN. DacăO ∈ IntABC, atunci π 2 −C = π −B, adică △ABC este isoscel; dacă O ∈ ExtABC,2deducem că π 2 +C = π +B, adică △ABC este isoscel; în sfârşit, dacă O ∈ (BC),2rezultă că △ABC este dreptunghic şi soluţia este completă.Notă. Redacţiaregretăincludereaacesteiproblemeînsecţiuneapentrupregătireaconcursurilor.L200. În raport cu un reper cartezian xOy, se consideră punctele M32 ,1,B3a−2,0şi C0, 2a+3 unde a ∈ R\0,a a, 2 3 ,−3 precum şi familia de drepte2,186


d m : y = mx+ 2−3m , m ∈ R. Notăm cu C a cercul circumscris triunghiului OBC.2a) Demonstraţi că, pentru orice m ∈ R, dreapta d m intersectează C a în douăpuncte distincte P şi Q.b) Arătaţi că produsul MP ·MQ este independent de a şi m.Gabriel Popa şi Paul Georgescu, IaşiSoluţie. a) Observăm că BC 2 = 13(a2 +1)a 2 , deci raza cercului C a este r =12 BC =13(a2 +1). Centrul cercului este mijlocul S al ipotenuzei BC, de coordonateS3a−22|a|2a , 2a+3√13iar SM =(x S −x M )2a, 2 +(y S −y M ) 2 =2|a| . Evidentcă SM < r, prin urmare M ∈ IntC a . Pe de altă parte, punctul M aparţine oricăreidrepte d m din familia considerată şi de aici cerinţa problemei.b) Faptul că MP ·MQ nu depinde de m rezultă din puterea punctului M faţă deC a . Pentru a arăta că produsul nu depinde de a, vom demonstra că toate cercurileC a au aceeaşi axă radicală, iar M se află pe această axă radicală. Observăm căOM 2 +SM 2 = 134 + 134a 2 = 13(a2 +1)4a 2= r 2 , prin urmare OM ⊥ SM. Fie A(3,2)simetricul punctului O faţă de M; deducem că A ∈ C a , ∀a ∈ R\0, 2 3 ,−3 2, deci axaradicală a tuturor cercurilor C a este OA.Notă. Au rezolvat problema şi d-nii I.V. Codreanu şi T. Zvonaru.L201. Demonstraţi că pentru orice număr prim p > 2 2k +1, ∀k ∈ N ∗ , numărulp 22k+1 − 1 se divide cu 2 2(k+1) ·mÉj=1prime din mulţimea {2 i +1|i = 1,2k}.q j , unde {q j |j = 1,m} este mulţimea numerelorNeculai Roman, Mirceşti (Iaşi)Soluţie. Avem că p 22k+1 − 1 = (p 22k + 1)(p 22k−1 − 1)...(p 2 + 1)(p + 1)(p − 1).Cum p este evident impar, fiecare factor din descompunerea precedentă se divide cu2, deci p 22k+1 −1 se divide cu 2 2(k+1) .Pe de altă parte, p ab −1 se divide cu p a −1 şi cu p b −1. Cum 2 2k+1 = 2 n 2 2k+1−n ,cu n = 1,k, rezultă că p 22k+1 − 1 se divide cu p 2n − 1 şi cu p 22k+1−n − 1, pentrun = 1,k. Folosind mica teorema a lui Fermat, dacă q j este un număr prim de forma2 i +1, atunci p qj−1 se divide cu q j şi deci p 22k+1 −1 se va divide cu q j . Cum numerele2 2(k+1) , q 1 ,q 2 ,...,q m sunt relativ prime două câte două, rezultă concluzia problemei.Notă. Pentru k = 2 obţinem problema 8258 din G.M.-B. 6/1968. Pentru k = 4găsim că pentru orice număr prim p > 2 8 +1 = 257, numărul p 512 −1 se divide cu2 10 ·2·5·17·257.L202. Determinaţi numerele reale a şi b pentru care a√√ n+1 n−1 ≤ b , ∀n ∈ N ∗ .n187√ n+2n+1≤ √ n+2 −Gheorghe Iurea, Iaşi


Soluţie. Ipoteza problemei este echivalentă cu a ≤ (n+1)(√ n+2− √ n+1)√ n+2=x n , ∀n ∈ N ∗ şi b ≥ n(√ n+2− √ n+1)√ = y n , ∀n ∈ N ∗ . Se arată că şirurile (x n ) n≥1n+1şi (y n ) n≥1 sunt strict crescătoare şi atunci a ≤ x 1 , iar b ≥ lim y n, prin urmaren→∞a ≤ 2(3−√ 6)şi b ≥ 1 nÉ3 2 .L203. Fie f,g polinoame cu coeficienţii reali, nu ambele constante, iar P =f + ig ∈ C[X]. Presupunem că rădăcinile lui P sunt numere complexe cu părţileimaginare strict negative. Dacă λ,µ ∈ R, λ 2 +µ 2 ≠ 0, arătaţi că rădăcinile polinomuluiQ = λf +µg sunt reale.Adrian Reisner, ParisSoluţie. Fie z k , k = 1,n, rădăcinile polinomului P, deci P = a · (X − z k ).k=1Dacă z este un număr complex cu partea imaginară strict pozitivă, este imediat că|z − z k | > |z − z k |, decinÉk=1mÉ |z − z k | >k=1|z − z k |, adică |P(z)| > |P(z)|. Dacăz este număr complex cu partea imaginară strict negativă, atunci |P(z)| < |P(z)|.Conchidem că |P(z)| = |P(z)| dacă şi numai dacă z ∈ R.Fie z o rădăcină a lui Q; vom arăta că |P(z)| = |P(z)|, de unde concluzia. Avem|P(z)| 2 − |P(z)| 2 = (f(z) − ig(z))(f(z) − ig(z)) − (f(z) + ig(z))(f(z) − ig(z)) =2i(g(z)f(z)− f(z)g(z)) = 4Im(f(z)g(z)). Cum λf(z) + µg(z) = 0, unde λ,µ ∈ R,λ 2 + µ 2 ≠ 0, rezultă că imaginile geometrice ale lui f(z) şi g(z) sunt coliniare cuoriginea, deci f(z)g(z) ∈ R şi astfel |P(z)| = |P(z)|, ceea ce încheie rezolvarea.L204. Fie A o matrice pătratică de ordinul n având elementele a ij din mulţimea{0,1} şi următoarele proprietăţi: i) a ii = 0 pentru orice i ∈ {1,2,...,n}; ii) dacăa ij = 1 (pentru i ≠ j din mulţimea {1,2,...,n}), atunci a ji = 0; iii) pentru fiecarep ∈ {0,1,...,n−1}, matricea are o linie pe care se află exact p elemente egale cu 1.Să se arate că există o permutare a mulţimii {1,2,...,n} astfel încât, dacă seaplică această permutare liniilor matricei A şi apoi coloanelor matricei astfel obţinute,rezultă în final o matrice cu toate elementele care sunt egale cu 1 situate deasupradiagonalei principale. Care este polinomul caracteristic al unei asemenea matrice?Marian Tetiva, BârladSoluţie. Fie i k linia matricei A pe care se găsesc exact n−k elemente egale cu1 (pentru fiecare 1 ≤ k ≤ n) şi permutarea α definită prin α(k) = i k , oricare ar fik ∈ {1,...,n}. Spunem că aplicăm liniilor matricei A permutarea α dacă formăm onouă matrice B obţinută din A mutându-i linia i 1 pe prima poziţie, linia i 2 pe poziţiaa doua şi aşa mai departe. Spunem aşa deoarece, de fapt, B = PA, unde P estematricea (numită de permutare) corespunzătoare permutării α, adică matricea careare toate elementele 0, cu excepţia elementelor din poziţiile (k,i k ), egale cu 1. Dacăo matrice se înmulţeşte cu P −1 (care este matricea de permutare corespunzătoareinversei lui α la dreapta, precum şi transpusa matricei P) rezultatul este matriceainiţială asupra coloanelor căreia s-a efectuat aceeaşi permutare α.Facemacestlucru,adicăformămdinAmatriceaB obţinutădinpermutarealiniilor188


lui A conform permutării α. Desigur, B are n−1 elemente egale cu 1 pe prima linie,n − 2 elemente egale cu 1 pe linia a doua şi tot aşa (are toate elementele nule peultima linie).Fie j k numărulelementeloregalecu1depe coloanai k , pentrufiecarek, 1 ≤ k ≤ n.Avem j k ≤ n−1−(n −k) = k −1 pentru orice 1 ≤ k ≤ n (conform proprietăţilori) şi ii)) şi j 1 +...+j n = n(n−1)/2 (conform proprietăţii iii) şi faptului că numărultotal de elemente egale cu 1 de pe coloane este egal cu numărul elementelor egalecu 1 numărate pe linii). Din acestea două obţinem că, de fapt, j k = k − 1 pentruorice 1 ≤ k ≤ n. Aplicăm coloanelor matricei B tot permutarea α şi obţinem astfelmatricea C = BP −1 care are pe linia i exact n−i elemente egale cu 1 pentru fiecare1 ≤ i ≤ n şi are pe coloana j exact j−1 elemente egale cu 1, pentru fiecare 1 ≤ j ≤ n.Mai mult, cele j − 1 elemente de pe coloana j trebuie să fie primele (de sus înjos), altfel nu s-ar respecta numărul elementelor de pe linii. Astfel, C este matriceacăutată, obţinută din A prin aplicarea permutării α asupra liniilor sale şi apoi prinaplicarea permutării α asupra coloanelor matricei B (care rezultă după permutarealiniilor).Cum C = PAP −1 ,C şi A au acelaşi polinom caracteristic; dar C este (strict)superior triunghiulară, deci polinomul este X n - adică şi polinomul caracteristic al luiA este tot X n .Nota autorului. Această problemă reprezintă 2nÈo variantă a problemei 2nÈ11487 dinThe American Mathematical Monthly 2/2010.(−1)L205. Calculaţi∞k=1k−1 lnk.kMarian Tetiva, BârladSoluţie. Seria este convergentă conform criteriului lui Leibniz. Avem:(−1)2nÈk=1k−1 lnk= 2nÈk=1lnk ln2k−2k knÈk=1 2k = lnk 1 lnkk=n+1 k −nÈk=1==kln2k=n+1 k −ln 2 2n−ln 2 n 12−nÈk=1definită prin γ = limn→∞ ( nÈk=1k −lnnln2+ ln2 2. După cum se ştie, constantalui Euler este21−lnn). Exactaşacum sejustifică existenţa şifinitudineaklnkk − ln2 n, ∀n ≥ 1, este convergent.2şi teorema lui Lagrange pentru a obţine ine-acestei limite, putem arăta că şirul x n = nÈk=1(Folosim monotonia funcţiei x ↦→ lnxxgalităţile ln(n+1) < ln2 (n+1)−ln 2 n< lnn , ∀n ≥ 3, iar aceste inegalităţi folosescpentru demonstrarea convergenţei şirului menţionat.) Din această convergenţăn+1 2 nlnkrezultă că limk − ln2 2n−ln 2 n(−1) k−1 lnk) = 0 şi atunci lim =2n→∞ kln 2 22n→∞ ( 2nÈk=n+1−γln2. Evident, aceasta va fi şi suma seriei din enunţ.2nÈk=1189


Probleme propuse 1Clasele primareP.216. Fie numerele a = 1+○ şi b = 9−□. Înlocuiţi cercul şi pătratul cu cifrecorespunzătoare astfel încât a+b = 15.(Clasa I)Amalia Munteanu, elevă, IaşiP.217. O elevă a desenat un trenuleţ cu 23 vagoane pe care le-a colorat folosind,pe rând, culorile roşu, galben, albastru, roşu, galben ş.a.m.d. Ce culoare a folositpentru vagonul din mijloc?(Clasa I)Mihaela Gîlcă, elevă, IaşiP.218. Mihaela are 14 ani. Ea s-a născut când sora sa avea 7 ani, dar cu 5 aniînaintea fratelui său. Câţi ani au împreună cei trei fraţi?(Clasa a II-a)Maria Racu, IaşiP.219. Un cioban păzea câteva oi şi câteva capre, în total 24 picioare. Câte oi şicâte capre sunt, dacă oile sunt mai multe decât caprele?(Clasa a II-a)Andreea Bîzdîgă, elevă, IaşiP.220. Suma a două numere este 2011. Dacă ştergem cifra miilor unuia dintreele, obţinem celălalt număr. Aflaţi cele două numere.(Clasa a III-a)Mihaela Cianga, IaşiP.221. Suma dintre un număr şi succesorul său este cu 10 mai mare decât predecesorulsău. Calculaţi produsul dintre număr şi vecinii săi.(Clasa a III-a)Petru Miron, PaşcaniP.222. Pe o farfurie sunt cireşe şi vişine. Un copil mănâncă o treime din cireşe şio jumătate din vişine şi constată că are 17 sâmburi. Pot rămâne pe farfurie 20 fructe?dar 34?(Clasa a III-a)Tatiana Ignat, elevă, IaşiP.223. Se consideră numerele naturale x,4x,2x+3,x+2 şi 3x+2, unde x > 2.a) Ordonaţi crescător numerele.b) Dacă notăm cu m cel mai mic număr şi cu M pe cel mai mare, care trebuie săfie valoarea lui x pentru ca şirul m,m+1,...,M să conţină 130 numere?(Clasa a IV-a)Mariana Nastasia, elevă, IaşiP.224. Un elev îşi ţine banii în două buzunare. Dacă ar cheltui un sfert din sumadin primul buzunar şi o doime din cea din al doilea, suma totală s-ar micşora cu 48lei. Care ar fi suma totală dacă s-ar dubla suma din al doilea buzunar?(Clasa a IV-a)Petru Asaftei, IaşiP.225. Aflaţi numerele de trei cifre distincte abc, dacă abc+bca+cab = 666.(Clasa a IV-a)Nicolae Ivăşchescu, Craiova1 Se primesc soluţii până la data de 31 decembrie 2011190


Clasa a V-aV.137. Se consideră numerele naturale A = 1 2010 + 2 2010 + ... + 9 2010 şi B =1 2011 +2 2011 +...+9 2011 . Demonstraţi că B −A se divide cu 10.Mariana Mărculescu şi Dumitru Cotoi, CraiovaV.138. Găsiţi un multiplu al lui 13 a cărui scriere în baza 10 conţine doar cifrede 1.Nicolae Ivăşchescu, CraiovaV.139. Scrieţi numărul 17689 ca diferenţă de două pătrate perfecte.Liviu Smarandache, CraiovaßÞÐ2011ßÞÐ2011ßÞÐ2011ßÞÐ2011ßÞÐ2011V.140. Determinaţi numerele de forma 5abc care, împăţite la abc5, dau câtul de595 ori mai mic decât restul.Petru Asaftei, IaşiV.141. Se consideră numerele naturale a 1 ,a 2 ,...,a n astfel încât a 1 = 1 şi fiecarenumăr, începândcu al doilea, este triplul sumei tuturor numerelordinaintea lui. Dacăa 1 +a 2 +...+a n = 2 20 , determinaţi n.Mirela Marin, IaşiV.142. Arătaţi că numărul A = 11...1122...22+33...3344...44−11...110este pătrat perfect.V.143. Reconstituiţi înmulţireaalăturată, ştiind că literele distinctereprezintă cifre distincte.Andrei Nedelcu, Iaşia b c d e f ×* * * * * *f a b c d ee f a b c dd e f a b cc d e f a bb c d e f aa b c d e f* * * * * * * * * * *Cătălin Calistru, IaşiClasa a VI-aVI.137. Dacă fracţia 3n+7 , n ∈ N, este reductibilă, determinaţi ultima cifră a2n+3lui n.Eugeniu Blăjuţ, BacăuVI.138. Determinaţi numerele prime abc cu proprietatea că dintre cele cincinumere (nu obligatoriu distincte) care se obţin prin permutarea cifrelor, există măcardouă pătrate perfecte.Nicolae Ivăşchescu, CraiovaVI.139. Demonstraţi că 287|(8 15 +8 5 +5)(42 117 +42 39 +39).Bogdan Victor Grigoriu, Fălticeni191


VI.140. Determinaţi numerele întregi nenule n 1 < n 2 < ... < n 7 , dacă 2 n1 +2 n2 +...+2 n7 = 9851024 . Mihai Haivas, IaşiVI.141. Demonstraţi că ecuaţia 12x+15y +20z = 73 nu are soluţii (x,y,z) cutoate componentele numere naturale, dar are o infinitate de soluţii cu toate componentelenumere întregi.Gheorghe Iurea, IaşiVI.142. Fie ABC un triunghi isoscel cu AB = AC. Dacă D este simetricul lui Bfaţă de C şi mediana din B taie AD în E, arătaţi că CE este mediatoareasegmentuluiBD.Silviu Boga, IaşiVI.143. Se consideră triunghiul ABC cu m(ACB) = 45 ◦ şi m(CBA) = 30 ◦ , iarM este un punct pe segmentul AB. Arătaţi că M este mijlocul lui AB dacă şi numaidacă m(MCB) = 15 ◦ .Andrei Paşa, elev, şi Narcisa Paşa, IaşiClasa a VII-aVII.137. Dacă x,y ∈ R, arătaţi că (x 2 +x+1)(y 2 +y +1) ≥ x+y +1.Gheorghiţa Stănică şi Iulian Stănică, Apele Vii (Dolj)VII.138. Dacă a,b,c sunt numere întregi distincte, arătaţi că a 2 +b 2 +c 2 −ab+ac+bc ≥ 1. Când se realizează egalitatea?Elena Iurea, IaşiVII.139. Determinaţi cifrele a,b şi c, dacăaab,b(bc) ∈ N.Romanţa Ghiţă şi Ioan Ghiţă, BlajVII.140. Determinaţi toate perechile (x,y) de numere întregi cu proprietatea că2 x+y (2 x2 +y 2 +1) = 1.Neculai Stanciu, BuzăuVII.141. Dacă ABCD este un patrulater convex, arătaţi că există un unic punctM ∈ (BD) astfel încât triunghiurile ABM şi CDM să fie echivalente.Cecilia Deaconescu, PiteştiVII.142. Determinaţi valoareaminimă a arieiunui paralelogramcircumscrisunuicerc de rază r.Adrian Corduneanu, IaşiVII.143. În interiorul triunghiului ascuţitunghic ABC cu m(A) = 60 ◦ se considerăun punct M astfel încât m(BMC) = 150 ◦ . Un cerc ce trece prin A şi M taie(AB) în Q şi (AC) în R, iar cercul circumscris triunghiului MQB taie (BC) în P.Demonstraţi că triunghiul PQR este dreptunghic.Neculai Roman, Mirceşti (Iaşi)192


Clasa a VIII-aVIII.137. Fie VABCD piramidă patruletară regulată, M şi N mijloacele muchiilorVA respectiv VD, iar P punctul în care înălţimea VO a piramidei înţeapă planul(<strong>MB</strong>C). Arătaţi că VO = 3·OP.Adrian Corduneanu, IaşiVIII.138. Rezolvaţi în R ecuaţia 25·{x} 2 −10x+1 = 0.Bogdan Chiriac, student, IaşiVIII.139. Numerele naturale a 1 ,a 2 ,...,a 100 au proprietatea că N = 6 a1 +6 a2 +...+6 a100 este pătrat perfect. Arătaţi că numărul a 1 +a 2 +...+a 100 se divide cu 5.Andrei Eckstein, TimişoaraVIII.140. Fien ∈ N ∗ şix 1 ,x 2 ,...,x n ∈ Z\{n}astfelîncâtn 3 +x 2 1 +x2 2 +...+x2 n ≤n[1+2(x 1 +x 2 +...+x n )]. Demonstraţi că x 1 ,x 2 ,...,x n ∈ N.Dan Nedeianu, Drobeta Tr. SeverinVIII.141. Dacă a,b,c,x,y,z > 0 şi ax+by+cz = 1, demonstraţi că ayz + bxz +cxy ≥ 27abc.D.M. Bătineţu-Giurgiu, BucureştiVIII.142. Fie a,b,c,d ∈ R şi E(x,y) =Ôa 2 x 2 +b 2 y 2 , F(x,y) =Ôc 2 x 2 +d 2 y 2 ,∀x,y ∈ R. Dacă |ad| = |bc|, demonstraţi căE(x,z)·F(x,z) ≤E(x,y)·F(x,y)+E(y,z)·F(y,z),∀x,y,z ∈ R.a 2011Valentina Blendea şi Gheorghe Blendea, IaşiVIII.143. Dacăa 2 +b 2 = b2011b 2 +c 2 = c2011c 2 +a2, demonstraţi că numerele realepozitive a,b şi c sunt egale.Cristina Ene, elevă, CraiovaClasa a IX-aIX.121. Arătaţi că1+sin 4 x+cos 4 x1+sin 4 y +cos y24= 1+sin8 x+cos 8 x1+sin 8 , ∀x,y ∈ R.y +cos 8 yMihály Bencze, BraşovIX.122. Fie a,b,c ∈ R cu b ≥ c > 0 şi a b ≥ 1+√ 5. Dacă numerele a,b,c pot fi2laturile unui triunghi, demonstraţi că şi 1 a , 1 b , 1 pot fi laturi ale unui triunghi.cOvidiu Pop, Satu MareIX.123. Considerăm patrulaterul ABCD şi fie M,N,P,Q mijloacele laturilorAB,BC,CD respectiv DA, iar T un punct interior patrulaterului. Notăm cu G 1 , G 2 ,G 3 şi G 4 centrele de greutate ale patrulaterelor TNCP,TPDQ, TQAM, T<strong>MB</strong>N.Arătaţi că −−→ AG 1 + −−→ BG 2 + −−→ CG 3 + −−→ DG 4 = −→ 0 dacă şi numai dacă {T} = MP ∩NQ.Florin Stănescu, Găeşti193


IX.124. Dacă ABCD este un patrulater inscriptibil, arătaţi că BC 2 · S ACD +CD 2 ·S ABC = AC 2 ·S BCD .D.M. Bătineţu-Giurgiu, BucureştiIX.125. Dacă x,y ∈ R, x > y > 1, arătaţi că xy +4 > x+3y.Dan Nedeianu, Drobeta Tr. SeverinClasa a X-aX.121. Dacă a ∈ R ∗ + , rezolvaţi în R ecuaţia (2a)x2 ·a x = 2.Luminiţa Mihalache, CraiovaX.122. Demonstraţi că triunghiul ABC este echilateral dacă şi numai dacă h aac +h bab + h cbc = h abc + h bac + h c(notaţiile sunt cele uzuale).abPetru Asaftei, IaşiX.123. Dacă n ∈ N,n ≥ 3 şi x ∈ (−1,1), demonstraţi inegalitatea n( n√ 1+x+n√ 1−x) ≤ 2(√ 1+x+√ 1−x+n−2).Lucian Tuţescu şi Petrişor Rocşoreanu, CraiovaX.124. Aflaţi numerele complexe nenule x,y,z cu proprietatea căx(x+y)(x+z) = y(y +x)(y +z) = z(z +x)(z +y) = −1.Vasile Chiriac, BacăuX.125. Dacăx,y ∈ NsuntastfelîncâtnumărulÔx 2 +2y +1+ 3Ôy 3 +3x 2 +3x+1este raţional, arătaţi că x = y.Gheorghe Iurea, IaşiClasa a XI-asinXI.121. Dat triunghiul ABC, arătaţi că¬1 A sin C 2 2sin A 1 sin B ¬≤ 1 2 2sin C sin B 2 .12 2Bogdan Victor Grigoriu, FălticeniXI.122. Fie A,B ∈ M n (Q) cu A 2 +B 2 = 2I n ; arătaţi că det(AB +BA) ≥ 0.Dumitru Crăciun, FălticeniXI.123. Fie A,B ∈ M 2 (R) astfel încât detA = 10, detB = 12, trA = trB = 7.Determinaţi numerele naturale n pentru care trA n = trB n .Răzvan Ceucă, elev, IaşiXI.124. Calculaţi limn→∞2 20 +Õ2 21 +2 22 +...+ √ 2 2n +1.Gheorghe Iurea, IaşiXI.125. Demonstraţi că ecuaţia 25 x +4 x = 10 x +9 x are cel puţin o soluţie realănegativă.Ionuţ Ivănescu, Craiova194


Clasa a XII-aXII.121. Fie a ∈ N ∗ şi G = (a,+∞) pe care definim operaţia x∗y = (x−a)(y−a) +a, ∀x,y ∈ G. Dacă H este subgrup al lui G astfel încât N ∩G ⊂ H, arătaţi căQ∩G ⊂ H.D.M. Bătineţu-Giurgiu, Bucureşti şi Neculai Stanciu, BuzăuXII.122. Fie n ∈ N, n ≥ 2 şi polinomul f = X n −2nX n−1 +(2n 2 −4)X n−2 +a 3 X n−3 +...+a n ∈ C[X]. Demonstraţi că f are toate rădăcinile reale dacă şi numaidacă n = 2.Florin Stănescu, Găeşti√659XII.123. Calculaţi I =arccostgx √ sinxdx.0Vasile Chiriac, BacăuXII.124. Fie f : R → R o funcţie continuă cu proprietatea că (f ◦f)(x) = sinx,∀x ∈ R. Demonstraţi că1f(x)dx ≤ 1.0Dumitru Crăciun, FălticeniXII.125. Fie f : [0,1] → R o funcţie derivabilă cu f ′ integrabilă. Dacă f(0) = 0,arătaţi că1(f ′ (t)) 2 dt ≥1f 2 (t)dt.00Ciprian Baghiu, IaşiProbleme pentru pregătirea concursurilorA. Nivel gimnazialG206. Câte submulţimi ale mulţimii M = {1,2,3,...,100} au 50 de elemente şinu conţin nicio pereche de numere consecutive?Gheorghe Iurea, IaşiG207. Arătaţi că numărul N = 2 2009 +3 2010 +4 2011 nu este pătrat perfect.Andrei Eckstein, TimişoaraG208. Demonstraţi că ecuaţia x 2 +y 2 = z(x+y+1) are o infinitate de soluţii înmulţimea numerelor naturale.Cosmin Manea şi Dragoş Petrică, PiteştiG209. Rezolvaţi în numere naturale ecuaţia 4abc = (a+2)(b+1)(c+1).Titu Zvonaru, ComăneştiG210. Demonstraţi că fracţia a3n+2 −a 3n+1 +(−1) na 3n+8 −a 3n+7 este reductibilă pentru+(−1)norice a,n ∈ N, a ≥ 2.Dan Popescu, Suceava195


G211. Demonstraţi că expresia22 (a 1 +a 2 )+x 3 a 1E = y 1xx 1 +x 2 +x 31 a 1 −x 2 a 2 1+y 3x− ·x 1 +x 2 +x 32y 1 +y 2 +y 321 (a 1 +a 2 )+x 3 a 2+y 2x+x 1 +x 2 +x 3x 2 (a 1 +a 2 )+x 3 a 1 x 1 (a 1 +a 2 )+x 3 a 2 x 1 a 1 −x 2 a 2·å−y 1 +y 2 +y 3 ,x 1 +x 2 +x 3 x 1 +x 2 +x 3 x 1 +x 2 +x 3è2unde a i ,x i ,y i ∈ R ∗ +(i = 1,2,3), nu depinde x 1 ,x 2 ,x 3 .Mircea Bîrsan, IaşiG212. Se considerătriunghiul ABC cu m(B) = 135 ◦ şim(C) = 30 ◦ . Determinaţimăsurile unghiurilor triunghiului ABD, unde D este simetricul lui C faţă de B.Eugeniu Blăjuţ, BacăuG213. Se consideră triunghiul ABC cu proprietatea că există M şi N puncteîn interiorul său astfel încât BN = CM şi △ABM ∼ △ACN. Demonstraţi căAB = AC.Crisitan Lazăr, IaşiG214. Se consideră triunghiul isoscel ABC cu AB = AC şi m(A) < 90 ◦ . Construimînălţimea CF şi fie E mijlocul segmentului BF, iar D un punct pe segmentulBC. DacăADE ≡B, arătaţi că D este mijlocul segmentului BC.Claudiu Ştefan Popa şi Gabriel Popa, IaşiG215. În planele paraleleP 1 şi P 2 se considerăcercurile C 1 = C(O 1 ,R 1 ), respectivC 2 = C(O 2 ,R 2 ). Fie K 1 conul de vârf O 2 şi bază C 1 şi K 2 conul de vârf O 1 şi bază C 2 .Arătaţi că intersecţia celor două conuri este un cerc şi determinaţi poziţia centruluişi mărimea razei acestuia.Temistocle Bîrsan, IaşiB. Nivel licealL206. Fie P un punct pe mediana din A a triunghiului ABC. Paralela prin Pla AC taie AB în M, iar simetricul lui P faţă de mijlocul lui AC este N. Arătaţi căMN‖BC dacă şi numai dacă P este centrul de greutate al triunghiului ABC.Silviu Boga, IaşiL207. Fie ABCD un patrulater convex şi M,N,P puncte pe segmentele AB,CDrespectiv BC astfel încât <strong>MB</strong>AB = NDDC = BP = k. Dacă R şi S sunt mijloaceleBCsegmentelor AP respectiv MN, calculaţi (în funcţie de k) raportul RSAD .Titu Zvonaru, ComăneştiL208. Un cilindru circular drept de axă d şi rază R 1 şi o sferă de centru O şi razăR 2 sunt tangente exterior în punctul A. Fie B simetricul lui A în raport cu d şi fieπ planul ce trece prin B, este perpendicular pe planul determinat de O şi d şi facecu axa d un unghi de 30 ◦ . Calculaţi raportul razelor celor două suprafeţe ştiind căsecţiunile lor cu planul π au arii egale.Temistocle Bîrsan, Iaşi196


L209. Se consideră triunghiul ABC şi punctele M,N,P,Q,R,S definite prin−−→BM = k · −−→ MC, −−→ CN = k · −−→ NA, −→ −→ −−→ −−→ −−→ −−→AP = k · PB, AM = p · MQ, BN = p · NR,−→CP = p·−→ PS, unde k,p ∈ R ∗ \{−1}. Demonstraţi că S MNP ≥ 1 4 ·S ABC, iar S QRS ≥p+3·S ABC .2p2Marius Olteanu, Rm. VâlceaL210. Cercul A-exînscris triunghiului ABC este tangent prelungirilor laturilorAB şi AC în P, respectiv Q. Bisectoarele ≤È exterioare ale unghiurilor B şi C intersecteazădreapta PQ în S respectiv T. Demonstraţi că PQ ≤ ST +BC.Titu Zvonaru, Comăneştisin 3 xL211. Arătaţi că(1+sin 2 x) + cos 3 x2 (1+cos 2 x) 2 ≤ 3√ 3, ∀x ∈ R.16Mihàly Bencze, BraşovL212. Demonstraţică 3 +Èab (ab+c 2 ) 22 a 2 +b 2 (a 2 +c 2 )(b 2 +c 2 ) (sumelefiindciclice)pentru orice numere reale a,b,c printre care nu se găsesc două egale cu 0.Marian Tetiva, BârladL213. Fie m 1 ,...,m k numere naturale nenule şi α un număr iraţional.a) Arătaţi că există x 1 ,...,x k ∈ N ∗ astfel încât [x 1α]= ... = [x kα].m 1 m kb) Arătaţi că există y 1 ,...,y k ∈ N ∗ astfel încât m 1 [y 1 α] = ... = m k [y k α].Marian Tetiva, BârladL214. Fie A ∈ M n (R) o matrice simetrică al cărei polinom caracteristic este X n .Arătaţi că A este matricea nulă.Marian Tetiva, BârladL215. Avem la dispoziţie 2n+1 pietricele (n ≥ 1) astfel încât orice submulţimede 2n pietricele poate fi împărţită în două grămezi de câte n pietricele având aceeaşimasă totală. Demonstraţi că toate pietricelele au aceeaşi masă.Adrian Reisner, ParisTraining problems for mathematical contestsA. Junior highschool levelG206. How many subsets of the set M = {1,2,3,...,100} have 50 elements anddo not contain any pair of successive numbers ?Gheorghe Iurea, IaşiG207. Show that the number N = 2 2009 +3 2010 +4 2011 is not a perfect square.Andrei Eckstein, TimişoaraG208. Show that the equation x 2 +y 2 = z(x+y+1) has infinitely many solutionsin the set of natural numbers.Cosmin Manea and Dragoş Petrică, Piteşti197


G209. Solve, in the set of natural numbers, the equation 4abc = (a + 2)(b +1)(c+1).Titu Zvonaru, ComăneştiG210. Prove that that the fraction a3n+2 −a 3n+1 +(−1) na 3n+8 −a 3n+7 is reducible for any+(−1)nnumbers a,n ∈ N, a ≥ 2.Dan Popescu, SuceavaG211. Show that the expression22 (a 1 +a 2 )+x 3 a 1E = y 1xx 1 +x 2 +x 31 a 1 −x 2 a 2 1+y 3x− ·x 1 +x 2 +x 32y 1 +y 2 +y 321 (a 1 +a 2 )+x 3 a 2+y 2x+x 1 +x 2 +x 3x 2 (a 1 +a 2 )+x 3 a 1 x 1 (a 1 +a 2 )+x 3 a 2 x 1 a 1 −x 2 a 2·å−y 1 +y 2 +y 3 ,x 1 +x 2 +x 3 x 1 +x 2 +x 3 x 1 +x 2 +x 3è2where a i ,x i ,y i ∈ R ∗ +(i = 1,2,3), does not depend of x 1 ,x 2 ,x 3 .Mircea Bîrsan, IaşiG212. Let ABC be a triangle with m(B) = 135 ◦ and m(C) = 30 ◦ . Determinethe measures of the angles of triangle ABD, where D is the symmetric of point Cwith respect to point B.Eugeniu Blăjuţ, BacăuG213. Let ABC be a triangle with the property that two points M and N existin its interior such that BN = CM and △ABM ∼ △ACN. Show that AB = AC.Crisitan Lazăr, IaşiG214. The isosceles triangle ABC is considered, with AB = AC and m(A)


and S are the midpoints of segments AP, respectively MN, calculate (as a functionof k) the ratio RSAD .Titu Zvonaru, ComăneştiL208. A right circular cone of axis d and radius R 1 , and a sphere of center O andradius R 2 , are exterior-tangent at the point A. Let B be the symmetric of A withrespect to d and let π be the plane that passes through B, is perpendicular on theplane determined by O and d and forms an angle of 30 ◦ with axis d. Calculate theratio between the radii of the two surfaces knowing that their sections through theplane π have equal areas.Temistocle Bîrsan, IaşiL209. ItisconsideredthetriangleABC andthepointsM,N,P,Q,R,S definedby−−→BM = k·−−→ MC, −−→ CN = k·−−→ NA, −→ AP = k·−−→ PB, −−→ AM = p·−−→ MQ, −−→ BN = p·−−→ −→NR, CP = p·−→ PS,where k,p ∈ R ∗ \{−1}. Prove that S MNP ≥ 1 4 ·S ABC, and S QRS ≥p+3·S ABC .2p2Marius Olteanu, Rm. VâlceaL210. The A-excircle to the triangle ABC is tangent to the prolongations of thesides AB and AC at P, respectively ≤È Q. The exterior angle bisectors of B and Cintersect the line PQ at S, respectively T. Prove that PQ ≤ ST +BC.Titu Zvonaru, Comăneştisin 3 xL211. Show that(1+sin 2 x) + cos 3 x2 (1+cos 2 x) 2 ≤ 3√ 3, ∀x ∈ R.16Bencze Mihàly, BraşovL212. Show that 3 +Èab (ab+c 2 ) 22 a 2 +b 2 (a 2 +c 2 )(b 2 +c 2 (the sums being cyclic))for any real numbers a,b,c with no two numbers equal to 0 among them.Marian Tetiva, BârladL213. Let m 1 ,...,m k be nonzero natural numbers and α an irrational number.a) Show that x 1 ,...,x k ∈ N ∗ exist such that [x 1α]= ... = [x kα].m 1 m kb) Show that y 1 ,...,y k ∈ N ∗ exist such that m 1 [y 1 α] = ... = m k [y k α].Marian Tetiva, BârladL214. Let A ∈ M n (R) be a symmetric matrix whose characteristic polynomial isX n . Show that A is the null matrix.Marian Tetiva, BârladL215. We have at our disposal 2n+1 small stones (n ≥ 1) such that any subsetof small stones can be divided into two heaps of n small stones each and having thesame total sum. Prove that all the small stones have the same mass.Adrian Reisner, Paris199


Pagina rezolvitorilorCÂMPULUNG MUSCELLiceul Naţional cu program de atletism. Clasa a XI-a (prof. POPESCU Ionela).GRUBINSCHI Anişoara-Marilena: IX(116,117), X.117, XI.117, XII.116.CRAIOVAColegiul Naţional ”Fraţii Buzeşti”. Clasa a IX-a (prof. TUŢESCU Lucian).GOLEA Monica: VIII(132, 133), IX(116,117,119), X.117.IAŞIŞcoala nr. 3 ”Al. Vlahuţă”. Clasa a III-a (inst. MAXIM Gabriela). CARACAŞSorin: P(206-210,213);DASCĂLULorena: P(206-210,213);NICAIoan-Daniel: P(206-210,213);ROBUCarmen: P(206-210,213);ŞERBĂNOIUAlexandru: P(206-210,213).Clasa a IV-a (înv. MĂRIUŢĂ Valentina). LUNGU Lucian: P(208,209,211-214);HERGHILIGIU-HENEA Mălina: P(208,209,211-214); HREAPCĂ Alin: P(208,209,211-214); HUHU Paula: P(209,211-214); PEREŞ Cristi: P(208,209,211-214); POPO-VICI Teodor-Andrei: P(208,209,211-214); ROMILĂ Andreea-Maria: P(208,209,211-214). Clasa a V-a(prof. MARINMirela). CIOCOIUAlexandra: P(214,215),V(130-132); MARIN Marius: P(214,215), V(130-132); VLAD Ioana: P(214,215), V(130-132). Clasa a VI-a (prof. MARIN Mirela). CREŢU Cristiana-Paula: V(130-132),VI(131,134); GĂINĂ Bianca: V(130-132), VI(131,134); GLIGA Dumitru: V(130-132), VI(131,132). Clasa a VIII-a (prof. MARIN Mirela). ASAVEI Alexandra:VI(131,132,134), VII.134, VIII.131; CELMARE Raluca-Iuliana: VI(131,132,134),VII(130,132-134,136),VIII(131,132);TIBAŞtefana-Alexandra: VII(130,133,134,136),VIII(131,132).Şcoala nr. 11 ”Otilia Cazimir”. Clasa a III-a (înv. MARDARE Carmen).OLENIUC Iulian: P(206-212). Clasa a IV-a (inst. PÂRÂIALĂ Dumitru). POPAIoana-Maria: P(196-215), V(131-134).Şcoala nr. 26 ”George Coşbuc”. Clasa a II-a (inst. RACU Maria). CIOPEICĂSebastian-Andrei: P(206-210); GROSU Victor-Alessandru: P(206,207,209-211); LU-CHIAN Maria-Clara: P(206-210); PASNICU Cosmin-Constantin: P(206-210); RĂI-LEANU Răzvan-Constantin: P(206-210); VASILE Raluca-Andreea: P(206-210).Colegiul Naţional Iaşi. Clasa a V-a (prof. POPA Gabriel). OBADĂ Ştefan:V(124,126-128,131,135,136), VI(123,134). Clasa a VI-a (prof. POPA Gabriel).BERBINSCHITudor: V(123-129,131-136),VI(123-132,134-136),VII(125,126),G(186,187,191,193,196-198); DOMINTE Ştefan: V(123-129,131-133,135), VI(123-125,128,134-136), VII.125, G(187,196); PALAGHIU Horaţiu: VI(123-136). Clasa a VIII-a(prof. POPA Gabriel). STOLERU Ingrid: VII(130-136), VIII(124,130,131), G(187,189,190,193,195-198,205). Clasa a XI-a (prof. POPA Gabriel). CEUCĂ Răzvan:X(111-113,115,117-119), XI(111-114,116-118), XII.111.PAŞCANIŞcoala ”Iordachi Cantacuzino”. Clasa a III-a (înv. MIRON Petru). CRĂCIUNŞtefana-Maria: P(206-212), V.113.200


ŢIGĂNAŞI (IAŞI)Şcoala cu clasele I-VIII ”M. Kogălniceanu”. Clasa I (înv. SAMSON Daniel-Mihai). DUCADaria-Călina-Stela: P(206-210);Clasa a II-a(prof.înv.primarBĂDI-ŢĂAurica). DUCAEmaŞtefania: P(206-210). ClasaaIV-a(înv. GALIAParaschiva).CAZADOI Ioana-Cristina: P(206-210,213,214);DUCAMihaela-Cristina: P(206-210,213,214); ILIOAE Mihai-Iulian: P(206-210,213,214);PIU Naomi Ştefana: P(206-210,213,214); SANDU Ana Paula: P(206-210,213,214); SANDU Rebeca: P(206-210,213,214).Au mai trimis redacţiei un număr mai mic de cinci probleme corect rezolvateelevii următori: (clasa a V-a) Duca Cristina Diana, Găină Raluca Mihaela, GioancăBianca Maria, Piu Raluca Diana, Toma Evelina; (clasa a VI-a) Duca Mirela Beatrice,Găneanu Tudor, Ghioancă Ştefan, Sandu Codruţ, Verner Mădălina Georgiana; (clasaa VII-a) Iacob Oana Alisa, Piu Mioara Florentina, Tănase Mădălina; (clasa a VIII-a)Ghioancă Andreea Lavinia, Ghioancă Claudia Oana.Elevi rezolvitori premiaţi1. ASAVEI Alexandra(clasaaVIII-a): 1/2010(5pb),2/2010(5pb),1/2011(7pb);2. CARACAŞ Sorin (clasa a III-a): 2/2010(5pb), 1/2011(5pb), 2/2011(6pb);3. CELMARE Raluca-Iuliana (clasa a VIII-a): 1/2010(5pb), 2/2010(5pb),1/2011(11pb);4. CIOPEICĂ Sebastian-Andrei (clasa a II-a): 2/2010(5pb), 1/2011(5pb),2/2011(5pb);5. CRĂCIUN Ştefana-Maria (clasa a III-a): 1/2010(10pb), 2/2010(6pb),1/2011(9pb);6. CREŢU Cristiana-Paula (clasa a VI-a): 1/2010(5pb), 2/2010(5pb),1/2011(5pb);7. GROSU Victor-Alessandru (clasa a II-a): 2/2010(5pb), 1/2011(5pb),2/2011(5pb);8. IFTIME Ioana Evelina(clasaaVI-a): 1/2010(6pb),2/2010(5pb),1/2011(7pb);9. LUCHIAN Maria-Clara (clasa a II-a): 2/2010(5pb), 1/2011(5pb),2/2011(5pb);10. PASNICU Cosmin-Constantin (clasa a II-a): 2/2010(5pb), 1/2011(5pb),2/2011(5pb);11. PEREŞ Cristi (clasa a IV-a): 2/2010(6pb), 1/2011(7pb), 2/2011(6pb);12. RĂILEANU Răzvan-Constantin(clasaaII-a): 2/2010(5pb),1/2011(5pb),2/2011(5pb);13. SOFIAN Elena Marina (clasa a VI-a): 1/2010(5pb), 2/2010(5pb),1/2011(5pb).201


COLECŢIA ”RECREAŢII <strong>MATEMATICE</strong>”Asociaţia ”Recreaţii Matematice” iniţiază şi stimuleazaă publicarea de cărţide matematică elementară adresate tuturor iubitorilor acestui domeniu. Condiţiilede publicare sunt specificate în regulamentul colecţiei (care poate fi solicitat la adresat birsan@yahoo.com).Primul număr al Colecţiei ”Recreaţii Matematice”, aflat în curs de tipărireeste1. D. Brânzei, Al. Negrescu –Probleme de pivotare.IMPORTANT• În scopulunei legăturirapidecu redacţiarevistei, pot fi utilizateurmătoareleadrese e-mail: t birsan@yahoo.com şi profgpopa@yahoo.co.uk. Peaceastă cale colaboratorii pot purta cu redacţia un dialog privitor la materialeletrimise acesteia, procurarea numerelor revistei etc. Sugerăm colaboratorilorcare trimit probleme originale pentru publicare să le numerotezeşi să-şi reţină o copie xerox a lor pentru a putea purta cu uşurinţă o discuţieprin e-mail asupra acceptării/neacceptăriiacestorade către redacţia revistei.• La problemele de tip L se primesc soluţii de la orice iubitor de matematicielementare (indiferent de preocupare profesională sau vârstă). Fiecare dintresoluţiile acestor probleme - ce sunt publicate în revistă după un an - va fiurmată de numele tuturor celor care au rezolvat-o.• Adresăm cu insistenţă rugămintea ca materialele trimise revisteisă nu fie (să nu fi fost) trimise şi altor publicaţii.• Rugăm ca materialele tehnoredactate să fie trimise pe adresa redacţieiînsoţite de fişierele lor (de preferinţă în L A TEX).• Pentru a facilita comunicarea redacţiei cu colaboratorii ei, autorii materialelorsunt rugaţi să indice adresa e-mail.202


<strong>Revista</strong> semestrială RECREAŢII <strong>MATEMATICE</strong> este editată de ASOCIAŢIA“RECREAŢII <strong>MATEMATICE</strong>”. Apare la datele de 1 martie şi 1 septembrie şi seadresează elevilor, profesorilor, studenţilor şi tuturor celor pasionaţi de matematicaelementară.În atenţia tuturor colaboratorilorMaterialele trimise redacţiei spre publicare (note şi articole, chestiuni de metodică,probleme propuse etc.) trebuie prezentate îngrijit, clar şi concis; ele trebuie să prezinteinteres pentru un cerc cât mai larg de cititori. Se recomandă ca textele să nu depăşeascăpatru pagini. Evident, ele trebuie să fie originale şi să nu fi apărut sau să fi fosttrimise spre publicare altor reviste. Rugăm ca materialele tehnoredactate să fieînsoţite de fişierele lor.Problemele destinate rubricilor: Probleme propuse şi Probleme pentrupregătirea concursurilor vor fi redactate pe foi separate cu enunţ şi demonstraţie/rezolvare(câte una pe fiecare foaie) şi vor fi însoţite de numele autorului, şcoala şilocalitatea unde lucrează/învaţă.Redacţia va decide asupra oportunităţii publicării materialelor primite.În atenţia elevilorNumele elevilor ce vor trimite redacţiei soluţii corecte la problemele din rubricilede Probleme propuse şi Probleme pentru pregatirea concursurilor vor fi menţionateîn Pagina rezolvitorilor. Elevii menţionaţi de trei ori vor primi o diplomă şi unpremiu în cărţi. Elevii rezolvitori vor ţine seama de regulile:1. Pot trimite soluţii la minimum cinci probleme propuse în numărul prezentşi cel anterior al revistei (pe o foaie va fi redactată o singură problemă).2. Elevii din clasele VI-XII au dreptul să trimită soluţii la problemele propusepentru clasa lor, pentru orice clasă mai mare, din două clase mai mici şi imediatanterioare. Cei din clasa a V-a pot trimite soluţii la problemele propuse pentru clasele aIV-a, a V-a şi orice clasă mai mare, iar elevii claselor I-IV pot trimite soluţii laproblemele propuse pentru oricare din clasele primare şi orice clasă mai mare. Oriceelev poate trimite soluţii la problemele de concurs (tip G şi L).3. Vor fi menţionate următoarele date personale: numele şi prenumele, clasa,şcoala şi localitatea, precum şi numele profesorului cu care învaţă.4. Plicul cu probleme rezolvate se va trimite prin poştă (sau va fi adus direct) laadresa Redacţiei:Prof. dr. Temistocle BîrsanStr. Aurora, nr. 3, sc. D, ap. 6,700 474, IaşiJud. IAŞIE-mail: t_birsan@yahoo.com


CUPRINSPetru MINUŢ (1936-2011) ................................................................................................. 99Congresul al VII-lea al matematicienilor români de pretutindeni (C. Corduneanu) ....... 101Profesorul ION CREANGĂ – o sută de ani de la naşterea sa (T. Bîrsan)........................ 103SPIRU HARET (1851-1912) – contribuţia ştiinţifică (V. Oproiu) .................................... 105ARTICOLE ŞI NOTET. ZVONARU, N. STANCIU – Alte proprietăţi caracteristicetriunghiului echilateral .................... 108D.M. BĂTINEŢU-GIURGIU, N. STANCIU – Inegalităţi geometriceîn poligoane convexe, de tip Bergström-Mitrinovič ................... 112D.Şt. MARINESCU, I. ŞERDEAN – În legătură cu o problemă de concurs ...................... 116L. TUŢESCU, M. DICU – Aplicaţii ale inegalităţii mediilorîn rezolvarea unor probleme de minim ................... 119NOTA ELEVULUID. DĂNĂILĂ – Asupra unei inegalităţi „aproape clasică” ..................................................... 121CORESPONDENŢEA. REISNER – Autour du cardinal d'un ensemble de matrices binaires ................................ 124CUM CONCEPEM ... CUM REZOLVĂMN. ROMAN – Comentarii pe marginea unei probleme .......................................................... 129CHESTIUNI COMPLEMENTARE MANUALELORI. PĂTRAŞCU – Proprietăţi ale triunghiurilor în care 3a = b + c ......................................... 132ŞCOLI ŞI DASCĂLIM. TETIVA – Colegiul Naţional „Gheorghe Roşca Codreanu” din Bârlad ....................... 134CONCURSURI ŞI EXAMENEConcursul de matematică "Al. Myller", ed. a IX-a, 2011 ................................................. 137Concursul de matematică "Florica T. Câmpan", ed. a XI-a, 2011 .................................... 140PROBLEME ŞI SOLUŢIISoluţiile problemelor propuse în nr. 2/2010 ......................................................................... 148Soluţiile problemelor propuse în nr. 1/2011 ......................................................................... 164Soluţiile problemelor pentru pregătirea concursurilor propuse în nr. 1/2011 ....................... 181Probleme propuse ..................................................................................................................... 190Probleme pentru pregătirea concursurilor ............................................................................... 195Training problems for mathematical contests ........................................................................ 197Pagina rezolvitorilor ................................................................................................................ 200Elevi rezolvitori premiaţi ......................................................................................................... 201ISSN 1582 – 17658 lei

Hooray! Your file is uploaded and ready to be published.

Saved successfully!

Ooh no, something went wrong!